Prep-U Cancer& MS

Pataasin ang iyong marka sa homework at exams ngayon gamit ang Quizwiz!

A patient is scheduled for cryosurgery for cervical cancer and tells the nurse, "I am not exactly sure what the doctor is going to do." What is the best response by the nurse?

"The physician is going to use liquid nitrogen to freeze the area."

The nurse is teaching a client with a left fractured tibia how to walk with crutches. Which instruction is appropriate?

All weight should be on the hands.

Which of the following disorders results in widespread hemorrhage and microthrombosis with ischemia?

Disseminated intravascular coagulation (DIC)

When describing malignant bone tumors to a group of students, which of the following would the instructor cite as the usual location? a) Distal femur around the knee b) Wrist-hand junction c) Proximal humerus d) Femur-hip area

Distal femur around the knee Explanation: Malignant bone tumors usually are located around the knee in the distal femur or proximal fibula; a few are found in the proximal humerus. The wrist-hand junction and femur-hip area are not common sites.

While a nurse on the orthopedic floor is administering medications to her group of clients, the physical therapy department calls to inform her that two of her clients are late for their scheduled session. What should the nurse do?

First, administer scheduled medications to the clients who require physical therapy, and then have them attend therapy.

Which of the following actions displayed by a grieving husband over his dying wife would cause the nurse to suggest counseling?

He refuses to acknowledge his wife's family and blames them for her current health problems.

Assessment of the musculoskeletal system usually proceeds from general to specific and from?

Head to toe

An oncology clinic nurse is reinforcing prevention measures for oropharyngeal infections to a client receiving chemotherapy. Which statement by the client indicates an understanding of the nurse's teaching?

I clean my teeth gently several times a day.

Radiographic evaluation of a client's fracture reveals that a bone fragment has been driven into another bone fragment. The nurse identifies this as which type of fracture?

Impacted

A nurse is administering an antineoplastic extravasation occurs. How can the nurse best prevent tissue damage caused by extravasation?

Inspect the site frequently for redness or swelling

A nursing student asks the instructor how to identify rheumatoid nodules in a client with rheumatoid arthritis. Which of the following characteristics would the instructor include?

Located over bony prominence

An oncology nurse educator is speaking to a women's group about breast cancer. Questions and comments from the audience reveal a misunderstanding of some aspects of the disease. Various members of the audience have made all of the following statements. Which statement is accurate?

Men can develop breast cancer.

Which agent would the nurse expect to be administered orally?

Methotrexate

In which phase of the cell cycle does cell division occur?

Mitosis

"Which intervention is appropriate for the nurse caring for a client in severe pain receiving a continuous I.V. infusion of morphine?

Obtaining baseline vital signs before administering the first dose

Hip fracture indication

One leg longer than the other may indicate a hip fracture

Which finding is an early indicator of bladder cancer?

Painless hematuria

A client has been receiving chemotherapy to treat cancer. Which assessment finding suggests that the client has developed stomatitis?

Red, open sores on the oral mucosa

A postmenopausal woman with breast cancer will most likely be treated with which anti-estrogen drug?

Tamoxifen

The radioulnar joint.

The radioulnar is at the radius and ulna.

After receiving chemotherapy for lung cancer, a client's platelet count falls to 98,000/?l. What term should the nurse use to describe this low platelet count?

Thrombocytopenia

A client diagnosed with leukemia asks the nurse why he must undergo bone marrow aspiration. Which response by the nurse is best?

To determine what type of leukemia you have; your physician will explain the procedure to you in more detail.

Which of the following would be inconsistent as a common side effect of chemotherapy?

Weight gain

A woman tells the nurse, "There has been a lot of cancer in my family." The nurse should instruct the client to report which possible sign of cervical cancer?

light bleeding or watery vaginal discharge

On discharge, a client who underwent left modified radical mastectomy expresses relief that "the cancer" has been treated. When discussing this issue with the client, the nurse should stress that she:

should continue to perform breast self-examination on her right breast.

A client with a diagnosis of cancer is frequently disruptive and challenges the nurse. This behavior may be caused by:

uncertainty and an underlying fear of recurrence

The nurse is caring for a client following foot surgery. Which nursing intervention is most important for the nurse to include in the nursing care plan?

Perform neuromuscular assessment every hour.

The nurse is caring for a client who recently underwent a total hip replacement. The nurse should:

instruct the client to limit hip flexion while sitting.

The nurse is developing a teaching plan for a client who must undergo an above-the-knee amputation of the left leg. After a leg amputation, exercise of the remaining limb:

should begin the day after surgery.

arthritis

Redness, heat, and swelling may be seen with bursitis of the olecranon process due to trauma or arthritis

A client is being discharged from the Emergency Department after being diagnosed with a sprained ankle. Which client statement indicates the client understands the discharge teaching?

"I'll make sure to keep my ankle elevated as much as possible."

A decrease in circulating white blood cells is 1. neutropenia. 2. granulocytopenia. 3. thrombocytopenia. 4. leukopenia.

4

A client is to undergo surgery to repair a ruptured Achilles tendon and application of a brace. The client demonstrates understanding of his activity limitations when he states that he will need to wear the brace for which length of time?

6 to 8 weeks

The oncology nurse is giving chemotherapy to a client in a short stay area. The client confides that they are very depressed. The nurse recognizes depression as which of the following?

A normal reaction to the diagnosis of cancer.

Of the following, which is not a risk factor for osteoporosis?

Being male

When an infection is blood borne the manifestations include which of the following symptoms?

Chills

A client with suspected cervical cancer is undergoing a colposcopy with conization. The nurse gives instructions to the client about her menstrual periods, emphasizing that:

Her next two or three periods may be heavier and more prolonged than usual. The client should be informed that her next two or three periods could be heavy and prolonged. The client is instructed to report any excessive bleeding. The nurse should reinforce the necessity for the follow-up check and the review of the biopsy results with the client. The client's periods will not be normal for 2 to 3 months.

Which of the following is a factor that inhibits fracture healing?

History of diabetes

A client, age 42, visits the gynecologist. After examining her, the physician suspects cervical cancer. The nurse reviews the client's history for risk factors for this disease. Which history finding is a risk factor for cervical cancer?

Human papillomavirus infection at age 32

A nurse is caring for a client who sustained a gunshot wound to the leg during a jewelry store robbery. The client is in police custody and receiving treatment in the emergency department. A member of the media asks the nurse about the client's condition. How should the nurse respond?

Notify the nursing supervisor so she can obtain a formal statement from the physician about the client's condition for the media.

"The nurse is caring for a client who has just had a modified radical mastectomy with immediate reconstruction. She's in her 30s and has two young children. Although she's worried about her future, she seems to be adjusting well to her diagnosis. What should the nurse do to support her coping?

Refer the client to the American Cancer Society's Reach for Recovery program or another support program.

To assess extension of the shoulders and arms, a nurse should ask a client to:

asking the client to move the arms backward elicits extension

A client receiving radiation therapy for bladder cancer tells the nurse " I feel as if I am voiding through my vagina." The nurse interprets this condition as. a. Altered perineal sensation as a side effect of radiation therapy b. The development of a vesicovaginal fistula c. Extreme stress caused by the diagnosis of cancer d. Rupture of the bladder

b. The development of a vesicovaginal fistula

The oncology nurse understands that chemotherapeutic agents are most effective during a particular phase of the cell cycle. This means the agents are:

cell cycle-specific.

A nurse is caring for a female client following a motor vehicle accident resulting in paraplegia. The client is ready for discharge to home with her husband, who states, "I'm scared to carry her because I'm afraid I'll either hurt my back or drop her." A nurse identifies a need for discharge teaching of the husband in regard to:

ergonomic principles and body mechanics.

Joints may be classified as

fibrous, cartilaginous, or synovial

A nurse suspects that a client with a recent fracture has compartment syndrome. Assessment findings may include:

inability to perform active movement and pain with passive movement.

The client asks the nurse to explain what it means that his Hodgkin's disease is diagnosed at stage 1A. What describes the involvement of the disease?

involvement of a single lymph node In the staging process, the designations A and B signify that symptoms were or were not present when Hodgkin's disease was found, respectively. The Roman numerals I through IV indicate the extent and location of involvement of the disease. Stage I indicates involvement of a single lymph node; stage II, two or more lymph nodes on the same side of the diaphragm; stage III, lymph node regions on both sides of the diaphragm; and stage IV, diffuse disease of one or more extralymphatic organs.

Before a client undergoes arthroscopy, the nurse reviews the data collection findings for contraindications for this procedure. Which finding is a contraindication?

Joint flexion of less than 50%

When performing a physical assessment of a client, the client reports numbness, tingling, and pain when the nurse percusses lightly over the median nerve. The nurse recognizes that this finding is consistent with:

Carpal tunnel syndrome

Which common problem of the upper extremity results from entrapment of the median nerve at the wrist?

Carpal tunnel syndrome

Which of the following would lead a nurse to suspect that a client has a rotator cuff tear?

Difficulty lying on affected side

Which of the following would a nurse encourage a client with gout to limit?

Purine-rich foods

Tinel's test

to determine the presence of carpel tunnel syndrome, Percuss lightly on the inner aspect of the wrist

(SELECT ALL THAT APPLY) What diagnostic studies are recommended to be performed annually on all women older than age 40?

(1) Mammography (4) Clinical breast examination

A young female client has received chemotherapeutic medications and asks about any effects the treatments will have related to her sexual health. The most appropriate statement by the nurse is

"You will need to practice birth control measures."

Which group is at the greatest risk for osteoporosis?

Caucasian women

Morse Fall Scale

*High fall risk = score of greater than 50*. Standard fall precautions initiated for all patients; fall prevention protocol initiated for high-risk patients 0-24 is no risk, 25-50 low risk, 51 and over is high risk

After a transsphenoidal adenohypophysectomy, a client is likely to undergo hormone replacement therapy. A transsphenoidal adenohypophysectomy is performed to treat which type of cancer?

Pituitary carcinoma

When an infection is blood borne the manifestations include which of the following symptoms? a) Hypothermia b) Hyperactivity c) Chills d) Bradycardia

Chills Explanation: Manifestations include chills, high fever, rapid pulse, and generalized malaise.

Ms. Bixley has been gardening all season and has come to the clinic with a complaint of shoulder pain. The physician assesses the shoulder and decided that she probably has a rotator cuff tear. What are the possible treatment options for treating this condition? Select all that apply.

-NSAIDs -Modify activities and rest the joint -Arthroscopic surgery

Sarah Butler, age 26 years, has been participating in a tennis tournament. She is being seen in the clinic for elbow discomfort. The physician has given her a diagnosis of tendinitis, epicondylitis, or tennis elbow. What symptoms and signs did she have? Choose all that are correct.

-Pain radiating down the dorsal surface of the forearm -Weak grasp

According to the tumor-node-metastasis (TNM) classification system, T0 means there is which of the following? 1. No evidence of primary tumor 2. Distant metastasis 3. No distant metastasis 4. No regional lymph node metastasis

1

A client receiving chemotherapy for breast cancer develops myelosuppression. Which instructions should the nurse expect in the discharge teaching? Select all that apply. 1. Avoid people who have recently received attenuated vaccines. 2. Avoid activities that may cause bleeding. 3. Wash hands frequently. 4. Increase intake of fresh fruits and vegetables. 5. Avoid crowded places such as shopping malls. 6. Treat a sore throat with OTC medications.

1. Avoid people who have recently received attenuated vaccines. 2. Avoid activities that may cause bleeding. 3. Wash hands frequently. 5. Avoid crowded places such as shopping malls.

The nurse is developing a plan of care for the client with multiple myeloma and includes which priority intervention in the plan? 1. Encouraging fluids 2. Providing frequent oral care 3. Coughing and deep breathing 4. Monitoring the red blood cell count

1. Encouraging fluids

A 42-year-old female highway construction worker is concerned about her cancer risks. She has been married for 18 years, has two children, smokes one pack of cigarettes per day, and occasionally drinks one to two beers. She is 30 lbs. overweight, eats fast food often, and rarely eats fresh fruits and vegetables. Her mother was diagnosed with breast cancer 2 years ago. Her father and an aunt both died of lung cancer. She had a basal cell carcinoma removed from her cheek 3 years earlier. What behavioral changes should the nurse instruct this client to make to decrease her risk of cancer? Select all that apply. 1. Improve nutrition. 2. Decrease alcohol consumption. 3. Use sunscreen. 4. Stop smoking. 5. Loose weight. 6. Change her job to work inside

1. Improve nutrition. 3. Use sunscreen. 4. Stop smoking. 5. Loose weight.

You are providing end of life care for a patient with terminal liver cancer. The patient is weak and restless. Her skin is cool and mottled. Dyspnea develops and the patient appears anxious and frightened. What should you do? 1. Obtain an order for morphine elixir 2. Alert the rapid response team and call the physician for orders 3. Deliver breaths at 20/min with a bag-valve mask and prepare for intubation 4. Sit quietly with the patient and offer emotional support and comfort

1. Obtain an order for morphine elixir

A client with carcinoma of the lung develops syndrome of inappropriate antidiuretic hormone (SIADH) as a complication of the cancer. The nurse anticipaes that which of the following may be prescribed? SELECT ALL THAT APPLY. 1. Radiation 2. Chemotherapy 3. Increased fluid intake 4. Serum sodium levels 5. Decreased oral sodium intake 6. Medication that is antagonistic to ADH

1. Radiation 2. Chemotherapy 4. Serum sodium levels 6. Medication that is antagonistic to ADH

When a blood transfusion is terminated following a reaction, the nurse must do which of the following? Select all that apply. 1. Send freshly collected urine samples to the laboratory. 2. Return the remainder of the blood component unit to the blood bank. 3. Return the intravenous administration set to the blood bank. 4. Alert Risk Management about the incident. 5. Report the incident to the Infection control Manager.

1. Send freshly collected urine samples to the laboratory. 2. Return the remainder of the blood component unit to the blood bank. 3. Return the intravenous administration set to the blood bank. 4. Alert Risk Management about the incident.

Which of the following should be included in the teaching plan for a client with cancer who is experiencing thrombocytopenia? Select all that apply. 1. Use an electric razor 2. Use a soft-bristle toothbrush 3. Avoid frequent flossing for oral care 4. Include an over the counter no steroidal anti-inflammatory (NSAID) daily for pain control. 5.Monitor temperature daily. 6.Report bleeding, such as nosebleed, petechiae, or Melena, to a healthcare professional.

1. Use an electric razor 2. Use a soft-bristle toothbrush 3. Avoid frequent flossing for oral care 6.Report bleeding, such as nosebleed, petechiae, or Melena, to a healthcare professional.

A client is hospitalized with lung cancer. To manage severe pain, the physician prescribes a continuous I.V. infusion of morphine. Which formula should the nurse use to check that the morphine dose is appropriate for the client?

10 mg/70 kg of body weight

A female client who is receiving external beam radiation therapy for colon cancer asks the nurse if after the treatment she is "radioactive." The nurse should explain that: 1. "Only your urine and stool will be radioactive." 2. "You will not be radioactive at any time." 3. "You will be radioactive only during the treatment period." 4. "Although your blood is radioactive, it will not affect anyone else."

2. "You will not be radioactive at any time."

A patient with lung cancer develops syndrome of inappropriate antidiuretic hormone secretion (SIADH). After reporting symptoms of weight gain, weakness, and nausea and vomiting to the physician, you would anticipate which initial order for the treatment of this patient? 1. A fluid bolus 2. Fluid restrictions 3. Urinalysis 4. Sodium restricted diet

2. Fluid restrictions

A nurse is counseling the family of a client who has terminal cancer about palliative care. The nurse explains that which of the following are goals of palliative care? Select all that apply. 1. Delays death 2. Offers support system 3. Provides relief from pain 4. Enhances the quality of life 5. Focuses only on the client, not on the family 6. Manages symptoms of disease and therapy

2. Offers support system 3. Provides relief from pain 4. Enhances the quality of life 6. Manages symptoms of disease and therapy

The nurse is monitoring a child for bleeding after surgery for removal of a brain tumor. The nurse checks the head dressing for the presence of blood and notes a colorless drainage on the back of the dressing. Which intervention should the nurse perform immediately? 1. Reinforce the dressing 2. Notify the health care provider (HCP) 3. Document the findings and continue to monitor. 4. Circle the area of drainage and continue to monitor.

2. Notify the health care provider (HCP)

A client has undergone mastectomy. The nurse determines that the client is having the most difficulty adjusting to the loss of the breast if which behavior is observed? 1. Performs arm exercises 2. Refuses to look at the dressing 3. Reads the postoperative care booklet 4. Requests pain medication when needed

2. Refuses to look at the dressing

A client is receiving monthly doses of chemotherapy for treatment of stage III colon cancer. The nurse should report which of the following laboratory results to the oncologist before the next dose of chemotherapy is administered? Select all that apply. 1. Hemoglobin of 14.5 g/dL. 2. Platelet count of 40,000/mm3 3. Blood urea nitrogen (BUN) level of 12 mg/dL. 4. White blood cell count of 2,300/mm3. 5. Temperature of 101.2 degrees F(38.4 degrees C) 6. Urine specific gravity of 1.020.

2. Platelet count of 40,000/mm3 4. White blood cell count of 2,300/mm3. 5. Temperature of 101.2 degrees F(38.4 degrees C)

A month following biopsy, a 75-year-old female client returns to the surgeon's office for a report on her diagnostic procedure to determine the cell composition of her abdominal neoplasm. Which of the following terms is significant to indicate the likelihood of the tumor spreading? 1. Neoplasm 2. Lesion 3. Benign 4. Primary site

3

A client undergoes a muscle biopsy. After the procedure, the nurse must keep the biopsy site elevated for:

24 hours.

A client diagnosed with acute myelocytic leukemia has been receiving chemotherapy. During the last 2 cycles of chemotherapy, the client developed severe thrombocytopenia requiring multiple platelet transfusions. The client is now scheduled to receive a third cycle. How can the nurse best detect early signs and symptoms of thrombocytopenia? 1. Check the client's history for a congenital link to thrombocytopenia. 2. Perform a cardiovascular assessment every 4 hours. 3. Closely observe the client's skin for petechiae and bruising. 4. Monitor daily platelet counts.

3

A client with diabetes punctured hisfoot with a broken acorn in the yard. Within a week, the client developed osteomyelitis of the foot. The client was admitted for IV antibiotic therapy. How long does the nurse anticipate the client will receive IV antibiotics?

3 to 6 weeks

A client with diabetes punctured hisfoot with a broken acorn in the yard. Within a week, the client developed osteomyelitis of the foot. The client was admitted for IV antibiotic therapy. How long does the nurse anticipate the client will receive IV antibiotics? a) 3 months b) 3 to 6 weeks c) 6 months d) 7 to 10 days

3 to 6 weeks Explanation: Identification of the causative organism to initiate appropriate and ongoing antibiotic therapy for infection control. IV antibiotic therapy is administered for 3 to 6 weeks. Oral antibiotics then follow for as long as 3 months.

At what age is an individual most at risk for acquiring acute lymphocytic leukemia (ALL)?

4 to 12 years Explanation: The peak incidence of ALL is at 4 years of age. ALL is uncommon after 15 years of age. The median age at incidence of CML is 40 to 50 years. The peak incidence of AML occurs at 60 years of age. Two-thirds of cases of chronic lymphocytic leukemia occur in clients older than 60 years of ag

A client is experiencing intense nausea while being treated with chemotherapeutics. What actions should the nurse perform? Select all that apply.

> Ensure that the client has had a dietician consult > Administer antiemetics as prescribed > Provide the client with small, frequent meals

Cytotoxic antineoplastic medications can produce several adverse effects that can affect the treatment used. In caring for patients undergoing chemotherapy, for which of the following should monitoring occur? (Select all that apply.)

> Mucositis > Tetany

Which of the following indicates hypovolemic shock in a client who had had a 15% blood loss? 1. Pulse rate less than 60bpm. 2.Respiratory rate of 4 breaths/min 3. Pupils unequally dilated 4. Systolic blood pressure less than 90 mm Hg.

4. Systolic blood pressure less than 90 mm Hg.

A client with cancer is scheduled for radiation therapy. The nurse knows that radiation at any treatment site may cause a certain adverse effect. Therefore, the nurse should prepare the client to expec

Fatigue

A client with suspected lung cancer is scheduled for thoracentesis as part of the diagnostic workup. The nurse reviews the client's history for conditions that might contraindicate this procedure. Which condition is a contraindication for thoracentesis?

A bleeding disorder

A client with suspected osteomalacia has a fractured tibia and fibula. What test would give a definitive diagnosis of osteomalacia? a) A bone biopsy b) Demineralization of the bone c) Increased and decreased areas of bone metabolism d) Elevated levels of alkaline phosphatase

A bone biopsy Explanation: A definitive diagnosis is obtained by bone biopsy. Radiographic studies demonstrate demineralization of the bone. A bone scan detects increased and decreased areas of bone metabolism. Alkaline phosphatase levels are detected from a blood sample.

The nurse is caring for a client on a second course of antibiotics to eliminate osteomyelitis. It is most essential for the nurse to instruct on which aspect of daily care?

A diet high in protein and nutrients Explanation: It is essential for the nurse to instruct on a diet that is high in protein and nutrients to increase healing and strengthen the immune system. This, in addition to the second course of antibiotics, may be sufficient to eliminate the osteomyelitis. Opioids may be needed for pain management but this is not most essential. Bedrest is not common in care and assistive devices are used only in the acute period.

During a breast examination, which finding most strongly suggests that the client has breast cancer?

A fixed nodular mass with dimpling of the overlying skin

During a breast examination, which finding most strongly suggests that a client has breast cancer?

A fixed nodular mass with dimpling of the overlying skin A fixed nodular mass with dimpling of the overlying skin is common during late stages of breast cancer. Many women have slightly asymmetrical breasts. Bloody nipple discharge is a sign of intraductal papilloma, a benign condition. Multiple firm, round, freely movable masses that change with the menstrual cycle indicate fibrocystic breasts, a benign condition. (less)

A 56-year-old client has lost 34 pounds since the start of chemotherapy for colon cancer. Which of the following laboratory findings should be reported to the health care provider? A. Albumin of 2.4 g/dL B. White blood cell (WBC) count 3800/mm3 C. Urine specific gravity 1.020 D. Platelet count 90,000/mm3

A. Albumin of 2.4 g/dL

A nurse providing teaching for skin protection to a patient undergoing radiation treatment includes? A. Using an umbrella when in direct sun exposure B. Use of a coarse bristled brush C. Leaving affected area uncovered when outdoors during the daytime D. Rubbing the skin dry with a soft towel E. Use of a belt or binding device to secure pants close to the skin

A. Using an umbrella when in direct sun exposure

Which of the following would the nurse identify as a neurotransmitter?

Acetylcholine

Ankylosing spondylitis

Ankylosing spondylitis is associated with a flattening of the lumbar curvature.

Which medication classification is prescribed when allergy is a factor causing the skin disorder?

Antihistamines

A client receiving external radiation to the left thorax to treat lung cancer has a nursing diagnosis of Risk for impaired skin integrity. Which intervention should be part of this client's plan of care?

Avoiding using soap on the irradiated areas

When assessing a client for early septic shock, the nurse should assess the client for which of the following? A. Cool, clammy skin B. Warm, flushed skin C. Increased blood pressure D. Hemorrhage

B. Warm, flushed skin

During the admission assessment of a client with advanced ovarian cancer, the nurse recognizes which symptom as typical of the disease? A. Hypermenorrhea B. Abdominal distention C. Diarrhea D. Abnormal bleeding

B. Abdominal distention

A client is undergoing tests for multiple myeloma. Diagnostic study findings in multiple myeloma include:

Bence Jones protein in the urine.

The nurse monitors a client receiving enoxaparin (Lovenox), 30 mg subQ b.i.d after hip replacement surgery. Which adverse reaction is the client most likely to experience?

Bleeding

Which of the following does a nurse thoroughly evaluate before a bone marrow transplant (BMT) procedure?

Blood studies

What food can the nurse suggest to the client at risk for osteoporosis?

Broccoli

A Client with bone metastasis has a calcium level of 20.8mg/dL. The nurse should be alert for which symptom correlating with the level? A) Hyperactivity B) Oliguria C) ECG changes D) Polyphagia

C) ECG changes

The nurse is assessing a client who has a new ureterostomy. Which statement by the client indicates the need for more education about the care of a urinary stoma? A. "I change my pouch every week." B. "I change the appliance in the evening" C. I empty the urinary collection bag when it is two-thirds full D. "When I'm in the shower, I direct the flow of water away from the stoma"

C. I empty the urinary collection bag when it is two-thirds full

When caring for a client experiencing an acute gout attack, the nurse anticipates administering which medication?

Colchicine

Which assessment findings would the nurse expect to find in the postoperative client experiencing fat embolism syndrome?

Column B

An x-ray demonstrates a fracture in which a bone has splintered into several pieces. Which type of fracture is this?

Comminuted

A client with supraglottic cancer undergoes a partial laryngectomy. Postoperatively, a cuffed tracheostomy tube is in place. When removing secretions that pool above the cuff, the nurse should instruct the client to

Cough as the cuff is being deflated

A client presents to the emergency department with an open fracture. What is the first action the nurse should take?

Cover the exposed bone with sterile dressing

Hallux valgus

Hallux valgus is noted as the great toe deviating laterally and possibly overlapping the second toe.

While administering cisplatin (Platinol-AQ) to a client, the nurse assesses swelling at the insertion site. The first action of the nurse is to

Discontinue the intravenous medication.

The nurse is providing care for a client with possible pancreatic cancer. Which test is the most definitive diagnostic exam for this type of cancer? A) Serum Amylase B) Pancreatic ultrasonography C) CT Scan D) ERCP with cytologies

D) ERCP with cytologies

During a routine physical examination of a client, the nurse observes a flexion deformity of the promixal interphalangeal (PIP) joint of two toes on the right foot. The nurse documents this finding as which of the following?

Hammer toe

During the admission assessment of a client with advanced ovarian cancer, the nurse recognizes which symptom as typical of the disease? A. Diarrhea B. Hypermenorrhea C. Abnormal bleeding D. Abdominal distention

D. Abdominal distention

The nurse manager is teaching the nursing staff about signs and symptoms related to hypercalcemia in a client with metastatic prostate cancer and tells the staff that which is a late sign of this oncological emergency? A. Headache B. Dysphagia C. Constipation D. Electrocardiographic changes

D. Electrocardiographic changes

Which of the following clinical manifestations would the nurse expect to find in a client who has had osteoporosis for several years? a) Diarrhea b) Decreased height c) Bone spurs d) Increased heel pain

Decreased height Explanation: Clients with osteoporosis become shorter over time.

A client comes to the emergency department complaining of pain in the right leg. When obtaining his history, the nurse learns that the client was diagnosed with diabetes mellitus at age 12. The nurse knows that this disease predisposes the client to which musculoskeletal disorder?

Degenerative joint disease

Which is a flexion deformity caused by a slowly progressive contracture of the palmar fascia? a) Callus b) Hammertoe c) Hallux valgus d) Dupuytren's contracture

Dupuytren's contracture Explanation: Dupuytren's disease results in a slowly progressive contracture of the palmar fascia, called Dupuytren's contracture. A callus is a discretely thickened area of skin that has been exposed to persistent pressure or friction. A hammertoe is a flexion deformity of the interphalangeal joint, which may involve several toes. Hallux valgus is a deformity in which the great toe deviates laterally

The nurse is planning an education program for women of childbearing years. The nurse recognizes that primary prevention of osteoporosis includes: a) Ensuring adequate calcium and vitamin D intake b) Engaging in non-weight-bearing exercises daily c) Having a DXA beginning at age 35 years d) Undergoing assessment of serum calcium levels every year

Ensuring adequate calcium and vitamin D intake Explanation: Nutritional intake of calcium and vitamin D are essential for the prevention of osteoporosis.

Which of the following terms refers to a fracture in which one side of a bone is broken and the other side is bent?

Greenstick

A nurse is assessing a client with metastatic lung cancer. The nurse should assess the client specifically for:

Hoarseness - may indicate metastatic disease to the recurrent laryngeal nerve and is commonly noted with left upper lobe lung tumors. Diarrhea and constipation are not associated with lung cancer. Weight loss, not weight gain, can be a symptom of extensive disease.

When evaluating for hypovolemic shock, the nurse should be aware of which of the following clinical manifestations?

Hypotension

Which of the following is a factor that inhibits fracture healing?

Local malignancy

Which of the following is a growth-based classification of tumors?

Malignant

Protraction

Protraction is moving forward

A client in the terminal stage of cancer is receiving a continuous infusion of morphine (Duramorph) for pain management. Which data collection finding suggests that the client is experiencing an adverse effect of this drug?

Respiratory rate of 8 breaths/minute

Retraction

Retraction is moving backward

Which intervention will the nurse suggest to a client with a leg amputation to help cope with loss of the limb?

Talking with an amputee close to the client's age who has had the same type of amputation Correct: Meeting with someone of a comparable age who has gone through a similar experience will help the client cope better with his or her own situation.

A client who sustained a right lower leg fracture in a motorcycle accident has an external fixation device in place. During visiting hours, a nurse sees a friend of the client using the device to lift the client's leg off of the bed. Which action should the nurse take?

Teach the client and his friend the proper technique for lifting the leg.

A nurse is administering daunorubicin to a patient with lung cancer. Which situation requires immediate intervention?

The I.V. site is red and swollen.

The nurse is teaching the client how to use a cane. Which statements is inaccurate?

The client should hold the cane on the involved side.

The spouse of a client with gastric cancer expresses concern that the couple's children may develop this type of cancer when they're older. When reviewing risk factors for gastric cancer with the client and family, the nurse explains that a certain blood type increases the risk by 10%. The nurse is referring to:

Type A.

Before a cancer client receiving total parenteral nutrition (TPN) resumes a normal diet, the nurse teaches him about dietary sources of minerals. Which foods are good sources of zinc?

Whole grains and meats

Which of the following would the nurse expect to assess as the most common finding associated with fibromyalgia?

Widespread chronic pain

A client is admitted to the orthopedic unit for treatment of a fractured right femur caused by a motor vehicle crash. He is scheduled to undergo an open reduction internal-fixation of his right femur. The night before surgery, the nurse administers 250 mg of glutethimide (Doriden) as prescribed. Which statement regarding usage of glutethimide is correct?

The nurse should store the drug in a tight, light-resistant container.

A patient with a recent diagnosis of chronic myelogenous leukemia (CML) is discussing treatment options with his care team. What aspect of the patient's condition would contraindicate the use of cyclophosphamide for the treatment of leukemia?

The patient's bone marrow function is significantly depressed.

A nurse is teaching a female client about preventing osteoporosis. Which teaching point is correct?

The recommended daily allowance of calcium may be found in a wide variety of foods.

The nurse is teaching a female client about preventing osteoporosis. Which of the following teaching points is correct?

The recommended daily allowance of calcium may be found in a wide variety of foods.

"A 43-year-old black client without a family history of prostate cancer asks the nurse when he should have a prostate-specific antigen (PSA) test and a digital rectal examination (DRE) performed. Which response by the nurse is appropriate?

You should start undergoing prostate cancer screening at age 45.

A 32-year-old woman recently diagnosed with Hodgkin's disease is admitted for staging by undergoing a bone marrow aspiration and biopsy. To obtain more information about the client's nutrition status, the nurse should review the results of which test?

albumin level

A patient who is receiving chemotherapy is admitted to the hospital with acute dehydration caused by nausea and vomiting. Which action will the nurse include in the plan of care to best prevent the development of shock, systemic inflammatory response syndrome (SIRS), and multiorgan dysfunction syndrome (MODS)? a. Administer all medications through the patient's indwelling central line. b. Place the patient in a private room. c. Restrict the patient to foods that have been well-cooked or processed. d. Insert a nasogastric (NG) tube for enteral feeding.

b. Place the patient in a private room.

In teaching about cancer prevention to a community group, the nurse stresses promotion of exercise, normal body weight, and low-fat diet because a. most people are willing to make these changes to avoid cancer. b. dietary fat and obesity promote growth of many types of cancer. c. people who exercise and eat healthy will make other lifestyle changes. d. obesity and lack of exercise cause cancer in susceptible people.

b. dietary fat and obesity promote growth of many types of cancer.

A patient with hypovolemic shock has a urinary output of 15 ml/hr. The nurse understands that the compensatory physiologic mechanism that leads to altered urinary output is... a. activation of the sympathetic nervous system (SNS), causing vasodilation of the renal arteries. b. stimulation of cardiac -adrenergic receptors, leading to increased cardiac output. c. release of aldosterone and antidiuretic hormone (ADH), which cause sodium and water retention. d. movement of interstitial fluid to the intravascular space, increasing renal blood flow.

c. release of aldosterone and antidiuretic hormone (ADH), which cause sodium and water retention.

Rheumatoid arthritis

can occur in any joint but usually affects the hands first

A client is taking methotrexate for severe rheumatoid arthritis. The nurse instructs the client that it will be necessary to monitor:

complete blood count (CBC) with differential and platelet count. Explanation: This client should be monitored for blood dyscrasias, evidenced by decreased platelet count and white blood cell count with changes in the CBC differential.

A 61 year old woman who is 5 feet 3 inches tall and weighs 125 pounds tells the nurse that she has a glass of wine two or three times a week. The patient works for the post office and has a 5 mile mail delivery route. This is her first contact with the health care system in 20 years. Which of these topics will the nurse plan to include in patient teaching about cancer? (select all that apply) a. alcohol use b. physical activity c. body weight d. colorectal screening e. tobacco use f. mammography g. Pap testing h. sunscreen use Answer: D, F, G, H

d. colorectal screening f. mammography g. Pap testing h. sunscreen use

A female patient is taking oral cyclophosphamide therapy for breast cancer. Because of possible adverse effects of the drug, the nurse will instruct the patient to:

drink a lot of water.

A 63-year-old male patient has just begun treatment with IV paclitaxel. About 10 minutes into the infusion, the nurse becomes concerned about a possible anaphylactic reaction to the drug because the patient is experiencing:

dyspnea

The nurse is evaluating the client's risk for cancer and recommends changes when the client states she

eats red meat such as steaks or hamburgers every day

The physician diagnoses primary osteoporosis in a client who has lost bone mass. In this metabolic disorder, the rate of bone resorption accelerates while bone formation slows. Primary osteoporosis is most common in:

elderly postmenopausal women.

After a lobectomy for lung cancer, the nurse instructs the client to perform deep-breathing exercises to:

expand the alveoli and increase lung surface available for ventilation.

A 3-year-old child receiving chemotherapy after surgery for a Wilms' tumor has developed neutropenia. The parent is trying to encourage the child to eat by bringing extra foods to the room. Which food would not be appropriate for this child?

fresh strawberries When a client receiving chemotherapy develops neutropenia, eating uncooked fruits and vegetables may pose a health risk due to possible bacterial contamination. All other foods are either cooked or pasteurized and would not produce a health risk.

A client with cancer is uncertain about how to cope with all the issues that will arise. The nurse can best support the coping behaviors of a client with cancer by:

helping the client identify available resources.

A client with suspected cervical cancer is undergoing a colposcopy with conization. The nurse gives instructions to the client about her menstrual periods, emphasizing that:

her next two or three periods may be heavier and more prolonged than usual.

The nurse is teaching a client with a T4 spinal cord injury and paralysis of the lower extremities how to transfer from the bed to a wheelchair. The nurse should instruct the client to move:

his upper body to the wheelchair first.

During a scoliosis screening in a college heath center, a student asks the public health nurse about the consequences of untreated scoliosis. The nurse would be accurate by identifying one of the direct complications as:

impingement on pulmonary function.

A nurse suspects that a client with a recent fracture has compartment syndrome. Assessment findings may include

inability to perform active movement and pain with passive movement. Explanation: With compartment syndrome, the client can't perform active movement, and pain occurs with passive movement. A body-wide decrease in bone mass is seen in osteoporosis. A growth in and around the bone tissue may indicate a bone tumor

Lordosis

increased lumbar curvature, compensates for the enlarging uterus.

The nurse is assessing a middle-aged client with cancer who has lost 1 lb (0.5 kg) in 4 weeks. The client is taking ondansetron for nausea and now has a temperature of 101° F (38.3° C). The fever is indicative of:

infection

A client with metastatic brain cancer is admitted to the oncology floor. According to the Self-Determination Act of 1991 concerning the execution of an advance directive, the hospital is required to:

inform the client or legal guardian of their rights to execute an advance directive.

The bulge test

is used to determine if knee swelling is due to accumulation of fluid or soft tissue swelling. It does not address range or motion. Knee swelling is never considered to be an age-related change

Lifestyle risk factors for osteoporosis include

lack of exposure to sunshine.

A client receives a sealed radiation implant to treat cervical cancer. When caring for this client, the nurse should:

maintain the client on complete bed rest with bathroom privileges only.

After a traumatic spinal cord injury, a client requires skeletal traction. When caring for this client, the nurse must:

maintain traction continuously to ensure its effectiveness.

Abduction

moving away from the midline of the body

A fracture is termed pathologic when the fracture:

occurs through an area of diseased bone

A client with gouty arthritis is prescribed a low-purine diet. The nurse should instruct this client to avoid:

organ meats.

A client with possible osteoarthritis is having X-rays performed on both knees. X-rays of an osteoarthritic joint reveal:

osteophyte formation.

A client is undergoing a diagnostic workup for suspected thyroid cancer. What is the most common form of thyroid cancer in adults?

papillary carcinoma Papillary carcinoma accounts for about 70% of thyroid cancer cases in adults. Follicular carcinoma accounts for roughly 15%; anaplastic carcinoma, about 5%; and medullary carcinoma, about 5%.

Elderly clients who fall are most at risk for:

pelvic fracture

A client asks the nurse what cancer cell growth is called. What would the nurse tell the client?

proliferation

The nurse is reviewing the medication administration record of the client. Which of the following medications would lead the nurse to suspect that the client is at risk for osteoporosis?

raloxifene (Evista)

The nurse is reviewing the medication administration record of the client. Which of the following medications would lead the nurse to suspect that the client is at risk for osteoporosis? a) methotrexate (Rheumatrex) b) penicillamine (Cuprimine) c) plicamycin (Mithracin) d) raloxifene (Evista)

raloxifene (Evista) Explanation: Raloxifene (Evista) is used for the prevention and treatment of osteoporosis.

Palliation refers to

relief of symptoms associated with cancer.

A client is hospitalized for open reduction of a fractured femur. After surgery, the nurse monitors for signs and symptoms of fat embolism, which include:

restlessness and petechiae.

With fractures of the femoral neck, the leg is:

shortened, adducted, and externally rotated

While assessing the range of motion in an adult client's shoulders, the client expresses pain and exhibits limited abduction and muscle weakness. The nurse plans to refer the client to a physician for possible

rotator cuff tear

A client has a herniated disk in the region of the third and fourth lumbar vertebrae. When collecting data, the nurse expects to note:

severe low back pain.

A right orchiectomy is performed on a client with a testicular malignancy. The client expresses concerns regarding his sexuality. The nurse should base the response on the knowledge that the client:

should retain normal sexual drive and function. Explanation: Unilateral orchiectomy alone does not result in impotence if the other testis is normal. The other testis should produce enough testosterone to maintain normal sexual drive, functioning, and characteristics. Sperm banking before treatment is commonly recommended because radiation or chemotherapy can affect fertility.

sternoclavicular joint

sternoclavicular is at the junction of the manubrium and clavicle.

After a laminectomy, the client states, "The doctor said that I can do anything I want to." Which activity that the client intends to do indicates the need for further teaching?

sweeping the front porch Explanation: Sweeping causes a twisting motion, which should be avoided because twisting can cause undue stress on the recently ruptured disc site, muscle spasms, and a potential recurrent disc rupture. Although the client should not bend at the waist, such as when washing dishes at the sink, the client can dry dishes because no bending is necessary. The client can sit in a firm chair that keeps the back anatomically aligned. The client should not twist and pull, so when making the bed, the client should pull the covers up on one side and then walk around to the other side before trying to pull the covers up there

Adduction

the movement towards the midline of the body.

Cancer prevalence is defined as:

the number of persons with cancer at a given point in time. Explanation: The word prevalence in a statistical setting is defined as the number of cases of a disease present in a specified population at a given time.

The nurse is preparing a community presentation on oral cancer. Which is a primary risk factor for oral cancer that the nurse should include in the presentation?

use of alcohol

The nurse suspects carpal tunnel syndrome after examining a patient in the clinic. A test result that would suggest this diagnosis would be

weak opposition of the thumb

temporomandibular joint

where the mandible and temporal joint articulate

The nurse is developing a care plan for a client who has leukemia. What instructions should the nurse include in the plan? Select all that apply

• Monitor temperature and report elevation. • Recognize signs and symptoms of infection. • Avoid crowds. • Maintain integrity of skin and mucous membranes.

Which statement indicates to the nursing instructor that the nursing student understands the normal healing process of bone after a fracture?

"A hematoma forms at the site of the fracture." Correct: In stage 1, within 24 to 72 hours after a fracture, a hematoma forms at the site of the fracture because bone is extremely vascular. This then prompts the formation of fibrocartilage, providing the foundation for bone healing.

During a routine physical examination on a 75-year-old female client, a nurse notes that the client is 5 feet, 3/8 inches (1.6 m) tall. The client states, "How is that possible? I was always 5 feet and 1/2? (1.7 m) tall." Which statement is the best response by the nurse?

"After menopause, the body's bone density declines, resulting in a gradual loss of height."

During a routine physical examination on a 75-year-old female client, a nurse notes that the client is 5 feet, 3/8 inches (1.6 m) tall. The client states, "How is that possible? I was always 5 feet and 1/2? (1.7 m) tall." Which statement is the best response by the nurse? a) "After menopause, the body's bone density declines, resulting in a gradual loss of height." b) "After age 40, height may show a gradual decrease as a result of spinal compression" c) "The posture begins to stoop after middle age." d) "There may be some slight discrepancy between the measuring tools used."

"After menopause, the body's bone density declines, resulting in a gradual loss of height." Explanation: The nurse should tell the client that after menopause, the loss of estrogen leads to a loss in bone density, resulting in a loss of height. This client's history doesn't indicate spinal compression. Telling the client that measuring tools used to obtain the client's height may have a discrepancy or that the posture begins to stoop after middle age doesn't address the client's question.

A client is treated in the emergency department for acute muscle strain in the left leg caused by trying a new exercise. During discharge preparation, the nurse should provide which instruction?

"Apply ice packs for the first 24 to 48 hours, and then apply heat packs."

A young man with early-stage testicular cancer is scheduled for a unilateral orchiectomy. The client confides to the nurse that he is concerned about what effects the surgery will have on his sexual performance. Which response by the nurse provides accurate information about sexual performance after an orchiectomy?

"Because your surgery does not involve other organs or tissues, you will likely not notice much change in your sexual performance." Explanation: Although there may not be a big change in sexual function with a unilateral orchiectomy, the loss of a gonad and testosterone may result in decreased libido and sterility. Sperm banking may be an option worth exploring if the number and motility of the sperm are adequate. Remember, the population most affected by testicular cancer is generally young men aged 15 to 34, and in this crucial stage of life, sexual anxieties may be a large concern

The client with a fracture asks the nurse about the difference between a compound fracture and a simple fracture. Which statement by the nurse is correct?

"Compound fracture, grade I, involves minimal skin damage." Correct: A grade I compound fracture involves minimal damage to the skin.

For a client with osteoporosis, the nurse should provide which dietary instruction?

"Eat more dairy products to increase your calcium intake."

A nurse is teaching a client who was recently diagnosed with carpal tunnel syndrome. Which statement should the nurse include?

"Ergonomic changes can be incorporated into your workday to reduce stress on your wrist."

A nurse is teaching a client who was recently diagnosed with carpal tunnel syndrome. Which statement should the nurse include? a) "Surgery is the only sure way to manage this condition." b) "Using arm splints will prevent hyperflexion of the wrist." c) "This condition is associated with various sports." d) "Ergonomic changes can be incorporated into your workday to reduce stress on your wrist."

"Ergonomic changes can be incorporated into your workday to reduce stress on your wrist." Explanation: Ergonomic changes, such as adjusting keyboard height, can help clients with carpal tunnel syndrome avoid hyperextension of the wrist. This condition is associated with repetitive tasks such as clerical work, not sports. The condition may be managed with medications, yoga, acupuncture, and wrist (not arm) splints.

A nurse is assessing a female who is receiving her second administration of chemotherapy for breast cancer. When obtaining this client's health history, what is the most important information the nurse should obtain?

"Have you had nausea or vomiting?" Chemotherapy agents typically cause nausea and vomiting when not controlled by antiemetic drugs. Antineoplastic drugs attack rapidly growing normal cells, such as in the gastrointestinal tract. These drugs also stimulate the vomiting center in the brain. Hair loss, loss of energy, and sleep are important aspects of the health history, but not as critical as the potential for dehydration and electrolyte imbalance caused by nausea and vomiting.

Which client statement identifies a knowledge deficit about cast care?

"I can pull out cast padding to scratch inside the cast." Explanation: Clients should not pull out cast padding to scratch inside the cast because of the hazard of skin breakdown and subsequent potential for infection. Clients are encouraged to elevate the casted extremity above the level of the heart to reduce edema and to exercise or move the joints above and below the cast to promote and maintain flexibility and muscle strength. Applying ice for 10 minutes during the first 24 hours helps to reduce edema.

A client has a total hip replacement. Which of the following client statements indicates a need for further teaching before discharge?

"I can't wait to take a tub bath when I get home." Explanation: The client will need to avoid extremes of motion in the hip to avoid dislocation. The hip should not be flexed more than 90 degrees, internally rotated, or legs crossed. It is not possible to safely sit in the bathtub without flexing the hip beyond the recommended 90 degrees. The client can implement the prescribed exercise program at the time of discharge home. The client should take care not to stress the hip for 3 to 6 months after surgery. An elevated toilet seat will be necessary during the recovery from surgery.

A client undergoes a total hip replacement. Which statement made by the client indicates to the nurse that the client requires further teaching?

"I don't know if I'll be able to get off that low toilet seat at home by myself."

A client undergoes a total hip replacement. Which statement made by the client would indicate to the nurse that the client requires further teaching?

"I don't know if I'll be able to get off that low toilet seat at home by myself."

Which of the following statements indicates that the client needs further teaching about taking medication to control cancer pain?

"I should skip doses periodically so I don't get hooked on my drugs." The client should not skip his dosages of pain medication to prevent addiction. Clients with cancer pain do not become psychologically dependent on the medication and should not fear becoming addicted. The nurse should allow the client and family members to verbalize their concerns about drug addiction.

Which statement by the client with rheumatoid arthritis would indicate the need for additional teaching to safely receive the maximum benefit of aspirin therapy?

"I try to take aspirin only on days when the pain seems particularly bad." Explanation: Aspirin therapy in rheumatoid arthritis involves continuous ongoing administration to establish and maintain therapeutic blood levels. Aspirin should not be used on an as-needed basis. Aspirin should always be buffered with food. Generic aspirin is acceptable. Clients should be instructed to observe for symptoms of bleeding.

On the night before a 58-year-old wife and mother is to have a lobectomy for lung cancer, she remarks to the nurse, "I am so scared of this cancer. I should have quit smoking years ago. Now I have brought all this fear and sadness on myself and now my family." The nurse should tell the client:

"It is okay to be scared. What is it about cancer that you are afraid of?" Explanation: Acknowledging the basic feeling that the client expressed and asking an open-ended question allows the client to explain her fears. Saying, "It is normal to be scared. We will help you through it," does not focus on the client's feelings; rather, it gives reassurance. Asking if the client feels guilty for having smoked assumes guilt, which might be present, but additional information is needed to confirm. Telling the client not to be so hard on herself does not acknowledge the client's feelings at all.

The nurse is instructing a client on how to care for skin that has become dry after radiation therapy. Which statement by the client indicates that the client understands the teaching?

"It is safe to apply a nonperfumed lotion to my skin." Explanation: Irradiated skin can become dry and irritated, resulting in itching and discomfort. The client should be instructed to clean the skin gently and apply nonperfumed, nonirritating lotions to help relieve dryness. Taking an antihistamine does not relieve the skin dryness that is causing the itching. Heat should not be applied to the area because it can cause further irritation. Medicated ointments should not be applied to the skin without the prescription of the radiation therapist.

The client is recovering from an above-knee amputation resulting from peripheral vascular disease. Which statement indicates that the client is coping well after the procedure?

"It will take me some time to get used to this." Correct: This statement indicates that the client is expressing acceptance and effective coping.

A client has sustained a right tibial fracture and has just had a cast applied. Which instruction should the nurse provide in his cast care?

"Keep your right leg elevated above heart level."

A female client is receiving L-asparaginase. She is at the end of her chemotherapeutic course and is concerned because her liver function tests are abnormal. What is an appropriate response by the nurse?

"Liver impairment usually subsides when chemotherapy is complete."

A client asks the nurse what the difference is between osteoarthritis (OA) and rheumatoid arthritis (RA). Which response is correct?

"OA is a noninflammatory joint disease. RA is characterized by inflamed, swollen joints." Explanation: OA is a degenerative arthritis, characterized by the loss of cartilage on the articular surfaces of weight-bearing joints with spur development. RA is characterized by inflammation of synovial membranes and surrounding structures. OA may occur in one hip or knee and not the other, whereas RA commonly affects the same joints bilaterally. RA is more common in women; OA affects both sexes equally

A client with osteoarthritis tells the nurse she is concerned that the disease will prevent her from doing her chores. Which suggestion should the nurse offer?

"Pace yourself and rest frequently, especially after activities."

The X-rays of a client who was brought to the emergency department after falling on ice reveal a leg fracture. After a cast is applied and allowed to dry, the nurse teaches the client how to use crutches. Which instruction should the nurse provide about climbing stairs?

"Place the unaffected leg on the first step, followed by the crutches and the injured leg, which should move together."

On an oncology unit, the nurse hears noises coming from a client's room. The client is found throwing objects at the walls and has just picked up the phone and is screaming, "How can God do this to me? It is the third type of cancer I have had. I have gone through all the treatment for nothing." In what order of priority from first to last should the nurse make the interventions? All options must be used.

"Please put the telephone down so we can talk." "I can hear how upset you are about the cancer." "Tell me what you are feeling right now." "I wonder if you would like to talk to a clergyman."

On a visit to the family physician, a client is diagnosed with a bunion on the lateral side of the great toe, at the metatarsophalangeal joint. Which statement should the nurse include in the teaching session?

"Some bunions are congenital; others are caused by wearing shoes that are too short or narrow."

On a visit to the family physician, a client is diagnosed with a bunion on the lateral side of the great toe, at the metatarsophalangeal joint. Which statement should the nurse include in the teaching session?

"Some bunions are congenital; others are caused by wearing shoes that are too short or narrow." Explanation: Bunions may be congenital or may be acquired by wearing shoes that are too short or narrow, which increases pressure on the bursa at the metatarsophalangeal joint. Acquired bunions can be prevented. Wearing shoes that are too big may cause other types of foot trauma but not bunions. Gout doesn't cause bunions. Although a client with gout may have pain in the big toe, such pain doesn't result from a bunion.

On a visit to the family physician, a client is diagnosed with a bunion on the lateral side of the great toe, at the metatarsophalangeal joint. Which statement should the nurse include in the teaching session? a) "Bunions are congenital and can't be prevented." b) "Bunions may result from wearing shoes that are too big, causing friction when the shoes slip back and forth." c) "Bunions are caused by a metabolic condition called gout." d) "Some bunions are congenital; others are caused by wearing shoes that are too short or narrow."

"Some bunions are congenital; others are caused by wearing shoes that are too short or narrow." Explanation: Bunions may be congenital or may be acquired by wearing shoes that are too short or narrow, which increases pressure on the bursa at the metatarsophalangeal joint. Acquired bunions can be prevented. Wearing shoes that are too big may cause other types of foot trauma but not bunions. Gout doesn't cause bunions. Although a client with gout may have pain in the big toe, such pain doesn't result from a bunion.

A nurse is performing an admission assessment on a client admitted with a pelvic fracture. Which statement by the client requires the nurse to seek more information from a legal standpoint?

"Sometimes my husband gets so angry with me." Explanation: Legally, the nurse must further investigate the client's statement concerning her husband's anger. This statement suggests that the client's injury might be caused by domestic abuse. The other statements are common and don't require further investigation, from a legal standpoint, by the nurse.

The nurse is caring for a client who complains of lower back pain. Which instructions should the nurse give the client to prevent back injury?

"Stand close to the object you're lifting."

Which statement by a staff nurse on the orthopedic floor indicates the need for further staff education?

"The client is receiving physical therapy twice per day, so he doesn't need a continuous passive motion device." Explanation: Further staff education is needed when the nurse states that the continuous passive motion device isn't needed because the client receives physical therapy twice per day. The continuous passive motion device should be used in conjunction with physical therapy because the device helps prevent adhesions. Bleeding is a complication associated with the continuous passive motion device; skin integrity should be monitored while the device is in use

A client suspected of having systemic lupus erythematosus (SLE) is being scheduled for testing. She asks which of the tests ordered will determine if she is positive for the disorder. Which statement by the nurse is most accurate?

"The diagnosis won't be based on the findings of a single test but by combining all data found." Explanation: There is no single test available to diagnose SLE. Therefore, the nurse should inform the client that diagnosis is based on combining the findings from the physical assessment and the laboratory tests results. Advising the client to speak with the physician, stating that SLE is a serious systemic disorder, and asking the client to express her feelings about the potential diagnosis don't answer the client's question.

A 76-year-old client tells the nurse that she has lived long and does not need mammograms. Which is the nurse's best response?

"The incidence of breast cancer increases with age." Explanation: Advancing age in postmenopausal women has been identified as a risk factor for breast cancer. A 76-year-old client needs monthly breast self-examination and a yearly clinical breast examination and mammogram to comply with the screening schedule. While mammograms are less painful as breast tissue becomes softer, the nurse should advise the woman to have the mammogram. Family history is important, but only about 5% of breast cancers are genetic.

A client with gout has been advised to lose weight. She informs the nurse that she plans to go on a "guaranteed rapid weight loss" plan that involves fasting and heavy exercise. Which response would be most appropriate?

"There might be some difficulties with your plan and fasting."

While doing a health history, a client tells you that her mother, grandmother, and sister died of cancer. The client wants to know what she can do to keep from getting cancer. What would be your best response?

"You can't prevent cancer, but you can have your blood analyzed for tumor markers to see what your risk level is."

A client who has just been diagnosed with mixed muscular dystrophy asks the nurse about the usual course of this disease. How should the nurse respond?

"You may experience progressive deterioration in all voluntary muscles."

A nurse is educating a patient diagnosed with osteomalacia. Which of the following statements by the nurse is appropriate? a) "You will need to avoid foods high in phosphorus, and vitamin D." b) "You will need to engage in vigorous exercise three times a week for 30 minutes." c) "You will need to decrease the amount of dairy products consumed." d) "You may need to be evaluated for an underlying cause, such as renal failure."

"You may need to be evaluated for an underlying cause, such as renal failure." Explanation: The patient may need to be evaluated for an underlying cause. If an underlying cause is discovered, that will guide the medical treatment. The patient needs to maintain an adequate to increased supply of calcium, phosphorus, and vitamin D. Dairy products are a good source of calcium. The patient is at risk for pathological fractures and therefore should not engage in vigorous exercise.

A nurse is educating a patient diagnosed with osteomalacia. Which of the following statements by the nurse is appropriate? a) "You will need to engage in vigorous exercise three times a week for 30 minutes." b) "You will need to decrease the amount of dairy products consumed." c) "You may need to be evaluated for an underlying cause, such as renal failure." d) "You will need to avoid foods high in phosphorus, and vitamin D."

"You may need to be evaluated for an underlying cause, such as renal failure." Explanation: The patient may need to be evaluated for an underlying cause. If an underlying cause is discovered, that will guide the medical treatment. The patient needs to maintain an adequate to increased supply of calcium, phosphorus, and vitamin D. Dairy products are a good source of calcium. The patient is at risk for pathological fractures and therefore should not engage in vigorous exercise.

An elderly client who is having difficulty using a walker tells the nurse, "I don't care what the doctor says; I'm not going to use that thing!" Which response by the nurse is best?

"You say that you don't like using the walker; is there any particular reason why?"

A middle-age client recovering from major back surgery must wear a back brace and walk with a cane after experiencing leg weakness. During routine care, the client tells a nurse, "I'm sorry I had this operation. Before surgery I didn't look like I had a problem, but now I do." Which response by the nurse is appropriate?

"You sound concerned about your appearance. In what way are you worse off than before?"

The nurse is planning discharge instructions for the client with osteomyelitis. What instructions should the nurse include in the discharge teaching?

"You will receive IV antibiotics for 3 to 6 weeks."

The nurse is planning discharge instructions for the client with osteomyelitis. What instructions should the nurse include in the discharge teaching? a) "You will receive IV antibiotics for 3 to 6 weeks." b) "You need to limit the amount of protein and calcium in your diet." c) "You need to perform weight-bearing exercises twice a week." d) "Use your continuous passive motion machine (CPM) 2 hours each day."

"You will receive IV antibiotics for 3 to 6 weeks." Explanation: Treatment of osteomyelitis requires IV antibiotics for 3 to 6 weeks.

A patient having chemotherapy is losing her hair. She asks the nurse if her hair will come back as it originally was. What is the best response by the nurse?

"Your hair will grow back, but the new hair may be a different color or texture."

The nurse is caring for a client newly diagnosed with a primary brain tumor. The client asks the nurse where the tumor came from. What would be the nurse's best response?

"Your tumor originated from a single cell that is genetically different from other cells in the surrounding tissue."

The nurse is performing an assessment on a 10-yr-old child suspected to have Hodgkin's disease. The nurse understands that which assessment findings are specifically characteristic of this disease? Select all that apply. 1. Abdominal pain 2. Fever and malaise 3. Anorexia and weight loss 4. Painful, enlarged inguinal lymph nodes 5. Painless, firm, and movable adenopathy in the cervical area

(1) Abdominal pain (5) Painless, firm, and movable adenopathy in the cervical area

(SELECT ALL THAT APPLY) A client receiving chemotherapy for breast cancer develops myelosuppression. Which instructions should the nurse include in the discharge teaching plan?

(1) Avoid people who have recently received attenuated vaccines. (2) Avoid activities that may cause bleeding. (3) Wash hands frequently. (5) Avoid crowded places, such as shopping malls.

(SELECT ALL THAT APPLY) A nurse is teaching a class about osteoporosis. Which factors place a client at greater risk for developing this disease?

(1) Being postmenopausal, (2) Long-term use of corticosteroids, (6) Sedentary lifestyle

(SELECT ALL THAT APPLY) A client in the terminal stage of cancer is being transferred to hospice care. Which information should the nurse include in the teaching plan regarding hospice care?

(1) Care focuses on controlling symptoms and relieving pain. (2) A multidisciplinary team provides care. (5) Bereavement care is provided to the family.

(SELECT ALL THAT APPLY) After having a lobectomy for lung cancer, a client receives a chest tube connected to a three-chamber chest drainage system. The nurse observes that the drainage system is functioning correctly when she notes which of the following?

(1) Fluctuations in the water-seal chamber occur when the client breathes. (3) Intermittent bubbling occurs in the water-seal chamber. (4) Gentle bubbling occurs in the suction control chamber. (5) Drainage is collecting in the drainage chamber

(SELECT ALL THAT APPLY) After undergoing chemotherapy to treat breast cancer, a client has a white blood cell (WBC) count of 1,300/microliter. Which nursing interventions should be used with this client?

(1) Monitor the client's temperature and vital signs; report temperature higher than 101.3° F (38.5° C). (2) Make sure no one with a viral or bacterial infection enters the client's room. (3) Administer hematopoietic growth factors as prescribed. (4) Reinforce good hand-washing techniques to anyone entering the client's room.

(SELECT ALL THAT APPLY) A client with laryngeal cancer has undergone laryngectomy and is receiving radiation therapy to the head and neck. The nurse should monitor the client for which adverse effects of external radiation?

(1) Xerostomia (2) Stomatitis (5) Dysgeusia

(SELECT ALL THAT APPLY) A client is diagnosed with gout. Which foods should the nurse instruct the client to avoid?

(2) Liver, (3) Cod, (5) Sardines

(SELECT ALL THAT APPLY) A client with bladder cancer undergoes surgical removal of the bladder and construction of an ileal conduit. Which data collection findings indicate that the client is developing complications?

(2) The stoma appears dusky. (3) The stoma protrudes from the skin. (6) The client experiences sharp abdominal pain and rigidity.

(SELECT ALL THAT APPLY) A client with a suspected fracture of the right hip is in the emergency department. Which data collection findings would the nurse expect to see in the client's right leg?

(2)The right leg is shorter than the left leg., (4) The right leg is adducted., (5) The right leg is externally rotated.

(SELECT ALL THAT APPLY) A client is about to undergo total hip replacement surgery. Before the surgery, the nurse conducts a preoperative teaching session with him. The nurse can tell that her teaching has been effective when the client verbalizes the importance of avoiding which actions?

(3) Internally rotating the feet, (4) Bending to pick items up from the floor

(SELECT ALL THAT APPLY) A client is preparing for discharge from the hospital after undergoing an above-the-knee amputation. Which instructions should the nurse include in the teaching plan for this client?

(4) Avoid exposing the skin around the stump to excessive perspiration.(5) Be sure to perform the prescribed exercises., (6) Rub the stump with a dry washcloth for 4 minutes three times per day if the stump is sensitive to touch.

(SELECT ALL THAT APPLY) A client who was involved in a motor vehicle accident has a fractured femur. The nurse caring for the client documents Acute pain as one of the nursing diagnoses in his care plan. Which nursing interventions are appropriate?

(5) Assess the client's perception of pain., (6) Ask the client about methods he has used previously to alleviate pain.

Ataxia

(irregular uncoordinated movements) or loss of balance may be due to cerebellar disorders, Parkinson disease, multiple sclerosis, strokes, brain tumors, inner ear problems, or medications

When joint manipulation is unsuccessful for a client, he is taken to surgery for surgical repair of his hip injury. He is brought to the ICU where you practice nursing for postoperative recovery. After surgery, the nurse implements measures to prevent complications. Select all of the following which are complications that the nurse seeks to prevent.

-Skin breakdown -Wound infection -Pneumonia

A client with osteoporosis is prescribed calcitonin (Miacalcin) 100 units subcutaneously. The medication is available 200 units per ml. How many milliliters will the nurse administer to the client?

0.5

A client with osteoporosis is prescribed calcitonin (Miacalcin) 100 units subcutaneously. The medication is available 200 units per ml. How many milliliters will the nurse administer to the client?

0.5 Explanation: 100 units x 1 ml/200 units = 0.5 ml.

A client receiving external radiation to the left thorax to treat lung cancer has a nursing diagnosis of Risk for impaired skin integrity. Which intervention should be part of this client's care plan? 1. Avoiding using soap on the irradiated areas 2. Wearing a lead apron during direct contact with the client 3. Applying talcum powder to the irradiated areas daily after bathing 4. Removing thoracic skin markings after each radiation treatment

1

A nurse is caring for a recently married, 29-year-old female client, who was diagnosed with acute lymphocytic leukemia. The client is preparing for an allogeneic bone marrow transplant. Which statement by the client demonstrates she understands the informed consent she gave about the diagnosis and treatment? 1. "I'll only need chemotherapy treatment before receiving my bone marrow transplant." 2. "I'll have to remain in the hospital for about 3 months after my transplant." 3. "I always had a good appetite. Even with chemo I shouldn't have to make any changes to my diet." 4. "I should be able to finally start a family after I'm finished with the chemo."

1

After being seen in the oncology clinic, a client with severe bone marrow suppression is admitted to the hospital. The client's cancer therapy consisted of radiation and chemotherapy. When developing the care plan for this client, which nursing diagnosis takes priority? 1. Risk for infection 2. Anxiety 3. Imbalanced nutrition: Less than body requirements 4. Risk for injury

1

The nurse evaluates teaching as effective when a female client states that she will 1. Use sunscreen when outdoors. 2. Obtain a cancer history from her parents. 3. Decrease tobacco smoking from one pack/day to half a pack/day. 4. Exercise 30 minutes 3 times each week.

1

What disadvantages of chemotherapy should the patient be informed about prior to starting the regimen? 1. It targets normal body cells as well as cancer cells. 2. It causes a systemic reaction. 3. It attacks cancer cells during their vulnerable phase. 4. It functions against disseminated disease.

1

When the client complains of increased fatigue following radiotherapy, the nurse knows this is most likely to be related to which factor? 1. Radiation can result in myelosuppression. 2. The cancer cells are dying in large numbers. 3. The cancer is spreading. 4. Fighting off infection is an exhausting venture.

1

Which of the following is a term used to describe the process of programmed cell death? 1. Apoptosis 2. Mitosis 3. Angiogenesis 4. Carcinogenesis

1

Which primary cancer treatment goal is prolonged survival and containment of cancer cell growth? 1. Control 2. Prevention 3. Palliation 4. Cure

1

Which type of hematopoietic stem cell transplantation (HSCT) is characterized by cells from a donor other than the patient? 1. Allogeneic 2. Autologous 3. Homogenic 4. Syngeneic

1

A client has been receiving chemotherapy. Upon assessing the client during morning rounds, the nurse notes the client is now bleeding from intravenous and venipuncture sites. Stool is positive for occult blood. The client is requesting to sit in a chair for a meal. The nurse implements the following interventions: (Select all that apply.) 1. Assess level of consciousness. 2. Apply pressure to the bleeding sites. 3. Check intake and output records. 4. Assist the client to a chair. 5. Monitor vital signs once a shift.

1, 2, 3

When a patient takes vincristine, an antineoplastic agent that inhibits DNA and protein synthesis, the patient needs to be informed that he should report which of the following symptoms that would be an expected side-effect of motor neuropathy? Select all that apply. 1. Loss of balance and coordination 2. Cramps and spasms in the legs 3. Muscle weakness 4. Burning and tingling sensations in the extremities

1, 2, 3

At a health fair, a woman, age 43, with a family history of osteoporosis asks the nurse how much calcium she should consume. The nurse tells her that the recommended daily calcium intake for premenopausal women is:

1,000 to 1,200 mg.

A client with breast cancer was admitted earlier in the day with sepsis and complaints of nausea, pain, and lethargy. She completed her third round of chemotherapy last week. A physician has just ordered IV D5 half normal saline solution at 150 ml/hr. Which notes would the nurse expect to see documented in the client's chart? Select all that apply. 1. Flushed skin. 2. Client groggy but answers questions appropriately. 3. Auscultation of lungs; Clear bilaterally. 4. Abdomen soft and non-tender. Bowel sounds present in all quadrants. 5. BP 96/50, HR 114/min, RR 22/min, T 103.2F, Urine output 150ml in the past 6 hours. 6. BUN 35, creatinine 0.4, Na 134, K 4.2, Cl 104.

1. Flushed skin. 2. Client groggy but answers questions appropriately. 5. BP 96/50, HR 114/min, RR 22/min, T 103.2F, Urine output 150ml in the past 6 hours. 6. BUN 35, creatinine 0.4, Na 134, K 4.2, Cl 104.

Age 21 year-old client undergoes bone marrow aspiration at the clinic to establish a diagnosis of possible lymphoma. Which statement made by the client demonstrates proper understanding of discharge teaching select all that apply 1. I will take Tylenol for pain 2. I do not need to inspect the puncture site 3. I will not be able to play basketball for the next two days 4. I will take aspirin if I have pain 5. I can apply an ice pack or a cold compress to the puncture site

1. I will take Tylenol for pain 3. I will not be able to play basketball for the next two days 5. I can apply an ice pack or a cold compress to the puncture site

A 5-year-old client has just received stem cell transplantation as treatment for his leukemia. What are the post procedural nursing interventions for clients receiving any form of stem cell transplantation? 1. Closely monitor the client for at least 3 days. 2. Closely monitor the client for at least 3 months. 3. Closely monitor the client for at least 5 months. 4. Closely monitor the client for at least 4 weeks.

2

A client is receiving chemotherapy to treat breast cancer. Which assessment finding indicates a chemotherapy-induced complication? 1. Blood pressure of 120/64 to 130/72 mm Hg 2. Serum potassium level of 2.6 mEq/L 3. Urine output of 400 ml in 8 hours 4. Sodium level of 142 mEq/L

2

A decrease in circulating white blood cells is 1. thrombocytopenia. 2. leukopenia. 3. neutropenia. 4. granulocytopenia.

2

A young female client has received chemotherapeutic medications and asks about any effects the treatments will have related to her sexual health. The most appropriate statement by the nurse is 1. "You will continue having your menses every month." 2. "You will need to practice birth control measures." 3. "You will experience menopause now." 4. "You will be unable to have children."

2

During chemotherapy, an oncology client has a nursing diagnosis of Impaired oral mucous membrane related to decreased nutrition and immunosuppression secondary to the cytotoxic effects of chemotherapy. Which nursing intervention is most likely to decrease the pain of stomatitis? 1. Recommending that the client discontinue chemotherapy 2. Providing a solution of viscous lidocaine for use as a mouth rinse 3. Checking regularly for signs and symptoms of stomatitis 4. Monitoring the client's platelet and leukocyte counts

2

Palliation refers to 1. the spread of cancer cells from the primary tumor to distant sites. 2. relief of symptoms associated with cancer. 3. the lowest point of white blood cell depression after therapy that has toxic effects on the bone marrow. 4. hair loss related to the treatment of cancer.

2

The client has finished the first round of chemotherapy. Which statement made by the client indicates a need for further teaching by the nurse? 1. "Hair loss may not occur until after the second round of therapy." 2. "I can continue taking my vitamins and herbs because they make me feel better." 3. "I will eat clear liquids for the next 24 hours." 4. "I will use birth control measures until after all treatment is completed."

2

The clinic nurse is caring for a client who has just been diagnosed with a tumor. The client says to the nurse "The doctor says my tumor is benign. What does that mean?" What is the nurse's best response? 1. "Benign tumors can spread from one place to another." 2. "Benign tumors don't usually cause death." 3. "Benign tumors grow very rapidly." 4. "Benign tumors invade surrounding tissue."

2

The nurse is caring for a patient undergoing an incisional biopsy. Which of the following statements does the nurse understand is true about an incisional biopsy? 1. It treats cancer with lymph node involvement. 2. It removes a wedge of tissue for diagnosis. 3. It removes an entire lesion and surrounding tissue. 4. It is used to remove the cancerous cells using a needle.

2

The nurse is conducting a screening for familial predisposition of cancer. Which of the following should the nurse note as a possible indication of hereditary cancer syndrome? 1. Onset of cancer after age 50 in family member 2. An aunt and uncle diagnosed with cancer 3. A second cousin diagnosed with cancer 4. A first cousin diagnosed with cancer

2

The nurse is teaching a client newly diagnosed with cancer about chemotherapy. The nurse tells the client he'll receive an antitumor antibiotic. The nurse knows that this type of medications is: 1. cell-cycle specific in the S phase. 2. cell-cycle nonspecific. 3. cell-cycle specific in the P phase. 4. cell-cycle specific in the M phase.

2

The nurse working on a bone marrow unit knows that it is a priority to monitor which of the following in a client who has just undergone a stem cell transplant? 1. Monitor the client's toilet patterns. 2. Monitor the client closely to prevent infection. 3. Monitor the client's heart rate. 4. Monitor the client's physical condition.

2

What should the nurse tell a female client who is about to begin chemotherapy and anxious about losing her hair? 1. Her hair will grow back within 2 months post therapy. 2. She should consider getting a wig or cap before she loses her hair. 3. Alopecia related to chemotherapy is relatively uncommon. 4. Her hair will grow back the same as it was before treatment.

2

When caring for a client who is receiving external beam radiation, which is the key point for the nurse to incorporate into the plan of care? 1. Avoid showering or washing over skin markings. 2. Inspect the skin frequently. 3. Time, distance, and shielding 4. The use of disposable utensils and wash cloths

2

Which of the following is a characteristic of a malignant tumor? 1. It is usually slow growing. 2. It gains access to the blood and lymphatic channels. 3. It grows by expansion. 4. It demonstrates cells that are well differentiated.

2

Which of the following occurs when there is accumulation of fluid in the pericardial space that compresses the heart? 1. DIC 2. Cardiac tamponade 3. SIADH 4. Superior Vena Cava Syndrome (SVCS)

2

Which of the following would be inconsistent as a common side effect of chemotherapy? 1. Alopecia 2. Weight gain 3. Fatigue 4. Myelosuppression

2

Which statement by a client undergoing external radiation therapy indicates the need for further teaching? 1. "I'll not use my heating pad during my treatment." 2. "I'm worried I'll expose my family members to radiation." 3. "I'll wash my skin with mild soap and water only." 4. "I'll wear protective clothing when outside."

2

Which statement by a client undergoing external radiation therapy indicates the need for further teaching? 1. "I'll wash my skin with mild soap and water only." 2. "I'm worried I'll expose my family members to radiation." 3. "I'll not use my heating pad during my treatment." 4. "I'll wear protective clothing when outside."

2

Which type of vaccine uses the patient's own cancer cells, which are prepared for injection, back into the patient? 1. Allogeneic 2. Autologous 3. Therapeutic 4. Prophylactic

2

A client with a history of colon cancer has a permanent colostomy. The nurse must irrigate the colostomy to prepare the client for diagnostic testing. When irrigating, how far into the stoma should the nurse insert the lubricated catheter?

2" to 4"

You are teaching clients about cancer prevention and explain that the root cause of cancer is damage to cellular deoxyribonucleic acid (DNA) as a result from multiple factors. You explain that which of the following are possible carcinogens? Select all that apply. 1. Gender 2. Dietary substances 3. Environmental factors 4. Viruses 5. Age

2, 3, 4

A newly diagnosed cancer client is crying and states the following to the nurse: "I promised God that I will be a better person if I can just get better." What is the appropriate assessment of this comment by the nurse? 1. Anger directed toward nursing staff is not unusual in dealing with cancer clients. 2. The client is just trying to protect self from potential loss. 3. The cancer is viewed as a punishment from past actions. 4. Loss is inevitable so client is making final plans.

3

A nurse is administering daunorubicin (DaunoXome) to a patient with lung cancer. Which situation requires immediate intervention? 1. The client begins to shiver. 2. The client states he is nauseous. 3. The I.V. site is red and swollen. 4. The laboratory reports a white blood cell (WBC) count of 1,000/mm3.

3

A nurse is administering daunorubicin through a peripheral I.V. line when the client complains of burning at the insertion site. The nurse notes no blood return from the catheter and redness at the I.V. site. The client is most likely experiencing which complication? 1. Erythema 2. Flare 3. Extravasation 4. Thrombosis

3

A nurse is caring for a client who is receiving chemotherapy and has a platelet count of 30,000/mm3. Which statement by the client indicates a need for additional teaching? 1. "I take a stool softener every morning." 2. "I use an electric razor to shave." 3. "I floss my teeth every morning." 4. "I removed all the throw rugs from the house."

3

A patient is scheduled for cryosurgery for cervical cancer and tells the nurse, "I am not exactly sure what the doctor is going to do." What is the best response by the nurse? 1. "The physician is going to use medication to inject the area." 2. "The physician is going to use radiofrequency to ablate the area." 3. "The physician is going to use liquid nitrogen to freeze the area." 4. "The physician is going to use a laser to remove the area."

3

A side-effect of chemotherapy is renal damage. To prevent this, the nurse should: 1. Withhold medication when the blood urea nitrogen level exceeds 20 mg/dL. 2. Modify the diet to acidify the urine, thus preventing uric acid crystallization. 3. Encourage fluid intake, if possible, to dilute the urine. 4. Limit fluids to 1,000 mL/day to minimize stress on the renal tubules.

3

According to the TNM classification system, T0 means there is 1. distant metastasis. 2. no regional lymph node metastasis. 3. no evidence of primary tumor. 4. no distant metastasis.

3

After being seen in the oncology clinic, a client with severe bone marrow suppression is admitted to the hospital. The client's cancer therapy consisted of radiation and chemotherapy. When developing the care plan for this client, which nursing diagnosis takes priority? 1. Risk for injury 2. Imbalanced nutrition: Less than body requirements 3. Risk for infection 4. Anxiety

3

After cancer chemotherapy, a client experiences nausea and vomiting. The nurse should assign highest priority to which intervention? 1. Withholding fluids for the first 4 to 6 hours after chemotherapy administration 2. Serving small portions of bland food 3. Administering metoclopramide and dexamethasone as ordered 4. Encouraging rhythmic breathing exercises

3

An important nursing function is monitoring factors that may indicate that bleeding is occurring. One serum indicator is a (an): 1. Lymphocyte count of 30%. 2. Reticulocyte count of 1%. 3. Platelet count of 60,000/mm3. 4. Neutrophil count of 60%.

3

During which step of cellular carcinogenesis do cellular changes exhibit increased malignant behavior? 1. Initiation 2. Promotion 3. Progression 4. Prolongation

3

The nurse at the clinic explains to the patient that the surgeon will be removing a mole on the patient's back that has the potential to develop into cancer. The nurse informs the patient that this is what type of procedure? 1. Palliative 2. Diagnostic 3. Prophylactic 4. Reconstructive

3

The nurse is caring for a client who is scheduled for chemotherapy. Which is the best statement the nurse can make about the client experiencing chemotherapy-induced alopecia? 1. "Clients with alopecia will have delay in grey hair." 2. "New hair growth will return without any change to color or texture." 3. "The hair loss is temporary." 4. "Wigs can be used after the chemotherapy is completed."

3

The nurse is providing client teaching for a client undergoing chemotherapy. What dietary modifications should the nurse advise? 1. Eat wholesome meals. 2. Avoid intake of fluids. 3. Avoid spicy and fatty foods. 4. Eat warm or hot foods.

3

The nurse working on a bone marrow unit knows that it is a priority to monitor which of the following in a client who has just undergone a stem cell transplant? 1. Monitor the client's heart rate. 2. Monitor the client's physical condition. 3. Monitor the client closely to prevent infection. 4. Monitor the client's toilet patterns.

3

Which of the following is a term used to describe the process of programmed cell death? 1. Mitosis 2. Carcinogenesis 3. Apoptosis 4. Angiogenesis

3

Which oncologic emergency involves the failure in the negative feedback mechanism that normally regulates the release of antidiuretic hormone (ADH)? 1. Cardiac tamponade 2. Disseminated intravascular coagulation (DIC) 3. Syndrome of inappropriate antidiuretic hormone release (SIADH) 4. Tumor lysis syndrome

3

While doing a health history, a client tells you that her mother, grandmother, and sister died of cancer. The client wants to know what she can do to keep from getting cancer. What would be your best response? 1. "With your family history, there is nothing you can do to prevent getting cancer." 2. "If you eat right, exercise, and get enough rest, you can prevent breast cancer." 3. "You can't prevent cancer, but you can have your blood analyzed for tumor markers to see what your risk level is." 4. "Cancer often skips a generation, so don't worry about it."

3

As part of chemotherapy education, the nurse teaches a female client about the risk for bleeding and self care during the period of greatest bone marrow suppression (the nadir). The nurse understands that further teaching is needed if the client makes which statement? 1. "I should avoid blowing my nose." 2. "I may need a platelet transfusion if my platelet count is too low." 3. "I'm going to take an aspirin for my headache as soon as I get home." 4. "I will count the number of pads and tampons I use when menstruating."

3. "I'm going to take an aspirin for my headache as soon as I get home."

A nurse is monitoring a client for signs and symptoms related to superior vena cava syndrome. Which of the following is an early sign of this oncological emergency? 1. Cyanosis 2. Arm Edema 3. Periorbital edema 4. Mental Status Changes

3. Periorbital edema

A bowel resection is scheduled for a client with the diagnosis of colon cancer with metastasis to the liver and bone. Which statement by the nurse best explains the purpose of the surgery? 1. "This surgery will prevent further tumor growth." 2. "Removing the tumor is a primary treatment for colon cancer." 3. "Once the tumor is removed, cell pathology can be determined." 4. "Tumor removal will promote comfort."

4

A client complains of sporadic epigastric pain, yellow skin, nausea, vomiting, weight loss, and fatigue. Suspecting gallbladder disease, the physician orders a diagnostic workup, which reveals gallbladder cancer. Which nursing diagnosis is appropriate for this client? 1. Disturbed body image 2. Chronic low self-esteem 3. Impaired swallowing 4. Anticipatory grieving

4

A client has been receiving chemotherapy to treat cancer. Which assessment finding suggests that the client has developed stomatitis? 1. Yellow tooth discoloration 2. Rust-colored sputum 3. White, cottage cheese-like patches on the tongue 4. Red, open sores on the oral mucosa

4

A nurse is caring for a client receiving chemotherapy. Which assessment finding places the client at the greatest risk for an infection? 1. Ate 75% of all meals during the day 2. White blood cell (WBC) count of 9,000 cells/mm3 3. Temperature of 98.3° F (36.8° C) 4. Stage 3 pressure ulcer on the left heel

4

A nurse is caring for a client receiving chemotherapy. Which nursing action is most appropriate for handling chemotherapeutic agents? 1. Disconnect I.V. tubing with gloved hands. 2. Throw I.V. tubing in the trash after the infusion is stopped. 3. Break needles after the infusion is discontinued. 4. Wear disposable gloves and protective clothing.

4

A patient is admitted for an excisional biopsy of a breast lesion. What intervention should the nurse provide for the care of this patient? 1. Clarify information provided by the physician. 2. Counsel the patient about the possibility of losing her breast. 3. Provide aseptic care to the incision postoperatively. 4. Provide time for the patient to discuss her concerns.

4

A patient with brain tumor is undergoing radiation and chemotherapy for treatment of cancer. Of late, the patient is complaining of swelling in the gums, tongue, and lips. Which of the following is the most likely cause of these symptoms? 1. Nadir 2. Extravasation 3. Neutropenia 4. Stomatitis

4

For a client newly diagnosed with radiation-induced thrombocytopenia, the nurse should include which intervention in the care plan? 1. Administering aspirin if the temperature exceeds 102° F (38.8° C) 2. Placing the client in strict isolation 3. Providing for frequent rest periods 4. Inspecting the skin for petechiae once every shift

4

For a client newly diagnosed with radiation-induced thrombocytopenia, the nurse should include which intervention in the care plan? 1. Providing for frequent rest periods 2. Administering aspirin if the temperature exceeds 102° F (38.8° C) 3. Placing the client in strict isolation 4. Inspecting the skin for petechiae once every shift

4

In which phase of the cell cycle does cell division occur? 1. G1 phase 2. S phase 3. G2 phase 4. Mitosis

4

The client is receiving a vesicant anti neo plastic for treatment of cancer. Which assessment finding would require the nurse to take immediate action? 1. Stomatitis 2. Bone pain 3. Nausea and vomiting 4. Extra vasation

4

The nurse is caring for a client who is scheduled for chemotherapy. Which is the best statement the nurse can make about the client experiencing chemotherapy-induced alopecia? 1. "New hair growth will return without any change to color or texture." 2. "Wigs can be used after the chemotherapy is completed." 3. "Clients with alopecia will have delay in grey hair." 4. "The hair loss is temporary."

4

The nurse is evaluating the client's risk for cancer and recommends changes when the client states she 1. drinks 1 glass of wine at dinner each night 2. uses the treadmill for 30 minutes on 5 days each week 3.works as a secretary at a medical radiation treatment center 4. eats red meat such as steaks or hamburgers every day

4

The physician is attending to a 72-year-old patient with a malignant brain tumor. The physician recommends immediate radiation therapy. Which of the following is a reason for the physician's recommendation? 1. To destroy marginal tissues 2. To remove the tumor from the brain 3. To analyze involved lymph nodes 4. To prevent the formation of new cancer cells

4

Which of the following does a nurse thoroughly evaluate before a bone marrow transplant (BMT) procedure? 1. Allergy history 2. Family history 3. Drug history 4. Blood studies

4

At what age is an individual most at risk for acquiring acute lymphocytic leukemia (ALL)?

4 to 12 years

A client who had a modified radical mastectomy for breast cancer has the drain removed and the dressing changed by the surgeon on the third postoperative day. The client appears shocked when she sees the operative area and states, "I look horrible! Will it ever look better?" Which response by the nurse would be the most appropriate? 1. "Would you like to meet another client who had a mastectomy?" 2. "After it heals and you're dressed, you won't even know you had surgery." 3. Don't worry. You know the tumor is gone, and the area will heal very soon." 4. "You are shocked by the change in your appearance from the surgery?"

4. "You are shocked by the change in your appearance from the surgery?"

A 32-year-old woman meets with the nurse on her first office visit since undergoing a left mastectomy. When asked how she is doing, the woman states her appetite is still not good, she is not getting much sleep because she doesn't go to bed until her husband is asleep, and she is really anxious to get back to work. Which of the following nursing interventions should the nurse explore to support the client's current needs? 1. Call the physician to discuss allowing the client to return to work earlier. 2. Suggest that the client learn relaxation techniques to help with her insomnia 3. Perform a nutritional assessment to assess for anorexia 4. Ask open-ended questions about sexuality issues related to her mastectomy

4. Ask open-ended questions about sexuality issues related to her mastectomy

Which client has the highest risk of ovarian cancer?

45-year-old woman who has never been pregnant

A client receiving chemotherapy for metastatic colon cancer is admitted to the hospital because of prolonged vomiting. Assessment findings include irregular pulse of 120 bpm, blood pressure 88/48 mm Hg, respiratory rate of 14 breaths/min, serum potassium of 2.9 mEq/L (2.9 mmol/L), and arterial blood gas—pH 7.46, PCO2 45 mm Hg (6.0 kPA), PO2 95 mm Hg (12.6 kPa), bicarbonate level 29 mEq/L (29 mmol/L). The nurse should implement which prescription first?

5% Dextrose in 0.45% normal saline with KCl 40 mEq/L at 125 mL/h Explanation: The vital signs suggest that the client is dehydrated from the vomiting, and the nurse should first infuse the IV fluids with the addition of potassium. There is no indication that the client needs oxygen at this time since the PO2 is 95 (12.6 kPa). Although the client has a rapid and irregular pulse, the infusion of fluids may cause the heart rate to return to normal, and the 12-lead ECG can be prescribed after starting the intravenous fluids.

The nurse is caring for a client admitted for pneumonia with a history of hypertension and heart failure. The client has reported at least one fall in the last 3 months. The client may ambulate with assistance, has a saline lock in place, and has demonstrated appropriate use of the call light to request assistance. Using the Morse Fall Scale (see chart), what is this client's total score and risk level?

60, high risk Explanation: Several factors designate this client as a high fall risk based on the Morse Fall Scale: history of falling (25), secondary diagnosis (15), plus IV access (20). The client's total score is 60. There is also concern that the client's gait is at least weak if not impaired due to hospitalization for pneumonia, which may add to the client's fall risk. After evaluating the client's risk, the nurse must develop a plan and take action to maximize the client's safety.

A nurse is developing a care plan for bone marrow suppression, the major dose-limiting adverse reaction to floxuridine (FUDR). How long after drug administration does bone marrow suppression become noticeable?

7 to 14 days

A client with suspected osteomalacia has a fractured tibia and fibula. What test would give a definitive diagnosis of osteomalacia?

A bone biopsy

A registered nurse (RN) on the oncology floor is busy with one client, so she delegates care of another client to her coworker, a licensed practical nurse (LPN). The client that the LPN begins caring for requires a 3-hour chemotherapy infusion. Which of the following actions is in accordance with her state's nurse practice act?

A chemotherapy-certified RN must begin the chemotherapy infusion; then the LPN may monitor the client.

A few minutes after beginning a blood transfusion, a nurse notes that a client has chills, dyspnea, and urticaria. The nurse reports this to the physician immediately because the client probably is experiencing which problem?

A hemolytic allergic reaction caused by an antigen reaction

"A few minutes after beginning a blood transfusion, the nurse notes that the client has chills, dyspnea, and urticaria. The nurse reports this to the physician immediately because the client probably is experiencing:

A hemolytic allergic reaction caused by an antigen reaction.

You are an oncology nurse giving chemotherapy in a short stay area. One client confides to you that they are very depressed. What is depression?

A normal reaction to the diagnosis of cancer.

A nurse is assessing an older patient. What will the the nurse suspecting leukemia ask the patient to describe? Select all that apply. A. Fatigue B.An unexplained increase in energy C. Unexplained fevers D. Night Sweats

A. Fatigue C. Unexplained fevers D. Night Sweats

Which nursing diagnosis is highest-priority for a child undergoing chemotherapy and experiencing nausea and vomiting? A. Fluid and Electrolyte Imbalance B. Alterations in Nutrition C. Alterations in Skin Integrity D. Body Image Disturbances

A. Fluid and Electrolyte Imbalance

When caring for a client with a history of benign prostatic hypertrophy (BPH), the nurse should do which of the following? Select all that apply. A. Provide privacy and time for the client to void. B. Monitor intake and output. C. Catheterize the client for post void residual urine. D. Ask the client if he has urinary retention. E. Test the urine for hematuria.

A. Provide privacy and time for the client to void. B. Monitor intake and output. D. Ask the client if he has urinary retention. E. Test the urine for hematuria.

In assessing a postmastectomy client, the nurse determines that the client is in denial. The nurse can best respond by:

Accepting the denial.

Which nursing goal is appropriate for a client with multiple myeloma?

Achieve effective management of bone pain.

A nurse is working with a dying client and his family. Which communication technique is most important to use?

Active listening

A rock climber has sustained an open fracture of the right tibia after a 20-foot fall. The nurse plans to assess the client for which potential complications? Select all that apply.

Acute compartment syndrome (ACS) Correct Fat embolism syndrome (FES) Correct Osteomyelitis Correct Correct Feedback: Correct: Acute compartment syndrome (ACS) is a serious condition in which increased pressure within one or more compartments reduces circulation to the area. Correct: A fat embolus is a serious complication in which fat globules are released from yellow bone marrow in the bloodstream within 12 to 48 hours after the injury. FES usually results from long bone fracture or fracture repair but is occasionally seen in clients who have received a total joint replacement. Correct: Bone infection, or osteomyelitis, is most common in open fractures.

The nurse is caring for the client with chronic osteomyelitis of the jaw with a draining wound. Which nursing diagnosis is appropriate for the client? Select all that apply.

Acute pain. Disturbed body image. Imbalanced nutrition: less than body requirements.

"A client is prescribed tamoxifen (Nolvadex), 20 mg by mouth twice per day for treatment of breast cancer. The client complains to the nurse that she has worsening bone pain. How should the nurse respond?

Acute worsening of bone pain commonly indicates that the drug will produce a good response.

To combat the most common adverse effects of chemotherapy, a nurse should prepare to do which of the following?

Administer an antiemetic.

To combat the most common adverse effects of chemotherapy, a nurse should prepare to do which of the following?

Administer an antiemetic. Explanation: Antiemetics, antihistamines, and certain steroids treat nausea and vomiting, which is a common adverse effect of chemotherapy. Lack of knowledge about adverse effects or information about support groups are important, but they are not common adverse effects. Clients often require appetitie enhancers to stimulate appetite after receiving chemotherapy. Antimetabolites and tumor antibiotics are classes of chemotherapeutic medications.

Two days after surgery to amputate his left lower leg, a client states that he has pain in the missing extremity. Which action by the nurse is most appropriate?

Administer medication, as ordered, for the reported discomfort. Explanation: The sensation of pain and discomfort in an amputated extremity is known as phantom pain. Phantom pain is a normal occurrence after an amputation. It should be treated with medication. The nurse doesn't need to contact the physician at this time. Consultation with the psychologist isn't indicated, and the nurse shouldn't take this action without consulting the physician.

A client receiving 5-fluorouracil is experiencing nausea and vomiting. Which is the nurse's best course of action?

Administer odansetron prior to administering the 5-fluorouracil

A client receiving 5-fluorouracil is experiencing nausea and vomiting. Which is the nurse's best course of action?

Administer odansetron prior to administering the 5-fluorouracil Explanation: Fluorouracil, an antimetabolite antineoplastic medication, may cause nausea, vomiting, diarrhea, bone marrow suppression, and stomatitis. Premedication with an antiemetic medication such as odansetron will prevent nausea and vomiting during treatment.

A client is admitted with acute osteomyelitis that developed after an open fracture of the right femur. When planning this client's care, the nurse should anticipate which measure?

Administering large doses of I.V. antibiotics as ordered

A client is admitted with acute osteomyelitis that developed after an open fracture of the right femur. When planning this client's care, the nurse should anticipate which measure? a) Withholding all oral intake b) Instructing the client to ambulate twice daily c) Administering large doses of I.V. antibiotics as ordered d) Administering large doses of oral antibiotics as ordered

Administering large doses of I.V. antibiotics as ordered Explanation: Treatment of acute osteomyelitis includes large doses of I.V. antibiotics (after blood cultures identify the infecting organism). Surgical drainage may be indicated, and the affected bone is immobilized. The client usually requires I.V. fluids to maintain hydration, but oral intake isn't necessarily prohibited.

A client is admitted with acute osteomyelitis that developed after an open fracture of the right femur. When planning this client's care, the nurse should include which measure?

Administering large doses of I.V. antibiotics as prescribed

After cancer chemotherapy, a client experiences nausea and vomiting. The nurse should assign highest priority to which intervention?

Administering metoclopramide (Reglan) and dexamethasone (Decadron) as ordered

A client is experiencing an acute exacerbation of rheumatoid arthritis. What should the nursing priority be?

Administering ordered analgesics and monitoring their effects Explanation: An acute exacerbation of rheumatoid arthritis can be very painful, and the nurse should make pain management the priority. Client teaching, skin care, and supplying adaptive devices are important, but these actions don't not take priority over pain management.

When is it advisable for the nurse to apply heat to a sprain or a contusion?

After 2 days

Which of the following is a risk factor associated with prostate cancer?

Age older than 40

A nurse is checking the laboratory results of an adult client with colon cancer admitted for further chemotherapy. The client has lost 30 lb (13.6 kg) since initiation of the treatment. Which laboratory result should be reported to the health care provider?

Albumin level of 2.8 g/dl (28 g/L). The nurse must recognize that an albumin level of 2.8 g/dl (28 g/L) indicates catabolism and potential for malnutrition. Normal albumin is 3.5 to 5.0 g/dl (30 to 50 g/L); less than 3.5 g/dl (35 g/L) indicates malnutrition. The other laboratory results are normal.

The nurse is caring for patient with a hip fracture. The physician orders the patient to start on a bisphosphonate. Which medication would the nurse document as given?

Alendronate (Fosamax)

The nurse is caring for patient with a hip fracture. The physician orders the patient to start on a bisphosphonate. Which medication would the nurse document as given? a) Raloxifene (Evista) b) Alendronate (Fosamax) c) Teriparatide (Forteo) d) Denosumab (Prolia)

Alendronate (Fosamax) Explanation: Alendronate (Fosamax) is a bisphosphonate medication. Raloxifene (Evista) is a selective estrogen receptor modulator. Terparatide (Forteo) is an anabolic agent, and denosumab (Prolia) is a monoclonal antibody agent.

The nurse may be asked to administer which medication to a client to counteract the increase in uric acid and subsequent hyperuricemia resulting from the metabolic waste buildup from rapid tumor lysis?

Allopurinol (Zyloprim)

"A client has an abnormal result on a Papanicolaou test. After admitting that she read her chart while the nurse was out of the room, the client asks what dysplasia means. Which definition should the nurse provide?

Alteration in the size, shape, and organization of differentiated cells

The nurse plans to refer a client with an amputation and the client's family to which community resource?

Amputee Coalition of America (ACA) Correct: The ACA is an available resource for clients with amputations and supports them and their families.

Several days before admission, a client reports finding a small lump in the left breast near the nipple. What should the nurse tell the client to do?

Answer :Inform the physician immediately.

A client suspected of having gallbladder cancer is admitted to the hospital for a series of diagnostic testing. What objective data collected by the nurse would be indicative of gallbladder cancer? Select all that apply.

Answer: • Jaundice • Weight loss of 10 kg in 2 months

The nurse is caring for a client with bronchogenic carcinoma. Which nursing interventions are essential in the care of the client receiving a chemotherapeutic regimen? Select all that apply.

Answer: • Maintain a patent airway. • Alleviate anxiety by explaining procedures and care delivery. • Instruct the client on signs and symptoms of infection. Rationale: Maintaining a patent airway is the first concern in a client with a condition that may compromise the airway. Therefore, ineffective airway clearance related to obstruction by a tumor or secretions takes highest priority and should be monitored at all times, especially during chemotherapy. All care should be explained prior to performing so that the client has alleviation of anxiety related to the care.

A client seeks care for hoarseness that has lasted for 1 month. To elicit the most appropriate information about this problem, the nurse should ask which question?

Answer: "Do you smoke cigarettes, cigars, or a pipe?" Rationale: persistent hoarseness may signal throat cancer, which is commonly associated with tobacco use. To asses the client's risk for throat cancer, the nurse should ask about smoking habits.

Which statement by a client undergoing external radiation therapy indicates the need for further teaching?

Answer: "I'm afraid I'll expose my family members to radiation."

A client diagnosed with leukemia asks the nurse why he must undergo bone marrow aspiration. Which response by the nurse is best?

Answer: "To determine what type of leukemia you have; your physician will explain the procedure to you in more detail." Rationale:Bone marrow aspiration identifies the type of white blood cell involved, which helps determine the type of leukemia and guide treatment. The nurse should offer a simple explanation of why the procedure is being performed, and tell the client that the physician will explain the procedure fully before informed consent is obtained.

As part of a primary cancer prevention program, an oncology nurse speaks to the public at a health fair. When someone asks about laryngeal cancer, which statement by the nurse would be most helpful?

Answer: "To reduce the risk of developing laryngeal cancer you shouldn't smoke or drink alcohol."

A client in the final stages of terminal cancer tells his nurse: "I wish I could just be allowed to die. I'm tired of fighting this illness. I have lived a good life. I only continue my chemotherapy and radiation treatments because my family wants me to." What is the nurse's best response?

Answer: "Would you like to meet with your family and your physician about this matter?" Rationale: The nurse has a moral and professional responsibility to advocate for clients who experience decreased independence, loss of freedom of action, and interference with their ability to make autonomous choices. Coordinating a meeting between the physician and family members may give the client an opportunity to express his wishes and promote awareness of his feelings as well as influence future care decisions.

During a breast examination, which finding most strongly suggests that the client has breast cancer?

Answer: A fixed nodular mass with dimpling of the overlying skin

The ABCD method offers one way to assess skin lesions for possible skin cancer. What does the A stand for?

Answer: Asymmetry

A client receiving external radiation to the left thorax to treat lung cancer has a nursing diagnosis of Risk for impaired skin integrity. Which intervention should be part of this client's plan of care?

Answer: Avoiding using soap on the irradiated areas

A 45-year-old client receiving radiation therapy for thyroid cancer reports mouth and throat pain. While inspecting the mouth and throat, the nurse notices white patches and ulcerations in the oral mucosa. What do these findings suggest?

Answer: Candidiasis Rationale: White patches and ulcers in the mouth and throat suggest candidiasis. Candidiasis is common in immunocompromised clients and in clients receiving radiation therapy.

A client diagnosed with acute myelocytic leukemia (AML) has been receiving chemotherapy. During the last two cycles of chemotherapy, the client developed severe thrombocytopenia requiring multiple platelet transfusions. The client is now scheduled to receive a third cycle. How can the nurse best detect early signs and symptoms of thrombocytopenia?

Answer: Closely observe the client's skin for petechiae and bruising.

A client is receiving chemotherapy to treat breast cancer. Which data collection finding indicates a fluid and electrolyte imbalance induced by chemotherapy?

Answer: Dry oral mucous membranes and cracked lips

What should a male client older than age 50 do to help ensure early identification of prostate cancer?

Answer: Have a digital rectal examination and prostate-specific antigen (PSA) test done yearly.

Which of the following actions displayed by a grieving husband over his dying wife would cause the nurse to suggest counseling?

Answer: He refuses to acknowledge his wife's family and blames them for her current health problems.

A client with metastatic brain cancer is admitted to the oncology floor. According to the Patient Self-Determination Act of 1991 (PSDA), what is the hospital required to do concerning the execution of advance directives?

Answer: Inform the client or legal guardian of his right to execute an advance directive.

For a client newly diagnosed with radiation-induced thrombocytopenia, the nurse should include which intervention in the plan of care?

Answer: Inspecting the skin for petechiae once every shift

A client is undergoing a left modified radical mastectomy for breast cancer. Postoperatively, blood pressure should be obtained from the client's right arm, and the left arm and hand should be elevated as much as possible to prevent which condition?

Answer: Lymphedema

An oncology nurse educator is speaking to a women's group about breast cancer. Questions and comments from the audience reveal a misunderstanding of some aspects of the disease. Various members of the audience have made all of the following statements. Which statement is accurate?

Answer: Men can develop breast cancer.

Which finding is an early indicator of bladder cancer?

Answer: Painless, intermittent hematuria

Which nursing intervention is most appropriate for a client with multiple myeloma?

Answer: Preventing bone injury Rationale: When caring for a client with multiple myeloma, the nurse should focus on relieving pain, preventing bone injury and infection, and maintaining hydration.

The nurse is reviewing the chart of a client receiving chemotherapy for lung cancer. She is alarmed to note that the client has developed thrombocytopenia. Which precaution should the nurse implement to ensure the clients safety?

Answer: Provide the client with an electric razor.

During chemotherapy, an oncology client has a nursing diagnosis of Impaired oral mucous membrane related to decreased nutrition and immunosuppression secondary to the cytotoxic effects of chemotherapy. Which nursing intervention is most likely to decrease the pain of stomatitis?

Answer: Providing a solution of hydrogen peroxide and water for use as a mouth rinse

A client has been receiving chemotherapy to treat cancer. Which data collection finding suggests that the client has developed stomatitis (inflammation of the mouth)?

Answer: Red, open sores on the oral mucosa

The nurse is caring for a client who has just had a modified radical mastectomy with immediate reconstruction. She's in her 30s and has two young children. Although she's worried about her future, she seems to be adjusting well to her diagnosis. What should the nurse do to support her coping?

Answer: Refer the client to the American Cancer Society's (Canadian Cancer Society's) Reach for Recovery program or another support program. Rationale: the nurse can refer her to talk to others who have had similar experiences through a support group.

A client with stage II ovarian cancer undergoes a total abdominal hysterectomy and bilateral salpingo-oophorectomy with tumor resection, omentectomy, appendectomy, and lymphadenectomy. During the second postoperative day, which data collection finding would raise concern in the nurse?

Answer: Shallow breathing and increasing lethargy Rationale: Shallow breathing with a change in level of consciousness such as increased lethargy may indicate a respiratory complication such as atelectasis or carbon dioxide retention.

The nurse is teaching a group of men about prostate health. Which of the following would be most important for the nurse to tell the men?

Answer: Starting at age 50, men should talk to a doctor about the pros and cons of PSA testing so they can decide if testing is the right choice for them"

A nurse is preparing a client for discharge after a prolonged hospitalization in which the client had a colon resection and colostomy formation for treatment of colon cancer. The client's family has concerns about managing his care at home. Which factor is most important in ensuring successful home care?

Answer: Support from friends and family Rationale: Home care success depends on a key factor: support from friends and family. Ability to care for the colostomy, the client's age, and the complexity of his care aren't as important to the success of home care as is the support from friends and family

A client with bladder cancer undergoes surgical removal of the bladder and construction of an ileal conduit. Which data collection findings indicate that the client is developing complications? Select all that apply.

Answer: The stoma appears dusky. • The stoma protrudes from the skin. • The client experiences sharp abdominal pain and rigidity. Rationale: A dusky appearance of the stoma indicates decreased blood supply; a healthy stoma should appear beefy red. Protrusion indicates prolapse of the stoma. Sharp abdominal pain and rigidity suggest peritonitis.

A client with metastatic cancer is experiencing neuropathic pain. Which alternative therapy is most beneficial in treating this type of pain?

Answer: Transcutaneous electrical nerve stimulation (TENS)

In assessing a postmastectomy client, the nurse determines that the client is in denial. The nurse can best respond by:

Answer: accepting the denial.

A client with a nagging cough makes an appointment to see the physician after reading that this symptom is one of the warning signs of cancer. What is another warning sign of cancer?

Answer: indigestion

The nurse in a long-term care facility notes a change in the color, shape, and texture of a nevus located on a client's shoulder. The nurse knows that this finding might suggest:

Answer: malignant melanoma.

On discharge, a client who underwent a left mastectomy expresses relief that "the cancer" has been treated. When discussing this issue with the client, the nurse should stress that she:

Answer: should continue to perform breast self-examination on her right breast.

A client is admitted to the hospital with severe bone marrow depression related to chemotherapy. What does the nurse include in the plan of care to decrease the risk of infection in this client? Select all that apply.

Answer: • Ensure the use of protective isolation protocol. • Wash hands before administering care.

The nurse is caring for a recently married, 29-year-old female client, who was diagnosed with acute lymphocytic leukemia (ALL). The client is preparing for an allogeneic bone marrow transplant. Which statement by the client demonstrates an understanding of the physician's description of the diagnosis and treatment?

Answer:"I'll only need chemotherapy therapy before receiving my bone marrow transplant."

A client undergoes a laryngectomy to treat laryngeal cancer. When teaching the client how to care for the neck stoma, the nurse should include which instruction?

Answer:"Keep the stoma moist."

A 65-year-old client is scheduled for a right lower lobectomy for lung cancer. During the admission assessment, the client asks for information about a living will and advance directive. The nurse knows that the client understands her teaching about the living will when he says:

Answer:"The advance directive allows me to state my health care wishes while I'm still able to do so."

The nurse is caring for a client who detected a lump in her right breast. The client asks the nurse " How do I find out if it's cancer?" Which statement by the nurse would be most appropriate?

Answer:"The physician will need to perform a fine needle aspiration and biopsy to confirm the diagnosis. Rationale: Fine needle aspiration and biopsy provide cells for histologic examination to confirm a diagnosis of cancer.

A client complains of sporadic epigastric pain, yellow skin, nausea, vomiting, weight loss, and fatigue. Suspecting gallbladder disease, the physician orders a diagnostic workup, which reveals gallbladder cancer. Which nursing diagnosis is appropriate for this client?

Anticipatory grieving

A client complains of sporadic epigastric pain, yellow skin, nausea, vomiting, weight loss, and fatigue. Suspecting gallbladder disease, the physician orders a diagnostic workup, which reveals gallbladder cancer. Which nursing diagnosis may be appropriate for this client?

Anticipatory grieving

A client complains of sporadic epigastric pain, yellow skin, nausea, vomiting, weight loss, and fatigue. Suspecting gallbladder disease, the physician orders a diagnostic workup, which reveals gallbladder cancer. Which nursing diagnosis is appropriate for this client?

Anticipatory grieving Rationale: Anticipatory grieving is an appropriate nursing diagnosis for this client because few clients with gallbladder cancer live more than 1 year after diagnosis

To combat the most common adverse effects of chemotherapy, the nurse would administer an:

Antiemetic.

Which medication classification is prescribed when allergy is a factor causing the skin disorder? a) Antihistamines b) Corticosteroids c) Local anesthetics d) Antibiotics

Antihistamines Explanation: Antihistamines are frequently prescribed when an allergy is a factor in causing the skin disorder. They relieve itching and shorten the duration of allergic reaction. Corticosteroids are used to relieve inflammatory or allergic symptoms. Antibiotics are used to treat infectious disorders. Local anesthetics are used to relieve minor skin pain and itching.

5-Fluorouracil (5FU) is classified as which type of antineoplastic agent?

Antimetabolite

Which of the following is a term used to describe the process of programmed cell death?

Apoptosis

Ms. Dowe was seen in the clinic for musculoskeletal pain, fatigue, mood disorders, and sleep disturbances. The physician has diagnosed fibromyalgia. Which of the following would not be a part of teaching plan for her condition?

Applications of ice

The nurse is caring for a client receiving the fentanyl transdermal system (Duragesic-50) for pain management. When applying a new system, the nurse should:

Apply the system immediately after removal from a package.

The nurse is caring for a client with burns on his legs. Which nursing intervention will help prevent leg contractures?

Applying knee splints

The nurse is preparing a client for a surgical procedure that will allow visualization of the extent of joint damage of the knee for a client with rheumatoid arthritis and also obtain a sample of synovial fluid. What procedure will the nurse prepare the client for?

Arthroscopy

The nurse is preparing a client for a surgical procedure that will allow visualization of the extent of joint damage of the knee for a client with rheumatoid arthritis and also obtain a sample of synovial fluid. What procedure will the nurse prepare the client for? a) Needle aspiration b) Arthroscopy c) Arthroplasty d) Open reduction

Arthroscopy Explanation: Arthroscopic examination may be carried out to visualize the extent of joint damage as well as to obtain a sample of synovial fluid. An open reduction would be used for the treatment of a fracture. Needle aspiration will not allow visualization of the joint damage but will allow obtaining the sample of synovial fluid. Arthroplasty is the restructure of the joint surface after diagnosis is made.

Articulation between the head of the femur and the acetabulum occurs in the

Articulation between the head of the femur and the acetabulum occurs in the hip joint.

subacromial bursae

Articulation of the head of the humerus in the glenoid cavity of the scapula. The acromioclavicular joint includes the clavicle and acromion process of the scapula. It contains the subacromial and subscapular bursae

Range of motion of the temporomandibular joint consists of three activities:

Asked the client to open and cloes the mouth • Asked the client to jut the jaw forward • Asked the client to rock the jaw laterally

To assess adduction of the shoulders and arms, a nurse should ask a client to

Asking the client to bring both hands in front of the body elicits adduction.

To assess flexion of the shoulders and arms, a nurse should ask a client to:

Asking the client to move the arms forward elicits flexion,

The nurse prepares to perform a neurovascular assessment on the client with closed multiple fractures of the right humerus. Which technique will the nurse use?

Assess sensation of the right upper extremity. Correct: Assessing sensation of the right upper extremity is part of a focused neurovascular assessment for the client with multiple fractures of the right humerus.

The client's left arm is placed in a plaster cast. Which assessment will the nurse perform before the client is discharged?

Assess that the cast is dry. Correct: The cast must be dry and free of cracking and crumbling.

After total hip replacement, a client is receiving epidural analgesia to relieve pain. Which of the following is a nursing priority for this client?

Assessing for sensation in the legs

A client has a Fiberglas cast on the right arm. Which action should the nurse include in the plan of care?

Assessing movement and sensation in the fingers of the right hand

The nurse is caring for a client placed in traction to treat a fractured femur. Which nursing intervention has the highest priority?

Assessing the extremity for neurovascular integrity

The ABCD method offers one way to assess skin lesions for possible skin cancer. What does the A stand for?

Asymmetry

degenerative joint disease indicators

Asymmetry, discomfort when touched, or crepitus during movement may occur with degenerative joint disease

the gait of a client with Parkinson's disease test

At the back of the client and nudge the sternum Explanation: To perform the nudge test, the nurse should stand at the back of the client and nudge his sternum. The nurse should put arms around the client to prevent a fall. Falling backward easily is seen with cervical spondylosis and Parkinson's disease.

For a client who has had a stroke, which nursing intervention can help prevent contractures in the lower legs?

Attaching braces or splints to each foot and leg

Which type of vaccine uses the patient's own cancer cells that are prepared for injection back into the patient?

Autologous

A client who has sustained a fracture reports an increase in pain and decreased function of the affected extremity. The nurse would suspect which of the following?

Avascular necrosis

There are a variety of problems that can become complications after a fracture. Which of the following is described as a condition that occurs from interruption of the blood supply to the fracture fragments after which the bone tissue dies, most commonly in the femoral head?

Avascular necrosis

The nurse is caring for a client who underwent a total hip replacement. What should the nurse and other caregivers do to prevent dislocation of the new prosthesis?

Avoid internal rotation of the affected leg.

The nurse is providing client teaching for a client undergoing chemotherapy. What dietary modifications should the nurse advise?

Avoid spicy and fatty foods.

The nurse is caring for a client experiencing bone cancer pain. When assessing the pain, which of the following components of the pain assessment is MOST important? A. Location of the pain B. Intensity of the pain C. Cause of the pain D. Duration of the pain E. Aggregating factors

B) Intensity of the pain

Breast self examination (BSE) is one of the ways to detect breast cancer earlier. The nurse is conducting a health teaching to female clients in a clinic. During evaluation the clients are asked to state what they learned. Which of the following statements made by a client requires further teaching about BSE? A. "BSE is done after menstruation." B. "BSE palpation is done by starting at the center and going to the periphery in a circular motion." C. "BSE can be done in either supine or standing position." D. "BSE should start from age 20"

B. "BSE palpation is done by starting at the center and going to the periphery in a circular motion."

When talking with the nurse, the brother of a client with leukemia says, "We used to play pretty rough games together. Maybe some of the bruises he got when I tackled him caused this." Which of the following would be the nurse's best response? A. "Don't feel guilty. You didn't cause your brother's illness." B. "I can see you're worried. Let's talk about how people get leukemia." C. "Here is some information about leukemia for you to read." D. "Lots of people worry about things like this. It isn't your fault."

B. "I can see you're worried. Let's talk about how people get leukemia."

When caring for a client with an internal radiation implant, the nurse should observe which principles? Select all that apply: A. Limiting the time with the client to 1 hour per shift B. Keeping pregnant woman out of the client's room. C. Placing the client in a private room with a private bath. D. Wearing a lead shield when performing direct patient care. E. Removing the dosimeter film badge when entering the client's room. F. Allowing individuals younger than 16 years old in the room as long as they are six feet from the client.

B. Keeping pregnant woman out of the client's room. C. Placing the client in a private room with a private bath. D. Wearing a lead shield when performing direct patient care.

The nurse teaches the client with chronic cancer pain about optimal pain control. Which of the following recommendations is most effective for pain control? A. Get used to some pain and use a little less medication than needed to keep from being addicted. B. Take prescribed analgesics on an around-the-clock schedule to prevent recurrent pain. C. Take analgesics only when pain returns D. Take enough analgesics around the clock so that you can sleep 12 to 16 hours a day to block the pain.

B. Take prescribed analgesics on an around-the-clock schedule to prevent recurrent pain.

Deep, aching back pain has prompted a 31-year-old woman to visit her primary care provider. The client claims that spinal movements do not exacerbate her pain and assessment reveals no deficits in range of motion. Which of the following etiologies would the clinician first suspect?

Back pain that is referred from the abdomen or pelvis.

Of the following, which is not a risk factor for osteoporosis? a) Being male b) Being postmenopausal c) Small-framed, thin White or Asian women d) Family history

Being male Explanation: Being male is not considered a risk factor. The following are some of the risk factors for osteoporosis: being a small-framed, thin White or Asian women; being postmenopausal; family history; inactivity; chronic low calcium intake; and excessive caffeine or tobacco use.

For a nursing exam, students must label a diagram using the correct medical terminology. Where would the students label the metacarpophalangeal joint?

Between the hand and the finger

A client is admitted to an acute care facility with esophageal cancer. The incidence of esophageal cancer is highest in:

Black males.

Which of the following does a nurse thoroughly evaluate before a hematopoietic stem cell transplant (HSCT) procedure?

Blood studies

A nurse is caring for a patient being treated with antineoplastic drugs. The patient is at risk for thrombocytopenia due to bone marrow suppression. What should the nurse monitor for in this patient?

Bloody urine

A client with cancer is receiving chemotherapy. The nurse should assess which of the following diagnostic values while the client is receiving chemotherapy?

Bone Marrow cells The fast-growing, normal cells most likely to be affected by certain cancer treatments are blood-forming cells in the bone marrow, as well as cells in the digestive tract, reproductive system, and hair follicles. Fortunately, most normal cells recover quickly when treatment is over. Bone marrow suppression (decreased ability of the bone marrow to manufacture blood cells) is a common side effect of chemotherapy. A low white blood cell count (neutropenia) increases the risk for infection during chemotherapy, but other blood cells made in the bone marrow can be affected as well. Most cancer agents do not affect tissues and organs, such as the heart, liver, and pancreas.

The nurse recognizes that the client with osteomyelitis is at risk for:

Bone abscess formation

A nurse is caring for an elderly female client with osteoporosis. When teaching the client, the nurse should include information about which major complication?

Bone fracture

The nurse is caring for an elderly female with osteoporosis. When teaching the client, the nurse should include information about which major complication of this condition?

Bone fracture

A nurse is caring for an elderly female client with osteoporosis. When teaching the client, the nurse should include information about which major complication? a) Negative calcium balance b) Loss of estrogen c) Dowager's hump d) Bone fracture

Bone fracture Explanation: Bone fracture is a major complication of osteoporosis; it results when loss of calcium and phosphate increases the fragility of bones. Estrogen deficiencies result from menopause — not osteoporosis. Calcium and vitamin D supplements may be used to support normal bone metabolism, but a negative calcium balance isn't a complication of osteoporosis. Dowager's hump results from bone fractures. It develops when repeated vertebral fractures increase spinal curvature.

bone marrow

Bones contain red marrow that produces blood cells and yellow marrow composed mostly of fat. The periosteum covers the bones and contains osteoblasts and blood vessels that promote nourishment and formation of new bone tissues. Composed of osseous tissue, bones can be divided into two types: compact bone, which is hard and dense and makes up the shaft and outer layers; and spongy bone, which contains numerous spaces and makes up the ends and centers of the bones.

Which of the following clinical manifestations would the nurse expect to find in a client who has Paget's disease?

Bowing of the legs

The nurse is preparing an educational program on breast cancer for women at a Black community center. What information is important for the nurse to consider for the discussion?

Breast cancer concerns vary between socioeconomic levels of Black women. The nurse needs to consider the beliefs and concerns for all socioeconomic levels of Black women when providing education on breast cancer. Access to screening and care may differ. Black women are more likely to develop breast cancer and be diagnosed later in the disease process than Caucasian women. Not all Black women believe that breast cancer is inevitable.

What food can the nurse suggest to the client at risk for osteoporosis? a) Broccoli b) Bananas c) Carrots d) Chicken

Broccoli Explanation: Calcium is important for the prevention of osteoporosis. Broccoli is high in calcium.

The client who is receiving chemotherapy is not eating well but otherwise feels healthy. Which meal suggestion would be best for this client?

Broiled chicken, green beans, and cottage cheese Carbohydrates are the first substance used by the body for energy. Proteins are needed to maintain muscle mass, repair tissue, and maintain osmotic pressure in the vascular system. Fats, in a small amount, are needed for energy production. Chicken, green beans, and cottage cheese are the best selection to provide a nutritionally well-balanced diet of carbohydrate, protein, and a small amount of fat. Cereal with milk and strawberries as well as toast, gelatin dessert, and cookies have a large amount of carbohydrates and not enough protein. Steak and french fries provide some carbohydrates and a good deal of protein; however, they also provide a large amount of fat.

The nurse admits an older adult client who sustained a left hip fracture and is in considerable pain. The nurse anticipates that the client will be placed in which type of traction?

Buck's traction Correct: Buck's traction may be applied before surgery to help decrease pain associated with muscle spasm.

A client is experiencing symptoms that are suspected to be related to systemic lupus erythematosus. What cutaneous symptom occurs in about 50% of clients affected by this disease?

Butterfly-shaped rash on the face over the bridge of the nose and cheeks

The client with cancer is receiving chemotherapy and develops thrombocytopenia. The nurse identifies which intervention as the highest priority in the nursing plan of care? A. Ambulation 3 times daily B. Monitoring temperature C. Monitoring the platelet count D. Monitoring for pathological fractures

C. Monitoring the platelet count

Which of the following inhibits bone resorption and promotes bone formation?

Calcitonin

Which medication directly inhibits osteoclasts thereby reducing bone loss and increasing BMD?

Calcitonin (Miacalcin)

Which medication directly inhibits osteoclasts thereby reducing bone loss and increasing BMD? a) Calcitonin (Miacalcin) b) Vitamin D c) Raloxifene (Evista) d) Teriparatide (Forteo)

Calcitonin (Miacalcin) Explanation: Miacalcin directly inhibits osteoclasts, thereby reducing bone loss and increased BMD. Evista reduces the risk of osteoporosis by preserving BMD without estrogenic effects on the uterus. Forteo has been recently approved by the FDA for the treatment of osteoporosis.

A client is diagnosed with osteoporosis. Which electrolytes are involved in the development of osteoporosis?

Calcium and phosphorous

A home care nurse assesses for disease complications in a client with bone cancer. Which laboratory value may indicate the presence of a disease complication?

Calcium level of 11.6 mg/dl

A home care nurse assesses for disease complications in a client with bone cancer. Which laboratory value may indicate the presence of a disease complication? a) Sodium level of 110 mEq/L b) Calcium level of 11.6 mg/dl c) Magnesium level of 0.9 mg/dl d) Potassium level of 6.3 mEq/L

Calcium level of 11.6 mg/dl Explanation: In clients with bone cancer, tumor destruction of bone commonly causes excessive calcium release. When the calcium-excreting capacity of the kidneys and GI tract is exceeded, the serum calcium level rises above normal, leading to hypercalcemia (a serum calcium level greater than 10.2 mg/dl). Hyperkalemia (a potassium level greater than 5 mEq/L) isn't caused by bone cancer and is seldom associated with chemotherapy. Hyponatremia (a sodium level less than 135 mEq/L) and hypomagnesemia (a magnesium level less than 1.3 mg/dl) are potential adverse effects of chemotherapy; these electrolyte disturbances don't result directly from bone cancer.

"A 45-year-old client receiving radiation therapy for thyroid cancer complains of mouth and throat pain. While inspecting the mouth and throat, the nurse notices white patches and ulcerations in the oral mucosa. What do these findings suggest?

Candidiasis

An important nursing assessment, post fracture, is to evaluate neurovascular status. Therefore, the nurse should check for:

Capillary refill.

When performing a physical assessment of a client, the client reports numbness, tingling, and pain when the nurse percusses lightly over the median nerve. The nurse recognizes that this finding is consistent with: a) Dupuytren's contracture b) Impingement syndrome c) Morton's neuroma d) Carpal tunnel syndrome

Carpal tunnel syndrome Explanation: Tinel's sign (numbness, tingling, and pain in response to light percussion over the median nerve) is a positive finding for carpal tunnel syndrome.

Cartilaginous joints

Cartilaginous joints (e.g., joints between vertebrae) are joined by cartilage.

Which group is at the greatest risk for osteoporosis? a) Men b) African American women c) Asian women d) Caucasian women

Caucasian women Explanation: Small-framed, nonobese Caucasian women are at greatest risk for osteoporosis. Asian women of slight build are at risk for low peak BMD. African American women, who have a greater bone mass that Caucasian women, are less susceptible to osteoporosis. Men have a greater peak bone mass and do not experience sudden estrogen reduction.

A client with acute osteomyelitis is to receive parenteral penicillin for 4 to 6 weeks. Before administering the first dose, the nurse asks the client about known drug allergies. An allergy to which antibiotic or antibiotic class necessitates cautious use of penicillin?

Cephalosporins

Cervical disc degenerative disease

Cervical disc degenerative disease is associated with impaired range of motion and pain that radiates to the back, shoulders, or arms.

Cervical spinal cord compression

Cervical spinal cord compression causes neck pain with loss of sensation in the legs

The nurse is teaching a group of women to perform breast self-examination. The nurse should explain that the purpose of performing the examination is to discover

Changes from previous self-examinations

A patient has been prescribed melphalan, an alkylating agent, for the treatment of an ovarian tumor. The patient wants to know the action of the drug in treating the tumor. Which of the following should the nurse inform the patient?

Changes the cell to a more alkaline environment.

A client is admitted to the emergency department after a motorcycle accident with a compound fracture of the left femur. Which action will be most essential for the nurse to take first?

Check the dorsalis pedis pulses. Correct: The first action should be to assess the circulatory status of the leg because the client is at risk for acute compartment syndrome (ACS), which can begin as early as 6 to 8 hours after an injury. Severe tissue damage can also occur if neurovascular status is compromised.

Which nursing action will the nurse on the orthopedic unit plan to delegate to unlicensed assistive personnel (UAP)?

Check the vital signs for a client who was admitted after a total knee replacement 3 hours ago. Correct: Vital sign assessment is a skill that is within the role of the UAP.

A cancer client makes the following statement to the nurse: "I guess I will tell my doctor to forego the chemotherapy. I do not want to be throwing up all the time. I would rather die."Which of the following facts supports the use of chemotherapy for this client?

Chemotherapy treatment can be adjusted to optimize effects while limiting adverse effects.

The nurse is caring for a client who was involved in an automobile accident and sustained multiple trauma. The client has a Volkmann's contracture to the right hand. What objective data does the nurse document related to this finding?

Clawlike deformity of the right hand without ability to extend fingers

A nurse is caring for a client with a long-term central venous catheter. Which steps should the nurse include in teaching how to care for his catheter at home?

Clean the port with an alcohol pad before administering I.V. fluid through the catheter.

A nurse is caring for a client with a long-term central venous catheter. Which steps should the nurse include in teaching how to care for his catheter at home?

Clean the port with an alcohol pad before administering I.V. fluid through the catheter. Clients should be instructed to clean the port with an alcohol pad before administering I.V. fluid through the catheter to prevent microorganisms from entering the bloodstream. Using clean technique when accessing the port with a needle, cleaning the needle with a povidone-iodine solution, or flushing each port using the same syringe would break sterile technique.

A nurse is caring for a client who had hip pinning surgery 6 hours ago to treat intertrochanteric fracture of the right hip. What assessment finding requires further investigation by the nurse?

Client is anxious and confused Explanation: The client is anxious and confused is the appropriate answer. Postoperative complications of hip fractures include hemorrhage, pulmonary emboli, and fat emboli. Anxiety and confusion may be indicative of hypoxia as a result of any of above these complications and needs further investigation. Capillary refill of 2-3 seconds is an expected finding, edema is present from both the injury and the surgical intervention. 100 milliliters of bright red drainage 6 hours after surgery should be watched, but is not of immediate concern.

"For a client with newly diagnosed cancer, the nurse assists in formulating a nursing diagnosis of Anxiety related to the threat of death secondary to cancer diagnosis. Which expected outcome would be appropriate for this client?

Client verbalizes feelings of anxiety.

For a client with newly diagnosed cancer, the nurse assists in formulating a nursing diagnosis of Anxiety related to the threat of death secondary to cancer diagnosis. Which expected outcome would be appropriate for this client?

Client verbalizes feelings of anxiety.

The nurse is conducting an initial nursing history of a client who is experiencing pain related to bone cancer. The most important information to gather in this initial assessment is the:

Client's self-reporting of the pain experience. The most important component of pain assessment is the client's self-report of the pain. The nurse should have the client describe the quality, location, and intensity of the pain; the client's response to the pain; and any alleviating or aggravating factors affecting the pain. The physical assessment should follow the pain assessment and should be delayed if the client is uncomfortable. The family's response to the client's illness casts light on the amount of support the client has and alerts the nurse to potential problems. With care, however, these concerns are secondary to the issue of pain control.

The nurse is working at the local family planning clinic doing family education. When devising a teaching plan, in which client group would the nurse stress the importance of an annual Papanicolaou test?

Clients infected with the human papillomavirus (HPV). Annual Papanicolaou testing is a screening to detect potential precancerous and cancerous cells in the endocervical canal of the female reproductive system. HPV causes genital warts, which are associated with an increased incidence of cervical cancer. Recurrent candidiasis, pregnancy before age 20, and use of oral contraceptives do not increase the risk of cervical cancer.

A client diagnosed with acute myelocytic leukemia (AML) has been receiving chemotherapy. During the last two cycles of chemotherapy, the client developed severe thrombocytopenia requiring multiple platelet transfusions. The client is now scheduled to receive a third cycle. How can the nurse best detect early signs and symptoms of thrombocytopenia?

Closely observe the client's skin for petechiae and bruising.

A client diagnosed with acute myelocytic leukemia has been receiving chemotherapy. During the last 2 cycles of chemotherapy, the client developed severe thrombocytopenia requiring multiple platelet transfusions. The client is now scheduled to receive a third cycle. How can the nurse best detect early signs and symptoms of thrombocytopenia?

Closely observe the client's skin for petechiae and bruising. Explanation: The nurse should closely observe the client's skin for petechiae and bruising. Daily laboratory testing may not reflect the client's condition as quickly as subtle changes in the client's skin. Performing a cardiovascular assessment every 4 hours and checking the clients history for a congenital link to thrombocytopenia don't help detect early signs and symptoms of thrombocytopenia.

A client reports to the emergency department after experiencing pain in the left arm. The client reports that he extended his arms in an attempt to prevent a fall. Which fracture type does the nurse anticipate?

Colles' fracture Explanation: A Colles' fracture occurs in the distal radius. Falling with outstretched arms and hands may increase the risk of this type of fracture. A spiral fracture results from a twisting movement. A greenstick fracture is a bent and incomplete fracture commonly seen in children. A compound fracture results in the bone extending through the skin.

"A client suspected of having colorectal cancer will require which diagnostic study to confirm the diagnosis?

Colonoscopy

The nurse is conducting a community education program using the American Cancer Society's colorectal screening and prevention guidelines. The nurse determines that the participants understand the teaching when they identify that people over the age of 50 should have which of the following screening tests every 10 years?

Colonoscopy

Which assessment findings would the nurse expect in the client with osteomalacia? a) Column A b) Column B c) Column C d) Column D

Column B Explanation: Osteomalacia is characterized by decreased serum calcium and phosphorus and elevated alkaline phosphatase levels

Which type of fracture produces several bone fragments?

Comminuted

A 35-year-old client is visiting a rheumatology group practice. The client has experienced increasing pain and progressing inflammation of the hands and feet. The rheumatologist has prescribed NSAID use to treat the condition. What client education is most important for the nurse to address with the use of these medications?

Common adverse effects

A 35-year-old client is visiting a rheumatology group practice. The client has experienced increasing pain and progressing inflammation of the hands and feet. The rheumatologist has prescribed NSAID use to treat the condition. What client education is most important for the nurse to address with the use of these medications? a) Activity restrictions b) Common adverse effects c) Dietary restrictions d) Loading-dose schedule

Common adverse effects Explanation: The most common adverse effects of NSAIDs are related to the GI tract: nausea, vomiting, diarrhea, and constipation. GI bleeding, which in some cases is severe, has been reported with the use of these drugs. Use of NSAIDs does not pose significant dietary or activity restrictions nor is there a loading-dose schedule.

Two days after application of a cast to treat a fractured femur, the patient tells the orthopedic heath care provider that he is experiencing severe, deep, and constant pain in his leg. The nurse suspects a diagnosis of:

Compartment syndrome.

Which assessment findings would cause the nurse to suspect compartment syndrome after casting of the leg?

Complaints of numbness and tingling in toes of affected leg

A nurse on the orthopedic floor is caring for a group of clients who are in various stages of recovery after knee replacement surgery. One client is ready for discharge. How should the nurse proceed with discharge planning?

Complete the discharge instructions for the client who is being discharged, and allow time for him to ask questions.

A patient who has extremity right wrist fracture complains of severe burning pain, frequent changes in the skin from hot and dry to cold and feeling clammy shiny skin that is growing more hair in the injured extremity. The nurse should anticipate providing care for what complication?

Complex regional pain syndrome (CRPS)

A client is admitted to the emergency room after being hit by a car while riding a bicycle. The client sustained a fracture of the left femur, and the bone is protruding through the skin. What type of fracture does the nurse recognize requires emergency intervention?

Compound

An older adult female has a bone density test that reveals severe osteoporosis. What does the nurse understand can be a problem for this client due to the decrease in bone mass and density? a) Diabetes b) Cardiac disease c) Hypertension d) Compression fractures

Compression fractures Explanation: In osteoporosis, loss of bone substance exceeds bone formation. The total bone mass and density is reduced, resulting in bones that become progressively porous, brittle, and fragile. Compression fractures of the vertebrae are common. Diabetes, hypertension, and cardiac disease may occur in response to the aging process but are not the result of osteoporosis.

Compression fractures of the neck

Compression fractures of the neck may also cause loss of sensation in the legs if the spinal cord becomes compressed. (less)

To help prevent osteoporosis, what should a nurse advise a young woman to do? a) Consume at least 1,000 mg of calcium daily. b) Keep the serum uric acid level within the normal range. c) Encourage the use of a firm mattress. d) Avoid trauma to the affected bone.

Consume at least 1,000 mg of calcium daily. Explanation: To help prevent osteoporosis, the nurse should encourage the client to consume at least the recommended daily allowance (RDA) of calcium. Before menopause, the RDA is 1,000 mg; after menopause, it's 1,500 mg. Because osteoporosis affects all bones, avoiding trauma to the affected bone only is inappropriate. Using a firm mattress and keeping the uric acid level within the normal range don't relate to osteoporosis. The nurse should encourage a client with ankylosing spondylitis to sleep on a firm mattress. The nurse should advise a client with gouty arthritis to keep the serum uric acid level in the normal range.

Which primary cancer treatment goal is prolonged survival and containment of cancer cell growth?

Control

A 13-year-old client is brought to the emergency department. The client's mother reports that the client was struck with a baseball bat on his upper arm while diving for a pitched ball. After diagnostic tests are completed, the physician reassures the mother that her son's humerus is not broken but he has suffered another type of injury. What type of injury would you expect the physician to diagnose?

Contusion

A client comes to the Emergency Department complaining of localized pain and swelling of his lower leg. Ecchymotic areas are noted. History reveals that the client got hit in the leg with a baseball bat. Which of the following would the nurse suspect as most likely?

Contusion

The patient presents to the emergency room with an open fracture of the femur. Which action would the nurse implement to prevent the most serious complication of an open fracture?

Cover the wound with a sterile dressing to prevent infection.

CN III

Cranial nerve III is involved with extraocular eye movements

CN V

Cranial nerve V is involved with facial sensation

CN VII

Cranial nerve VII is associated with facial muscles

General nursing measures employed when caring for the client with fracture include all of the following, except?

Cranial nerve assessment

Which typical clinical manifestation does the nurse expect to observe for a client with a right tibial fracture?

Crepitation of extremity Correct: On assessment, crepitation (a continuous grating sound created by bone fragments) may be heard when the affected extremity is moved.

Which of the following is a type of procedure that uses liquid nitrogen to freeze tissue that causes cell destruction?

Cryoablation

A client is undergoing a diagnostic workup for suspected testicular cancer. When obtaining the client's history, the nurse checks for known risk factors for this type of cancer. Testicular cancer has been linked to?

Cryptorchidism (failure of one or both testes to descend into the scrotum)

A 68-year old client with prostate cancer who is being treated with chemotherapy has developed neutropenia. He has had a poor appetite and his wife wants to bring him food from outside of the hospital to encourage him to eat. Which of the following foods would NOT be appropriate for the client? A. French fries B. A vanilla milkshake C. A grilled cheese sandwich D. Fresh strawberries E. Cereal

D) Fresh strawberries

Mina, who is suspected of an ovarian tumor is scheduled for a pelvic ultrasound. The nurse provides which preprocedure instruction to the client? A. Eat a light breakfast only B. Maintain an NPO status before the procedure C. Wear comfortable clothing and shoes for the procedure D. Drink six to eight glasses of water without voiding before the test

D. Drink six to eight glasses of water without voiding before the test

A 12 year old boy seen in the clinic, and a diagnosis of Hodgkin's disease is suspected. Which diagnostic test results confirm the diagnosis of Hodgkin's disease? A . Elevated vanillylmandelic acid urinary level. B. The presence of blast cells in the bone marrow C. The presence of Epstein-Barr virus in the blood. D. The presence of Reed-Sternberg cells in the lymph nodes

D. The presence of Reed-Sternberg cells in the lymph nodes

How can breast cancer prevention programs best serve women who are at risk and come from lower socioeconomic backgrounds?

Develop screening and educational programs.

A client receiving chemotherapy for cervical cancer indicates that she has an advance directive. She tells the nurse that she worries her children will not honor her wishes if her condition should worsen. In order to facilitate the honoring of the client's wishes, what should the nurse encourage the client to do?

Discuss her end-of-life wishes with her family.

The client diagnosed with osteosarcoma is scheduled for a surgical amputation. Which nursing diagnosis would be a priority for this client compared with other surgical clients?

Disturbed body image

The client diagnosed with osteosarcoma is scheduled for a surgical amputation. Which nursing diagnosis would be a priority for this client compared with other surgical clients? a) Impaired physical mobility b) Risk for infection c) Inadequate nutrition d) Disturbed body image

Disturbed body image Explanation: Amputation of a body part can result in disturbances in body image.

A client with gangrene of the left foot is scheduled for below-knee amputation. When planning preoperative care, the nurse should assign highest priority to which nursing diagnosis?

Disturbed body image related to loss of body part

The nurses instructs the client not to cross their legs and to have someone assist with tying their shoes. Which additional instruction should the nurse provide to client?

Do not flex the hip more than 90 degrees.

"A client seeks care for hoarseness that has lasted for 1 month. To elicit the most appropriate information about this problem, the nurse should ask which question?

Do you smoke cigarettes, cigars, or a pipe?

A client seeks medical attention for a ganglion. Which statement about this musculoskeletal disorder is true?

Dorsiflexion exacerbates signs and symptoms of a ganglion.

A client seeks medical attention for a ganglion. Which statement about this musculoskeletal mass is true?

Dorsiflexion exacerbates signs and symptoms of a ganglion.

A client is receiving chemotherapy to treat breast cancer. Which data collection finding indicates a fluid and electrolyte imbalance induced by chemotherapy?

Dry oral mucous membranes and cracked lips

Which is a flexion deformity caused by a slowly progressive contracture of the palmar fascia?

Dupuytren's contracture

Which of the following is a flexion deformity caused by a slowly progressive contracture of the palmar fascia?

Dupuytren's contracture

Which of the following diagnostics are used to evaluate spinal nerve root disorders (radiculopathies)? a) Electromyogram b) Magnetic resonance imaging c) Bone scan d) Computed tomography

Electromyogram Explanation: An electromyogram and nerve conduction studies are used to evaluate spinal nerve toot disorders (radiculopathies) for patients with low back pain. A bone scan may disclose information about infections, tumors, and bone marrow abnormalities. A computed tomography scan is useful in identifying underlying problems, such as obscure soft tissue lesions adjacent to the vertebral column and problems of vertebral disks. Magnetic resonance imaging permits visualization of the nature and location of spinal pathology.

What would be the most important nursing intervention in caring for the client's residual limb during the first 24 hours after amputation of the left leg?

Elevate the residual limb on a pillow. Explanation: Elevating the residual limb on a pillow for the first 24 hours after surgery helps prevent edema and promotes comfort by increasing venous return. Elevating the residual limb for longer than the first 24 hours is contraindicated because of the potential for developing a hip flexion contracture. Keeping the limb flat will be an important intervention after the first 24 hours. Preventing excessive swelling, however, is a priority in the first 24 hours. Adducting the residual limb on a scheduled basis prevents abduction contracture. Traction may be used to prevent or treat a hip flexion contracture—however, not in the first 24 hours.

The nurse is reviewing the diagnostic test findings of a client with rheumatoid arthritis. Which of the following would the nurse expect to find?

Elevated erythrocyte sedimentation rate

A client presents to the emergency department gently holding the left arm, which is slightly swollen and painful to the touch. Based on these findings, the nurse:

Elevates the arm and applies an ice pack

After a mastectomy for breast cancer, the nurse teaches the client how to avoid the development of lymphedema. Which of the following instructions should be included?

Elevating the affected arm on a pillow.

A client with arterial insufficiency undergoes below-knee amputation of the right leg. Which action should the nurse include in the postoperative care plan?

Elevating the stump for the first 24 hours

A client with arterial insufficiency undergoes below-knee amputation of the right leg. Which action should the nurse include in the postoperative plan of care?

Elevating the stump for the first 24 hours

A client with arterial insufficiency undergoes below-knee amputation of the right leg. Which action should the nurse include in the postoperative care plan?

Elevating the stump for the first 24 hours Explanation: Stump elevation for the first 24 hours after surgery helps reduce edema and pain by increasing venous return and decreasing venous pooling at the distal portion of the extremity. Bed rest isn't indicated and could predispose the client to complications of immobility. Heat application would be inappropriate because it promotes vasodilation, which may cause hemorrhage and increase pain. The initial pressure dressing usually remains in place for 48 to 72 hours after surgery.

After sustaining injuries in a motor vehicle accident, a client spends 10 days recovering in the intensive care unit. His condition stabilizes and he's transferred to the orthopedic unit. Upon arrival at the unit, his vital signs are stable, his temperature is 100° F (37.8° C), and he has an indwelling urinary catheter in place. He is currently on bed rest and able to consume a regular diet. Which independent nursing action should the nurse include in this client's plan of care?

Encourage the client to increase his intake of fluids.

When the client who has had a modified radical mastectomy returns from the operating room to the recovery room, what should the nurse do first?

Ensure that the client's airway is free of obstruction. Explanation: The highest priority when a nurse receives a client from the operating room is to assess airway patency. If the airway is not clear, immediate steps should be taken so that the client is able to breathe. Vital signs can be assessed after airway patency is assured. Assessing the patency and functioning of drainage tubes can be done after the airway is assessed and vital signs are taken. The dressing can be assessed once airway patency has been determined.

Which of the following observations should the nurse make first when the client who has had a modified radical mastectomy returns from the operating room to the recovery room?

Ensuring that the client's airway is free of obstruction.

After a lobectomy for lung cancer, the nurse instructs the client to perform deep-breathing exercises to:

Expand the alveoli and increase lung surface available for ventilation. Deep breathing helps prevent microatelectasis and pneumonitis and also helps force air and fluid out of the pleural space into the chest tubes. It does not decrease blood flow to the lungs or control the rate of air flow. The diaphragm is the major muscle of respiration; deep breathing causes it to descend, thereby increasing the ventilating surface.

During a visit to a client who underwent total hip replacement surgery, the home health care nurse notes small area rugs located throughout the house. How should the nurse intervene to ensure the client's safety?

Explain to the client that small area rugs pose a danger to clients after hip replacement surgery.

The nurse recognizes that proper positioning of an amputated lower extremity for the prevention of contractures is:

Extension

A nurse is administering daunorubicin (Daunoxome) through a peripheral I.V. line when the client complains of burning at the insertion site. The nurse notes no blood return from the catheter and redness at the I.V. site. The client is most likely experiencing which complication?

Extravasation

The client is receiving an infusion of cytarabine (Cytosar-U) through a peripheral I.V. catheter when he complains of burning at the insertion site. The nurse notes no blood return from the catheter, but she sees redness at the I.V. site. The client is most likely experiencing which complication?

Extravasation

A patient sustained an open fracture of the femur 24 hours ago. While assessing the patient, the nurse observes the patient is having difficulty breathing, and oxygen saturation decreases to 88% from a previous 99%. What does the nurse understand is likely occurring with this patient?

Fat emboli

The client with a fractured left humerus reports dyspnea and chest pain. Pulse oximetry is 88%. Temperature is 100.2 degrees Fahrenheit (38.5 degrees Centigrade); heart rate is 110 beats per minute; respiratory rate is 32 breaths per minute. The nurse suspects the client is experiencing:

Fat embolism syndrome

Fibrous joints .

Fibrous joints (e.g., sutures between skull bones) are joined by fibrous connective tissue and are immovable.

The nurse is teaching a client who suspects that she has a lump in her breast. The nurse instructs the client that a diagnosis of breast cancer is confirmed by:

Fine needle aspiration and biopsy.

What range of motion is the nurse testing by asking a client to stoop to pick an object off the floor?

Flexion

An 85-year-old retired housewife comes with her daughter to establish care. Her daughter is concerned because the client has experienced frequent falls in recent months. As part of the physical examination, the nurse asks the client to walk across the examination room. Which of the following is not part of the stance phase of gait?

Foot arched

Dupuytren's contracture causes flexion of which area(s)? a) Fourth and fifth fingers b) Ring finger c) Index and middle fingers d) Thumb

Fourth and fifth fingers Explanation: Dupuytren's contracture causes flexion of the fourth and fifth fingers, and frequently the middle finger.

Which of the following terms refers to a break in the continuity of a bone?

Fracture

A client is treated in the emergency department for a Colles' fracture sustained during a fall. What is a Colles' fracture?

Fracture of the distal radius

An adult client has been diagnosed with carpal tunnel syndrome. What type of working conditions may have contributed to this diagnosis?

Frequent Repetitive Movements

A staff nurse on the oncology unit must teach a new unit assistant about infection control practices on the unit. The nurse should explain that the most important measure to prevent the spread of infection is what?

Frequent hand washing

The nurse is teaching a client with osteomalacia how to take prescribed vitamin D supplements. The nurse stresses the importance of taking only the prescribed amount because high doses of vitamin D can be toxic. Early signs and symptoms of vitamin D toxicity include:

GI upset and metallic taste.

A client admitted to the hospital for internal fixation of a fractured left femur is attending physical therapy to learn how to use crutches. After the session, a nurse observes improper crutch use by the client. How should the nurse intervene?

Gently remind the client of the proper technique.

A client is admitted with severe pain in the knees. Which form of arthritis is characterized by urate deposits and joint pain, usually in the feet and legs, and occurs primarily in men older than age 30?

Gouty arthritis

An X-ray of the left femur shows a fracture that extends through the midshaft of the bone and multiple splintering fragments. What is this type of fracture called?

Greenstick Fracture

The nurse notes that the patient's left great toe deviates laterally. This finding would be recognized as which of the following?

Hallux valgus

Which of the following was formerly called a bunion?

Hallux valgus

Which of the following was formerly called a bunion? a) Ganglion b) Plantar fasciitis c) Morton's neuroma d) Hallux valgus

Hallux valgus Explanation: Hallux valgus (commonly called a bunion) is a deformity in which the great toe deviates laterally. Plantar fasciitis, an inflammation of the foot-supporting fascia, present as an acute onset of heel pain experienced with the first steps in the morning. Morton's neuroma is a swelling of the third (lateral) branch of the median plantar nerve. A ganglion, a collection of gelatinous material near the tendon sheaths and joints, appears as a round, firm compressible cystic swelling, usually on the dorsum of the wrist

Which is a deformity in which the great toe deviates laterally? a) Plantar fasciitis b) Hammertoe c) Pes cavus d) Hallux valgus

Hallux valgus Explanation: Hallux valgus is a deformity in which the great toe deviates laterally. A hammertoe is a flexion deformity of the interphalangeal joint, which may involve several toes. Pes cavus refers to a foot with an abnormally high arch and a fixed equines deformity of the forefoot. Plantar fasciitis is an inflammation of the foot-supporting fascia.

The nurse notes that the patient's left great toe deviates laterally. This finding would be recognized as which of the following? a) Pes cavus b) Hammertoe c) Flatfoot d) Hallux valgus

Hallux valgus Explanation: Hallux valgus is commonly referred to as a bunion. Hammertoes are usually pulled upward. Pes cavus refers to a foot with an abnormally high arch and a fixed equinus deformity of the forefoot. In flatfoot, the patient demonstrates a diminished longitudinal arch of the foot.

The nurse notes that the patient's left great toe deviates laterally. This finding would be recognized as which of the following?

Hallux valgus (bunion)

Ms. Simpson has come to the clinic with foot pain. The physician has described her problem as a flexion deformity of the proximal interphalangeal joint. What is the name of this disorder?

Hammer toe

During a routine physical examination of a client, the nurse observes a flexion deformity of the promixal interphalangeal (PIP) joint of two toes on the right foot. The nurse documents this finding as which of the following? a) Bunion b) Mallet toe c) Hallux valgus d) Hammer toe

Hammer toe Explanation: Hammer toe is a flexion deformity of the proximal interphalangeal (PIP) joint and may involve several toes. Mallet toe is a flexion deformity of the distal interphalangeal joint (DIP), and also can affect several toes. Hallux valgus, also called a bunion, is a deformity of the great (large) toe at its metatarsophalangeal joint.

Ms. Simpson has come to the clinic with foot pain. The physician has described her problem as a flexion deformity of the proximal interphalangeal joint. What is the name of this disorder? a) Mallet toe b) Hammer toe c) Heberden's nodes d) Hallux valgus (bunion)

Hammer toe Explanation: Hammer toe is a flexion deformity of the proximal interphalangeal joint. Mallet toe is a flexion deformity of the distal interphalangeal joint. Bunion is a deformity of the great toe at its metatarsophalangeal joint. Heberden's nodes are bony enlargements of the distal interphalangeal joints. This is a finding in degenerative joint disease.

The infection control team has identified a 25% infection rate on the orthopedic floor. The nursing staff members are asked to record their care activities by recording them in a log to help identify the cause of the high infection rate. Which of the following care activities should be recorded in the activity log?

Hand washing between client contacts

What should a male client older than age 50 do to help ensure early identification of prostate cancer?

Have a digital rectal examination and prostate-specific antigen (PSA) test done yearly.

What should a male client older than age 50 do to help ensure early identification of prostate cancer?

Have a digital rectal examination and prostate-specific antigen (PSA) test done yearly. Explanation: The incidence of prostate cancer increases after age 50. The digital rectal examination, which identifies enlargement or irregularity of the prostate, and PSA test, a tumor marker for prostate cancer, are effective diagnostic measures that should be done yearly. Testicular self-examinations won't identify changes in the prostate gland because of its location in the body. A transrectal ultrasound, CBC, and BUN and creatinine levels are usually done after diagnosis to identify the extent of the disease and potential metastasis

What should a male client older than age 50 do to help ensure early identification of prostate cancer?

Have a digital rectal examination and prostate-specific antigen (PSA) test done yearly. Correct Explanation: The incidence of prostate cancer increases after age 50. The digital rectal examination, which identifies enlargement or irregularity of the prostate, and PSA test, a tumor marker for prostate cancer, are effective diagnostic measures that should be done yearly. Testicular self-examinations won't identify changes in the prostate gland because of its location in the body. A transrectal ultrasound, CBC, and BUN and creatinine levels are usually done after diagnosis to identify the extent of the disease and potential metastasis.

The nurse is speaking to a group of women about early detection of breast cancer. Which screening does the nurse recommend to women age 50 and older?

Have a mammogram annually The Canadian Cancer Society states at 50 years that women should have a mammogram annually and a clinical examination at least annually (not every 2 years). The American Cancer Society recommends mammography yearly beginning at age 40. All women should perform breast self-examination monthly (not annually). The hormonal receptor assay is done on a known breast tumor to determine whether the tumor is estrogen or progesterone dependent. An annual breast exam by a healthcare provider should be performed.

The nurse is speaking to a group of women about early detection of breast cancer. The average age of the women in the group is 47. Following the American Cancer Society guidelines, the nurse should recommend that the women:

Have a mammogram annually.

A patient has been prescribed alendronate (Fosamax) for the prevention of osteoporosis. Which of the following is the highest priority nursing intervention associated with the administration of the medication? a) Have patient sit upright for 60 minutes following administration b) Encourage patient to get yearly dental exams c) Assess for the use of corticosteroids d) Ensure adequate intake of vitamin D in the diet

Have patient sit upright for 60 minutes following administration Explanation: While all interventions are appropriate, the highest priority is having the patient sit upright for 60 minutes following the administration of the medication. This will prevent irritation and potential ulceration of the esophagus. The patient should have adequate intake of vitamin D and obtain yearly dental exams. The concurrent use of corticosteroids and Fosamax is link to a complication of osteonecrosis.

A physician prescribes raloxifene (Evista) to a hospitalized patient. The patient's history includes a right hip fracture, hysterectomy, deep vein thrombosis, and hypertension. Which of the following actions by the nurse demonstrates safe nursing care? a) Administering the raloxifene (Evista) with food or milk b) Having the patient sit upright for 30-60 minutes following administration c) Holding the raloxifene (Evista) and notifying the physician d) Administering the raloxifene (Evista) in the evening

Holding the raloxifene (Evista) and notifying the physician Explanation: Raloxifene (Evista) is contraindicated in patients with a history of deep vein thrombosis. The nurse should hold the medication and notify the physician. Raloxifene (Evista) can be given without regard to food or time of day. Raloxifene (Evista) is a selective estrogen receptor modulation (SERM) medication. Sitting upright for 30-60 minutes is for the classification of bisphosphonates.

A home care nurse assesses for disease complications in a client with bone cancer. The nurse knows that bone cancer may cause which electrolyte disturbance?

Hypercalcemia

A group of students are reviewing information about osteoporosis in preparation for a class discussion. The students demonstrate a need for additional review when they state which of the following as a risk factor?

Hypothyroidism

A group of students are reviewing information about osteoporosis in preparation for a class discussion. The students demonstrate a need for additional review when they state which of the following as a risk factor? a) Hypothyroidism b) Excess caffeine intake c) Prolonged corticosteroid use d) Prolonged immobility

Hypothyroidism Explanation: Factors associated with an increased risk for osteoporosis include: family history of osteoporosis, chronic low calcium intake, excessive intake of caffeine, tobacco use, Cushing's syndrome, prolonged use of high doses of corticosteroids, prolonged periods of immobility, hyperthyroidism, hyperparathyroidism, eating disorders, malabsorption syndromes, breast cancer (especially if treated with chemotherapy that suppresses estrogen, excluding Tamoxifen, which may reduce the risk of fractures), renal or liver failure, alcoholism, lactose intolerance, and dietary deficiency of vitamin D and calcium.

A client becomes neutropenic 11 days after his last chemotherapy cycle. It's obvious that the client understands his condition when he states:

I'll monitor my temperature frequently and go to the nearest emergency department if my temperature rises above 100.4° F."

The nurse is caring for a recently married, 29-year-old female client, who was diagnosed with acute lymphocytic leukemia (ALL). The client is preparing for an allogeneic bone marrow transplant. Which statement by the client demonstrates an understanding of the physician's description of the diagnosis and treatment?

I'll only need chemotherapy therapy before receiving my bone marrow transplant.

"A client receiving chemotherapy has a nursing diagnosis of Deficient diversional activity related to decreased energy. Which statement by the client indicates an understanding of appropriate ways to deal with this deficit?

I'll play card games with my friends.

Which statement by a client undergoing external radiation therapy indicates the need for further teaching?

I'm afraid I'll expose my family members to radiation.

"A client recently diagnosed with metastatic liver cancer is admitted for hospice care. An acquaintance of the client calls the nurse to ask how the client is doing. Which response by the nurse is most appropriate?

I'm not permitted to give you any information about his condition for confidentiality reasons; I'll tell him you were asking about him.

The client with leukemia received induction chemotherapy 2 days ago and is now reporting severe diarrhea, decreased urination, cardiac dysrhythmias, and parasthesias with tetany. Laboratory reports reveal hyperkalemia, hyperuricemia, and hypocalcemia. Which of the following actions would the nurse anticipate?

IV fluids to increase urine output and allopurinol to inhibit uric acid. Explanation: The client likely has tumor lysis syndrome which occurs when a person with cancer (such as leukemia) initiate treatment, causing the rapid destruction and breakdown of large numbers of cells. The syndrome results in the signs and symptoms noted above as well as hyperuricemia, hyperkalemia, hyperphosphatemia, renal failure, and hypocalcemia. Early recognition is important to prevent renal damage. Increased IV fluids will flush the cellular debris from the system while increasing urine volume and restoring alkalinity, and allopurinol to decrease production of uric acid. Fluids should not contain potassium.

A client is newly diagnosed with cancer and is beginning a treatment plan. Which action by the nurse will be most effective in helping the client cope?

Identify available resources for the client and family.

Ms. Wilson is being discharged from the hospital after being diagnosed with and treated for systemic lupus erythematosus (SLE). You are teaching her and her family information about managing her disease. All of the following would be included, except?

If you have problems with a medication, you may stop it until your next physician visit.

The nurse is instructing a premenopausal woman about breast self-examination. The nurse should tell the client to do her self-examination:

Immediately after her menses

A nurse notices a client lying on the floor at the bottom of the stairs. He's alert and oriented and states that he fell down several stairs. He denies pain other than in his arm, which is swollen and appears deformed. After calling for help, what should the nurse do?

Immobilize the client's arm.

Which of the following is the most appropriate nursing diagnosis for a client with a strained ankle?

Impaired physical mobility

The nurse is assessing a client's ability to shrug her shoulders against resistance. The nurse is assessing which cranial nerve?

Inability to shrug shoulders against resistance suggests a lesion of cranial nerve XI (spinal accessory nerve

Which of the following aspects should a nurse include in the teaching plan for a patient with osteomalacia? a) Include the supplements of calcium, phosphorus, and vitamin D b) Avoid any activity or exercise c) Avoid dairy products d) Avoid green leafy vegetables

Include the supplements of calcium, phosphorus, and vitamin D Explanation: The nurse should encourage the patients with osteomalacia to include the supplements of calcium, phosphorus, and vitamin D; adequate nutrition; exposure to sunlight; and progressive exercise and ambulation. Patients need not avoid dairy products, leafy vegetables or mild exercise.

Which type of fracture involves a break through only part of the cross-section of the bone?

Incomplete

A client is experiencing chronic arthritis and active range of motion is ordered. The nurse understands that the goal is to do which of the following?

Increase muscle strength. Explanation: Active range of motion is to enhance client strength and abilities. The other choices are reflective of passive range of motion and reduction of inflammation, and maintaining circulation is not applicable to this question

A patient undergoing antineoplastic drug therapy is administered an alkylating drug. The patient is also administered an aminoglycoside as an anti-infective agent. Which of the following interactions of the alkylating drug with the aminoglycoside should the nurse assess the patient for?

Increased risk of nephrotoxicity

A client with a nagging cough makes an appointment to see the physician after reading that this symptom is one of the warning signs of cancer. What is another warning sign of cancer?

Indigestion

The nurse should consider which of the following principles when developing a plan of care to manage a client's pain from cancer?

Individualize the pain medication regimen for the client. The nurse should work with the client to individualize the plan of care for managing pain. Cancer pain is best managed with a combination of medications, and each client needs to be worked with individually to find the treatment regimen that works best. Cancer pain is commonly undertreated because of fear of addiction. The client who is in pain needs the appropriate level of analgesic and needs to be reassured that he will not become addicted. Cancer pain is best treated with regularly scheduled doses of medication. Administering the medication only when the client asks for it will not lead to adequate pain control. As drug tolerance develops, the dosage of the medication can be increased.

The nurse is caring for a client with bronchogenic carcinoma. Which nursing diagnosis takes highest priority?

Ineffective airway clearance related to obstruction by a tumor or secretions

Which of the following is a priority nursing diagnosis for the client with an amputated extremity?

Ineffective tissue perfusion: peripheral related to injury and amputation

A patient suffered an open fracture to the left femur during a horse-riding accident. For which of the following complications is this patient at highest risk?

Infection

The nurse is caring for a patient who sustained an open fracture of the right femur in an automobile accident. What does the nurse understand is the most serious complication of an open fracture?

Infection

A client with metastatic brain cancer is admitted to the oncology floor. According to the Patient Self-Determination Act of 1991 (PSDA), what is the hospital required to do concerning the execution of advance directives?

Inform the client or legal guardian of his right to execute an advance directive.

Several days before admission, a client reports finding a small lump in the left breast near the nipple. What should the nurse tell the client to do?

Inform the physician immediately.

A female client is at risk for developing osteoporosis. Which action will reduce the client's risk?

Initiating weight-bearing exercise routines

A female client is at risk for developing osteoporosis. Which action will reduce the client's risk?

Initiating weight-bearing exercise routines Explanation: Performing weight-bearing exercise increases bone health. A sedentary lifestyle increases the risk of developing osteoporosis. Estrogen is needed to promote calcium absorption. The recommended daily intake of calcium is 1,000 mg, not 300 mg

A female client is at risk for developing osteoporosis. Which action will reduce the client's risk? a) Taking a 300-mg calcium supplement to meet dietary guidelines b) Stopping estrogen therapy c) Living a sedentary lifestyle to reduce the incidence of injury d) Initiating weight-bearing exercise routines

Initiating weight-bearing exercise routines Explanation: Performing weight-bearing exercise increases bone health. A sedentary lifestyle increases the risk of developing osteoporosis. Estrogen is needed to promote calcium absorption. The recommended daily intake of calcium is 1,000 mg, not 300 mg.

The client is in skeletal traction. Which nursing intervention ensures proper care of this client?

Inspect the skin at least every 8 hours. Correct: Inspect the skin every 8 hours for signs of irritation, inflammation, or actual skin breakdown.

When caring for a client who is receiving external beam radiation, which is the key point for the nurse to incorporate into the plan of care?

Inspect the skin frequently.

"For a client newly diagnosed with radiation-induced thrombocytopenia, the nurse should include which intervention in the plan of care?

Inspecting the skin for petechiae once every shift

For a client newly diagnosed with radiation-induced thrombocytopenia, the nurse should include which intervention in the care plan?

Inspecting the skin for petechiae once every shift Correct

The nurse is caring for a client with a cast on his left arm. Which data collection finding is most significant for this client?

Intact skin around the cast edges

What does the nurse understand is the rationale for administering allopurinol for a patient receiving chemotherapy?

It lowers serum and uric acid levels.

The nurse is caring for a patient undergoing an incisional biopsy. Which of the following statements does the nurse understand is true about an incisional biopsy?

It removes a wedge of tissue for diagnosis.

What disadvantages of chemotherapy should the patient be informed about prior to starting the regimen?

It targets normal body cells as well as cancer cells.

A 78-year-old client has a history of osteoarthritis. Which signs and symptoms would the nurse expect to find on physical assessment?

Joint pain, crepitus, Heberden's nodes

Which of these findings best correlates with a diagnosis of osteoarthritis?

Joint stiffness that decreases with activity

The older adult client has had a right open reduction internal fixation (ORIF) of a fractured hip. Which intervention will the nurse implement for this client?

Keep the client's heels off the bed at all times. Correct: Because the client is an older adult and is more at risk for skin breakdown because of impaired circulation and sensation, the client's heels must be kept off the bed at all times to avoid constant pressure on this sensitive area.

"A client undergoes a laryngectomy to treat laryngeal cancer. When teaching the client how to care for the neck stoma, the nurse should include which instruction?

Keep the stoma moist.

A client has sustained a right tibial fracture and has just had a cast applied. Which instruction should the nurse provide regarding cast care?

Keep your right leg elevated above heart level.

A client undergoes hip-pinning surgery to treat an intertrochanteric fracture of the right hip. The nurse should include which intervention in the postoperative plan of care?

Keeping a pillow between the client's legs at all times

"A client who underwent surgery 2 days ago is unable to tolerate anything by mouth and is experiencing mild to moderate cancer pain. Which nonsteroidal anti-inflammatory drug (NSAID) can the nurse safely administer by the I.M. route?

Ketorolac (Toradol)

During a senior citizen health screening, the nurse observes a 75-year-old female with a severely increased thoracic curve, or "humpback". What is this condition called?

Kyphosis

Which of the following is an inaccurate clinical manifestation of a fracture?

Lengthening

Which assessment findings would the nurse expect in a client diagnosed with osteomyelitis?

Leukocytosis and localized bone pain

Which assessment findings would the nurse expect in a client diagnosed with osteomyelitis? a) Leukocytosis and localized bone pain b) Thrombocytopenia and ecchymosis c) Petechiae over the chest and abnormal ABGs d) Pruritus and uremic frost

Leukocytosis and localized bone pain Explanation: Clinical manifestations of osteomyelitis include signs and symptoms of sepsis and localized infection.

A decrease in circulating white blood cells (WBC) is referred to as which of the following?

Leukopenia

"A client with cancer is being evaluated for possible metastasis. Which of the following is a common metastasis site for cancer cells?

Liver

A client with cancer is being evaluated for possible metastasis. What is one of the most common metastasis sites for cancer cells?

Liver

Assessment of a client reveals signs and symptoms of Paget's disease. Which of the following would be most likely?

Long bone bowing

Assessment of a client reveals signs and symptoms of Paget's disease. Which of the following would be most likely? a) Skull narrowing b) Waddling gait c) Lordosis d) Long bone bowing

Long bone bowing Explanation: Some clients with Paget's disease are asymptomatic with only some mild skeletal deformity. Other clients have marked skeletal deformities which may include enlargement of the skull, bowing of the long bones, and kyphosis. Waddling gait is associated with osteomalacia.

Which of the following would the nurse most commonly assess in a client with ankylosing spondylitis?

Low back pain

Which area of the spinal column is subject to the greatest mechanical stress and degenerative changes?

Lower lumbar

Which area of the spinal column is subject to the greatest mechanical stress and degenerative changes? a) Lower lumbar b) Cervical c) Thoracic d) Upper lumbar

Lower lumbar Explanation: The lower lumbar disks, L4 to L5 and L5 to S1, are subject to the greatest mechanical stress and greatest degenerative changes.

"A client with uterine cancer asks the nurse, ""Which type of cancer causes the most deaths in women?"" How should the nurse respond?

Lung cancer.

A client with ankylosing spondylitis has a stooped position and is being positioned in the bed prior to the nurse taking vital signs. The nurse listens to the client's lungs after positioning. What finding does the nurse hear when listening to lung sounds?

Lung sounds are diminished in the apical area.

A client is undergoing a left modified radical mastectomy for breast cancer. Postoperatively, blood pressure should be obtained from the client's right arm, and the left arm and hand should be elevated as much as possible to prevent which condition?

Lymphedema

Which nursing intervention is appropriate for minimizing muscle spasms in the client with a hip fracture?

Maintain Buck's traction.

A client with degenerative joint disease asks the nurse for suggestions to avoid unusual stress on the joints. Which suggestion would be most appropriate?

Maintain good posture.

A client who reports increasing difficulty swallowing, weight loss, and fatigue is diagnosed with esophageal cancer. Because this client has difficulty swallowing, the nurse should assign highest priority to:

Maintaining a patent airway.

The nurse is teaching the family of a client diagnosed with leukemia about ways to prevent infection. Which instruction has the most impact?

Maintaining an intact skin integrity

A client is in Buck's skin traction after fracturing his right hip. The nurse should include which action in the plan of care?

Maintaining correct body alignment

On a visit to the gynecologist, a client complains of urinary frequency, pelvic discomfort, and weight loss. After a complete physical examination, blood studies, and a pelvic examination with a Papanicolaou test, the physician diagnoses stage IV ovarian cancer. The nurse expects to prepare the client for which initial treatment?

Major surgery

Which should be included in the teaching plan for a patient diagnosed with plantar fasciitis? a) Plantar fasciitis presents as an acute onset of pain localized to the ball of the foot. b) Complications of plantar fasciitis include neuromuscular damage and decreased ankle range of motion. c) Management of plantar fasciitis includes stretching exercises. d) The pain of plantar fasciitis diminishes with warm water soaks.

Management of plantar fasciitis includes stretching exercises. Explanation: Management also includes wearing shoes with support and cushioning to relieve pain, orthotic devices (e.g., heel cups, arch supports), and the use of nonsteroidal anti-inflammatory drugs (NSAIDs). Plantar fasciitis, an inflammation of the foot-supporting fascia, presents as an acute onset of heel pain experienced with taking the first steps in the morning. The pain is localized to the anterior medial aspect of the heel and diminishes with gentle stretching of the foot and Achilles tendon. Unresolved plantar fasciitis may progress to fascial tears at the heel and eventual development of heel spurs.

A nurse is caring for a client who is undergoing chemotherapy. The client has experienced several episodes of vomiting. Which activity should the nurse perform as part of ongoing assessment for signs of dehydration and electrolyte imbalances?

Measure all fluid intake and output

A nurse notices that a client's flexibility of the right elbow is less than the left elbow. What is an appropriate action by the nurse in regard to this finding?

Measure movement with a goniometer.

A medication nurse is preparing to administer 9 a.m. medications to a client with liver cancer. Which consideration is the nurse's highest priority?

Metabolism of the medication. The rate and ability of the liver to metabolize medications will be altered in a client with liver cancer. Therefore, it is essential to understand how each medication is metabolized. The other considerations are important but not as vital.

A patient with metastatic pancreatic cancer underwent surgery for removal of a malignant tumor in his pancreas. Despite the tumor being removed, the physician informs the patient that he needs to start chemotherapy. Which of the following may be a reason for the physician to opt for chemotherapy?

Metastasis

The nurse is discussing the new medication that a client will be taking for treatment of rheumatoid arthritis. Which disease-modifying antirheumatic drug (DMARD) will the nurse educate the client about?

Methotrexate (Rheumatrex)

Which drug should be prescribed with nonsteroidal inflammatory drugs (NSAIDs) in elderly cancer clients to reduce the risk of GI adverse effects?

Misoprostol (Cytotec)

A client is admitted to the orthopedic unit with septic arthritis of the knee. The case manager should be consulted if which of the following complications occurs during hospitalization?

Mobility decreases.

The nurse is planning a presentation on osteoporosis to a group of high school students. Which of the following should the nurse plan to include in the presentation?

Moderate strenuous exercise tends to increase bone density.

osteoporosis risk factors

Modifiable risk factors include low estrogen levels. Small-boned thin frame, personal history of fractures, and age cannot be modified.

The nurse working on a bone marrow unit knows that it is a priority to monitor which of the following in a client who has just undergone a stem cell transplant?

Monitor the client closely to prevent infection.

Which of the following classic symptoms would the nurse assess for to detect the development of plantar fasciitis? a) Shortening of affected leg b) Elevated temperature c) Morning heel pain d) Shortened height

Morning heel pain Explanation: Plantar fasciitis is characterized by heel pain.

The nurse is testing the client for extension of the wrist and notes weakness on the right side. This assessment finding is consistent with what disease of the nervous system?

Multiple Sclerosis

A client with muscle weakness and an abnormal gait is being evaluated for muscular dystrophy. Which of the following confirms muscular dystrophy?

Muscle biopsy

A client is undergoing an extensive diagnostic workup for suspected muscular dystrophy. The nurse knows that muscular dystrophy has many forms, but that one data collection finding is common to all forms. Which finding belongs in this category?

Muscle weakness

According to the tumor-node-metastasis (TNM) classification system, T0 means there is which of the following?

No evidence of primary tumor

After a person experiences a closure of the epiphyses, which of the following is true?

No further increase in bone length occurs.

A nurse has completed a medication reconciliation of a patient who has been admitted following a motor vehicle accident. Among the many drugs that the patient has received in the previous year is rituximab. The nurse would be justified in suspecting the patient may have received treatment for which of the following diseases?

Non-Hodgkin's lymphoma

A client tells the nurse that she has found a painless lump in her right breast during her monthly self-examination. Which assessment finding would strongly suggest that this client's lump is cancerous?

Nonmobile mass with irregular edges Breast cancer tumors are fixed, hard, and poorly delineated with irregular edges. A mobile mass that is soft and easily delineated is most commonly a fluid-filled benign cyst. Axillary lymph nodes may or may not be palpable on initial detection of a cancerous mass. Nipple retraction — not eversion — may be a sign of cancer.

According to the World Health Organization's analgesic ladder, which drugs would likely be used first to treat mild pain?

Nonsteroidal anti-inflammatory drugs (NSAIDs)

Which of the following terms refers to failure of fragments of a fractured bone to heal together?

Nonunion

During assessment, a licensed practical nurse (LPN) notes that a client who had a total hip replacement 5 days ago has a temperature of 100.4° F (38° C) and a red, edematous incision. The dressing that was covering the incision contains yellow-green, foul-smelling drainage, and the client complains of pain at the site. How should the nurse proceed?

Notify a registered nurse (RN) coassigned to the client, apply a sterile dressing to the incision, and administer pain medication as prescribed.

A client recovering from back surgery tells a nurse that she's concerned about going home. She explains that she has many stairs to navigate and household responsibilities she must perform. How can the nurse help ease this client's concerns?

Notify the charge nurse of the client's concerns, and request a team meeting to discuss the client's discharge planning.

After a physician describes the surgical procedure for lumbar spinal fusion and its associated risks, the nurse provides a consent form for the client to sign. The client asks the nurse what the term "fusion" means and whether he'll lose a lot of blood during the procedure. Which action should the nurse take?

Notify the physician of the client's questions about the procedure before having the client sign the informed consent form.

After surgery for gastric cancer, a client is scheduled to undergo radiation therapy. It will be most important for the nurse to include information about which of the following in the client's teaching plan?

Nutritional intake. Correct Explanation: Clients who have had gastric surgery are prone to postoperative complications, such as dumping syndrome and postprandial hypoglycemia, that can affect nutritional intake. Vitamin absorption can also be an issue, depending on the extent of the gastric surgery. Radiation therapy to the upper gastrointestinal area also can affect nutritional intake by causing anorexia, nausea, and esophagitis. The client would not be expected to develop alopecia. Exercise and activity levels as well as access to community resources are important teaching areas, but nutritional intake is a priority need.

A 69-year-old client asks the nurse what the difference is between osteoarthritis (OA) and rheumatoid arthritis (RA). Which response is correct?

OA is a noninflammatory joint disease. RA is characterized by inflamed, swollen joints.

At a health fair, an oncology nurse answers questions about risk factors for various types of cancer. One person with a family history of colon cancer asks the nurse to identify risk factors for this type of cancer. The nurse should identify

Obesity

The client has undergone an elective below-knee amputation of the right leg as a result of severe peripheral vascular disease. In postoperative care teaching, the nurse instructs the client to notify the health care provider if which change occurs?

Observation of a large amount of serosanguineous or bloody drainage Correct: A large amount of serosanguineous or bloody drainage may indicate hemorrhage or, if an incision is present, that the incision has opened. This requires immediate attention.

A family member is caring for an older adult client with osteomalacia in the home. When the home health nurse comes to evaluate the client, what should be a focus point of the visit?

Observing for safety hazards that could be a fall risk

A family member is caring for an older adult client with osteomalacia in the home. When the home health nurse comes to evaluate the client, what should be a focus point of the visit? a) Making sure the client has adequate financial resources b) Observing for safety hazards that could be a fall risk c) Ensuring that the client is eating enough d) Making sure the client is receiving a daily bath

Observing for safety hazards that could be a fall risk Explanation: Clients with osteomalacia exhibit a waddling type of gait, putting them at risk for falls and fractures. Safety would be the priority in this circumstance such as scatter rugs, loose boards, and stairs. Older adult clients do not require a daily bath, and it may dry the skin. Nutrition is a necessity to question but the priority would be safety. Whether the client has adequate financial resources would be referred to social service.

"A 30-year-old client whose mother died of breast cancer at age 44 and whose sister has ovarian cancer is concerned about developing cancer. A nurse who is a member of the oncology multidisciplinary team should suggest that the client ask the physician about which of the following actions?

Obtaining genetic counseling

The nurse anticipates providing collaborative care for a client with a traumatic amputation of the right hand with which health care team members? Select all that apply.

Occupational therapist Correct Physical therapist Correct Psychologist Correct Correct Feedback: Correct: An occupational therapist will help to enable the client to become more independent in performing activities of daily living. Correct: A physical therapist will help to enable the client to become more independent in performing activities of daily living. Correct: An amputation can be traumatic to the client; loss of a body part should not be underestimated because the client may experience an altered self-concept. Counseling support should be made available to the client..

A client is brought to the emergency department after injuring his right arm in a bicycle accident. The orthopedic surgeon tells the nurse that the client has a greenstick fracture of the arm. What does this mean?

One side of the bone is broken and the other side is bent

A client is brought to the emergency department after injuring his right arm in a bicycle accident. The orthopedic surgeon tells the nurse that the client has a greenstick fracture of the arm. What does this mean?

One side of the bone is broken and the other side is bent.

A nurse is teaching a community class about how to decrease the risk of cancer. What is the best food for the nurse to recommend?

Oranges

The client is complaining that his lower joints are increasingly painful as the day progresses. The nurse suspects the client is experiencing what musculoskeletal disorder?

Osteoarthritis

Osteoarthritis

Osteoarthritis is characterized by pain with motion that increases throughout the day

Which cells are involved in bone resorption?

Osteoclasts

Which of the following is the most common and most fatal primary malignant bone tumor?

Osteogenic sarcoma (osteosarcoma)

Which of the following is the most common and most fatal primary malignant bone tumor? a) Osteochondroma b) Enchondroma c) Rhabdomyoma d) Osteogenic sarcoma (osteosarcoma)

Osteogenic sarcoma (osteosarcoma) Explanation: Osteogenic sarcoma (osteosarcoma) is the most common and most often fatal primary malignant bone tumor. Benign primary neoplasms of the musculoskeletal system include osteochondroma, enchondroma, and rhabdomyoma.

Which of the following is a metabolic bone disease that is characterized by inadequate mineralization of bone?

Osteomalacia

The client presents to the emergency department with fever, chills, restlessness, and limited movement of a fractured jaw. The nurse interprets these findings as indicating which of the following complications?

Osteomyelitis

The client presents to the emergency department with fever, chills, restlessness, and limited movement of a fractured jaw. The nurse interprets these findings as indicating which of the following complications? a) Osteomyelitis b) Fat embolism c) Avascular necrosis d) Compartment syndrome

Osteomyelitis Explanation: Clinical manifestations of osteomyelitis include signs and symptoms of sepsis and localized infection.

Osteoporosis creates risks for

Osteoporosis creates risks for injury, activity intolerance, and impaired mobility as consequences of musculoskeletal changes. The disease does not normally result in infection or impaired sensation. • Risk for injury related to osteoporosis • Activity intolerance related to osteoporosis • Impaired physical mobility related to osteoporosis

Localized rapid bone turnover, most commonly affecting the skull, femur, tibia, pelvic bones, and vertebrae, is characterized by which of the following bone disorders? a) Osteomyelitis b) Ganglion c) Osteomalacia d) Paget's disease

Paget's disease Explanation: Paget's disease results in bone that is highly vascularized and structurally weak, predisposing to pathologic fractures. Osteomalacia is a metabolic bone disease characterized by inadequate mineralization of bone. A ganglion is a collection of neurological gelatinous material. Osteomyelitis is an infection of bone that comes from the extension of a soft tissue infection, direct bone contamination, or hematogenous spread.

arthritis.

Pain and stiffness in the joints is associated with arthritis Crepitus and decreased range of motion is associated with arthritis

A client seeks care for low back pain of 2 weeks' duration. Which data collection finding suggests a herniated intervertebral disk?

Pain radiating down the posterior thigh

Rotator cuff tear

Painful and limited abduction accompanied by muscle weakness and atrophy are seen with rotator cuff tears. Rotator cuff tendinitis causes the client to report sharp catches of pain when bringing the hands overhead. A bone fracture presents with acute, severe pain, and often weakness of the entire extremity. Degenerative joint disease may cause limited range of motion for all of the shoulder movements and most likely occurs symmetrically

Which finding is an early indicator of bladder cancer?

Painless, intermittent hematuria

Treatment of metastatic bone cancer includes which of the following? a) Radiation b) Palliation c) Chemotherapy d) Combination chemotherapy and radiation

Palliation Explanation: The treatment of metastatic bone cancer is palliative. The therapeutic goal is to relieve the patient's pain and discomfort while promoting quality of life.

A client is undergoing a diagnostic workup for suspected thyroid cancer. What is the most common form of thyroid cancer in adults?

Papillary carcinoma

The nurse is assessing a client with multiple myeloma. The nurse should keep in mind that clients with multiple myeloma are at risk for:

Pathologic bone fractures.

The client presents at the clinic with a history of cerebral palsy. When examining the patient the nurse notes increased resistance that is rate dependent and increases with rapid movement. What would the nurse chart about this patient?

Patient demonstrates spasticity.

An older adult client has multiple tibia and fibula fractures of the left lower extremity after a motor vehicle accident. Which pain medication does the nurse anticipate will be requested for this client?

Patient-controlled analgesia (PCA) with morphine Correct: Morphine is an opioid narcotic analgesic; given through PCA, it is the most appropriate mode of pain management for this type of acute pain associated with multiple injuries.

Which action by a nurse is a correct method for performing the Tinel's test to determine the presence of carpel tunnel syndrome?

Percuss lightly on the inner aspect of the wrist.

After being in remission from Hodgkin's disease for 18 months, a client develops a fever of unknown origin. The physician orders a blind liver biopsy to rule out advancing Hodgkin's disease and infection. Twenty-four hours after the biopsy, the client has a fever, complains of severe abdominal pain and rigidity, and seems increasingly confused. The nurse suspects that these findings result from:

Perforation of the colon caused by the liver biopsy.

The nurse is caring for the client following removal of a Morton's neuroma. Which of the following nursing interventions would be inappropriate?

Perform N/V assessment of the hand

The nurse is caring for a client following foot surgery. Which nursing intervention is most important for the nurse to include in the nursing care plan? a) Administer pain medication per client request. b) Monitor vital signs every 4 hours. c) Examine surgical dressing every hour. d) Perform neuromuscular assessment every hour.

Perform neuromuscular assessment every hour. Explanation: The priority nursing intervention is to perform a neuromuscular assessment every hour. Early detection of neurological and perfusion problems is critical.

The nurse is caring for the client following removal of a Morton's neuroma. Which of the following nursing interventions would be inappropriate? a) Assess the surgical dressing. b) Assist the client with incentive spirometry. c) Elevate the foot on two pillows. d) Perform neurovascular assessment of the hand.

Perform neurovascular assessment of the hand. Explanation: Morton's neuroma is a foot problem characterized by swelling of the median plantar nerve.

Which of the following would the nurse use to determine that a client is exhibiting signs and symptoms of chronic osteomyelitis?

Persistent draining sinus

Which of the following is an early sign of laryngeal cancer?

Persistent mild hoarseness.

Pes cavus refers to

Pes cavus refers to feet with high arches. (less)

"A client is in isolation after receiving an internal radioactive implant to treat cancer. Two hours later, the nurse discovers the implant in the bed linens. What should the nurse do first?

Pick up the implant with long-handled forceps and place it in a lead-lined container.

After a transsphenoidal adenohypophysectomy, a client is likely to undergo hormone replacement therapy. A transsphenoidal adenohypophysectomy is performed to treat which type of cancer?

Pituitary carcinoma Pituitary carcinoma most commonly arises in the anterior pituitary (adenohypophysis) and must be removed by way of a transsphenoidal approach, using a bivalve speculum and rongeur. Surgery to treat esophageal carcinoma usually is palliative and involves esophagogastrectomy with jejunostomy. Laryngeal carcinoma may necessitate a laryngectomy. To treat colorectal cancer, the surgeon removes the tumor and any adjacent tissues and lymph nodes that contain cancer cells.

Which of the following presents with an onset of heel pain with the first steps of the morning?

Plantar fasciitis

Which of the following presents with an onset of heel pain with the first steps of the morning? a) Plantar fasciitis b) Morton's neuroma c) Ganglion d) Hallux valgus

Plantar fasciitis Explanation: Plantar fasciitis, an inflammation of the foot-supporting fascia, present as an acute onset of heal pain experienced with the first steps in the morning. Hallux valgus (commonly called a bunion) is a deformity in which the great toe deviates laterally. Morton's neuroma is a swelling of the third (lateral) branch of the median plantar nerve. A ganglion, a collection of gelatinous material near the tendon sheaths and joints, appears as a round, firm compressible cystic swelling, usually on the dorsum of the wrist.

The nurse is interviewing a client about his past medical history. Which preexisting condition may lead the nurse to suspect that a client has colorectal cancer?

Polyps

A client has had several diagnostic tests to determine if he has systemic lupus erythematosus (SLE). What result is very specific indicator of this diagnosis?

Positive Anti-dsDNA antibody test

A client comes to the outpatient department with suspected carpal tunnel syndrome. When assessing the affected area, the nurse expects to find which abnormality typically associated with this syndrome?

Positive Tinel's sign

A 55-year-old woman with a history of type 2 diabetes went through menarche at age 19 and menopause 2 years ago. Which of the preceding is a risk factor for osteoporosis?

Postmenopausal status

Which goal is the priority for a client with a fractured femur who is in traction at this time?

Prevent effects of immobility while in traction. Explanation: The priority for this client is to prevent the effects of prolonged immobility, such as preventing skin breakdown and encouraging the client to take deep breaths, and use active range-of-motion exercises for the joints that are not immobilized. Although not the priority, the nurse also should seek ways to help the client adjust to and cope with the present state of immobility. Emphasis should be placed on what the client can do, such as participating in daily care and exercises to maintain muscle strength. Finding diversional activities is not a priority at this moment. Although the client must adapt to the inactivity, helping the client develop coping skills is the priority at this time.

A nurse is caring for a client who underwent a total hip replacement. What should the nurse and other caregivers do to prevent dislocation of the new prosthesis?

Prevent internal rotation of the affected leg.

A nurse is caring for a client who underwent a total hip replacement. What should the nurse and other caregivers do to prevent dislocation of the new prosthesis?

Prevent internal rotation of the affected leg. Explanation: The nurse and other caregivers should prevent internal rotation of the affected leg. However, external rotation and abduction of the hip will help prevent dislocation of a new hip joint. Postoperative total hip replacement clients may be turned onto the unaffected side. The hip may be flexed slightly, but it shouldn't exceed 90 degrees. Maintenance of flexion isn't necessary.

Which nursing intervention is most appropriate for a client with multiple myeloma

Preventing bone injury

During which step of cellular carcinogenesis do cellular changes exhibit increased malignant behavior?

Progression

The nurse recognizes that goal of treatment for metastatic bone cancer is to: a) Promote pain relief and quality of life b) Diagnose the extent of bone damage c) Reconstruct the bone with a prosthesis d) Cure the diseased bone and cartilage

Promote pain relief and quality of life Explanation: Treatment of metastatic bone cancer is palliative.

The nurse at the clinic explains to the patient that the surgeon will be removing a mole on the patient's back that has the potential to develop into cancer. The nurse informs the patient that this is what type of procedure?

Prophylactic

You are a clinic nurse. One of your clients has found she is at high risk for breast cancer. She asks you what can be done to reduce her risk. What is a means of reducing the risk for breast cancer?

Prophylactic surgery

An elderly man has been admitted to a residential care facility and the nurse has conducted a medication reconciliation. The man has taken numerous drugs in the past, including a course of bicalutamide (Casodex) several years earlier. The nurse recognizes this drug as being an antiandrogen and is consequently justified in presuming that the man has a history of what disease?

Prostate cancer

A patient with a traumatic amputation of the right lower leg is refusing to look at the leg. Which of the following actions by the nurse is most appropriate?

Provide feedback on the patient's strengths and available resources.

A client with a cancerous tumor must undergo a modified radical mastectomy, which includes axillary node removal and immediate breast reconstruction. The nurse explains to the client that the axillary nodes will be removed in order to:

Provide prognostic information.

Doxorubicin is prescribed for a female client with breast cancer. The client is distressed about hair loss. The nurse should do which of the following?

Provide resources for a wig selection before hair loss begins. Correct Explanation: Resources should be provided for acquiring a wig since it is easier to match hair style and color before hair loss begins. The client has expressed negative feelings of self image with hair loss. Excessive shampooing and manipulation of hair will increase hair loss. Hair usually grows back in 3 to 4 weeks after the chemotherapy is finished, however new hair may have a new color or texture. A wig, hairpiece, hat, scarf, or turban can be used to conceal hair loss. Social isolation should be avoided and the client should be encouraged to socialize with others.

Which of the following observations should the nurse make first when the client who has had a modified radical mastectomy returns from the operating room to the recovery room?

Provide resources for a wig selection before hair loss begins. Correct Explanation: Resources should be provided for acquiring a wig since it is easier to match hair style and color before hair loss begins. The client has expressed negative feelings of self image with hair loss. Excessive shampooing and manipulation of hair will increase hair loss. Hair usually grows back in 3 to 4 weeks after the chemotherapy is finished, however new hair may have a new color or texture. A wig, hairpiece, hat, scarf, or turban can be used to conceal hair loss. Social isolation should be avoided and the client should be encouraged to socialize with others.

After a physician explains the risks and benefits of a clinical trial to a client, the client agrees to participate. Later that day, the client requests clarification of the process involved in the clinical trial. As a member of the multidisciplinary team, how should the nurse respond?

Provide the information requested.

A patient is admitted for an excisional biopsy of a breast lesion. What intervention should the nurse provide for the care of this patient?

Provide time for the patient to discuss her concerns.

During chemotherapy, an oncology client has a nursing diagnosis of Impaired oral mucous membrane related to decreased nutrition and immunosuppression secondary to the cytotoxic effects of chemotherapy. Which nursing intervention is most likely to decrease the pain of stomatitis?

Providing a solution of hydrogen peroxide and water for use as a mouth rinse

When the client complains of increased fatigue following radiotherapy, the nurse knows this is most likely to be related to which factor?

Radiation can result in myelosuppression.

A client with vaginal cancer asks the nurse, "What is the usual treatment for this type of cancer?" Which treatment should the nurse name?

Radiation therapy

Which of the following terms refers to disease of a nerve root?

Radiculopathy

Which of the following terms refers to disease of a nerve root? a) Radiculopathy b) Involucrum c) Sequestrum d) Contracture

Radiculopathy Explanation: When the patient reports radiating pain down the leg, he or she is describing radiculopathy. Involucrum refers to new bone growth around the sequestrum. Sequestrum refers to dead bone in an abscess cavity. Contracture refers to abnormal shortening of muscle or fibrosis of joint structures.

Which of the following is the only selective estrogen receptor modulator approved for osteoporosis in post menopausal women?

Raloxifene

Which of the following is the only selective estrogen receptor modulator approved for osteoporosis in post menopausal women? a) Forteo b) Fosamax c) Raloxifene d) Denosumab

Raloxifene Explanation: Raloxifene is the only selective estrogen receptor modulator (SERM) approved for osteoporosis in post menopausal women as it does not increase the risk of breast or uterine cancer, but it does come with an increased risk of thromboembolism. Fosamax is a bisphosphonate. Forteo is a subcutaneously administered medication that is given one daily for the treatment of osteoporosis. Denosumab has recently been approved for treatment of postmenopausal women with osteoporosis who are at risk for fractures.

A client with osteoporosis is prescribed a selective estrogen receptor modifier (SERM) as treatment. The nurse would identify which drug as belonging to this class? a) Raloxifene (Evista) b) Calcium gluconate c) Tamoxifen (Nolvadex) d) Alendronate (Fosamax)

Raloxifene (Evista) Explanation: An example of a selective estrogen receptor modifier (SERM) is raloxifene (Evista). Alendronate is a biphosphonate; calcium gluconate is an oral calcium preparation; tamoxifen is an antiestrogen agent.

Which of the following measures is most important for pain management for a client after a lobectomy for lung cancer?

Reassess the client after administering pain medication. Correct Explanation: It is essential for the nurse to evaluate the effects of pain medication after it has had time to act. Although other interventions may be appropriate, continual reassessment is most important to determine the effectiveness and need for additional intervention, if any. Repositioning could provide some comfort, but assessment of the client's pain level is essential. Reassuring the client is important, but it will be of no value unless the nurse evaluates the client's pain level. To readjust the pain dosage is appropriate only if titration is prescribed by the physician.

Which intervention should the nurse implement, when caring for the client who complains of phantom limb pain two months after amputation?

Reassure the client that phantom pain is common

A client has been receiving chemotherapy to treat cancer. Which data collection finding suggests that the client has developed stomatitis (inflammation of the mouth)?

Red, open sores on the oral mucosa

Ms. Malcolm has come to your clinic complaining of jaw pain. This is also associated with muscle spasm and tenderness of the masseter and temporalis muscles. The physician has diagnosed a temporomandibular disorder (TMD). What would the treatment course for this client include? Select all that apply.

Referral to a dentist who has experience managing clients with TMD. Analgesics. Custom-fitted mouth guard during sleep.

A client with a cerebellar brain tumor is admitted to an acute care facility. The nurse assists with formulating a nursing diagnosis of Risk for injury. Which "related-to" phrase is appropriate for the nurse to add to complete the nursing diagnosis statement?

Related to impaired balance

A client with a cerebellar brain tumor is admitted to an acute care facility. The nurse formulates a nursing diagnosis of Risk for injury. Which "related-to" phrase should the nurse add to complete the nursing diagnosis statement?

Related to impaired balance Explanation: A client with a cerebellar brain tumor may suffer injury from impaired balance as well as disturbed gait and incoordination. Visual field deficits, difficulty swallowing, and psychomotor seizures may result from dysfunction of the pituitary gland, pons, occipital lobe, parietal lobe, or temporal lobe — not from a cerebellar brain tumor. Difficulty swallowing suggests medullary dysfunction. Psychomotor seizures suggest temporal lobe dysfunction.

What is the most important postoperative instruction the nurse must give a client who has just returned from the operating room after receiving a subarachnoid block?

Remain supine for the time specified by the physician.

Which nursing intervention is essential in caring for a client with compartment syndrome?

Removing all external sources of pressure, such as clothing and jewelry

A nurse is caring for a client with bronchogenic carcinoma. Which nursing intervention takes highest priority?

Removing pulmonary secretions

A client with Paget's disease comes to the hospital and complains of difficulty urinating. The emergency department physician consults urology. What should the nurse suspect is the most likely cause of the client's urination problem?

Renal calculi

A client with Paget's disease comes to the hospital and complains of difficulty urinating. The emergency department physician consults urology. What should the nurse suspect is the most likely cause of the client's urination problem? a) Benign prostatic hyperplasia b) Dehydration c) Urinary tract infection (UTI) d) Renal calculi

Renal calculi Explanation: Renal calculi commonly occur with Paget's disease, causing pain and difficulty when urinating. A UTI commonly causes fever, urgency, burning, and hesitation with urination. Benign prostatic hyperplasia is common in men older than age 50; however, because the client has Paget's disease, the nurse should suspect renal calculi, not benign prostatic hyperplasia. Dehydration causes a decrease in urine production, not a problem with urination.

The nurse is giving instructions to a client who's going home with a cast on his leg. Which point is most critical?

Reporting signs of impaired circulation

Four clients on an orthopedic unit are scheduled to attend physical therapy at the same time. Facility policy dictates that each client be escorted to therapy in a wheelchair or on a stretcher. When it's time for therapy, only three wheelchairs are available. One of the four clients is learning crutch-walking and is scheduled for discharge in the morning. What should the nurse do to ensure the clients' safety and timely arrival to physical therapy?

Request that a physical therapist accompany the client to therapy while he uses the crutches.

An incarcerated client is admitted to the hospital after sustaining multiple contusions and a fractured femur in an assault. After surgical repair of the femur, the client develops paralytic ileus. A nasogastric (NG) tube and cleansing enemas are prescribed. The client has a prison guard assigned to his bedside. How should a nurse proceed to implement a physician's orders?

Request that the guard remain outside the client's door during the prescribed procedures.

The client is brought to the emergency department (ED) via ambulance after a motor vehicle accident. What condition will the nurse assess for first?

Respiratory distress Correct: The client is first assessed for respiratory distress, and any oxygen interventions are instituted accordingly.

Rheumatoid arthritis

Rheumatoid arthritis discomfort decreases with motion.

Rheumatoid arthritis

Rheumatoid arthritis is a systemic disease and accounts for multiple symmetrically involved joints. Septic arthritis is usually monoarticular, as are gout and trauma related joint pain.

A patient with non-Hodgkin's lymphoma (NHL) will be starting a course of doxorubicin shortly. When planning this patient's care, what nursing diagnosis should the nurse prioritize?

Risk for Infection related to suppressed bone marrow function

A client has Paget's disease. An appropriate nursing diagnosis for this client is:

Risk for falls

A client has Paget's disease. An appropriate nursing diagnosis for this client is: a) Risk for falls b) Delayed wound healing c) Fatigue d) Risk for infection

Risk for falls Explanation: The client with Paget's disease is at risk for falls secondary to pathological fractures and impaired gait/mobility.

After being seen in the oncology clinic, a client is admitted to the hospital with severe bone marrow depression. The client's cancer therapy consisted of radiation and chemotherapy. Which nursing diagnosis takes priority when developing this client's plan of care?

Risk for infection

After being seen in the oncology clinic, a client with severe bone marrow suppression is admitted to the hospital. The client's cancer therapy consisted of radiation and chemotherapy. When developing the care plan for this client, which nursing diagnosis takes priority?

Risk for infection

Which nursing diagnosis takes highest priority for a client with a compound fracture?

Risk for infection related to effects of trauma

Which nursing diagnosis is most appropriate for an elderly client with osteoarthritis?

Risk for injury related to altered mobility

Which of the following musculoskeletal injuries is manifested by acromioclavicular joint pain?

Rotator cuff tears

Scale for grading muscle strength

Scale for grading muscle strength: muscle strength is graded on a 0 to 5 scale: 0—No muscular contraction detected 1— A barely detectable flicker or trace of contraction 2— Active movement of the body part with gravity eliminated 3— Active movement against gravity 4— Active movement against gravity and some resistance 5— Active movement against full resistance without evident fatigue. This is normal muscle strength

A 19-year-old patient presents to the emergency room with an injury to her left ankle that occurred during a high school basketball game. She complains of limited motion and pain on walking, which increased over the last 2 hours. The nurse knows that her diagnosis is most likely which of the following?

Second-degree sprain

A client is prescribed diazepam (Valium) to treat severe skeletal muscle spasms. During this therapy, the nurse monitors the client closely for adverse reactions. Which adverse reaction is most likely to occur?

Sedation

Which information about a client who was admitted with pelvic and bilateral femoral fractures after being crushed by a tractor is most important for the nurse to report to the physician?

Serum potassium level is 7 mEq/L. Correct: The elevated potassium level may indicate that the client has rhabdomyolysis and acute tubular necrosis caused by the crush injury. Further assessment and treatment are needed immediately to prevent further renal damage or cardiac dysrhythmias.

A client is receiving chemotherapy to treat breast cancer. Which assessment finding indicates a chemotherapy-induced complication?

Serum potassium level of 2.6 mEq/L

A client with testicular cancer is scheduled for a right orchiectomy. The day before surgery, the client tells the nurse that he is concerned about the effect that losing a testicle will have on his manhood. Which information about orchiectomy should form the basis for the nurse's response?

Sexual drive and libido are unchanged. Explanation: The remaining testicle undergoes hyperplasia and produces enough testosterone to maintain sexual drive, libido, and secondary sexual characteristics. Testosterone levels will return to normal. Sperm count can decrease after a unilateral orchiectomy; this is attributed to the stress of the surgery. Secondary sexual characteristics do not change because the remaining testicle continues to produce testosterone.

"A client with stage II ovarian cancer undergoes a total abdominal hysterectomy and bilateral salpingo-oophorectomy with tumor resection, omentectomy, appendectomy, and lymphadenectomy. During the second postoperative day, which data collection finding would raise concern in the nurse?

Shallow breathing and increasing lethargy

A client with stage II ovarian cancer undergoes a total abdominal hysterectomy and bilateral salpingo-oophorectomy with tumor resection, omentectomy, appendectomy, and lymphadenectomy. During the second postoperative day, which assessment finding requires immediate intervention?

Shallow breathing and lethargy - Shallow breathing and a change in the level of consciousness, such as increasing lethargy requires immediate intervention because they may indicate a respiratory complication — for example, atelectasis or carbon dioxide retention. To avoid respiratory complications, the nurse should encourage turning, coughing, deep breathing, and ambulation during the early postoperative period. Abdominal pain, hypoactive bowel sounds, and serous drainage from the incision are expected findings during the first few days after this type of surgery.

What should the nurse tell a female client who is about to begin chemotherapy and anxious about losing her hair?

She should consider getting a wig or cap before she loses her hair.

A 35-year-old female client is requesting information about mammograms and breast cancer. She isn't considered at high risk for breast cancer. What should the nurse tell this client?

She should eat a low-fat diet to further decrease her risk of breast cancer.

On discharge, a client who underwent a left mastectomy expresses relief that "the cancer" has been treated. When discussing this issue with the client, the nurse should stress that she:

Should continue to perform breast self-examination on her right breast

A client with rheumatoid arthritis has infiltration of the lacrimal and salivary glands with lymphocytes as a result of the disease. What does the nurse understand that this clinical manifestation is?

Sicca syndrome

A male client is to have an amputation. He is acutely ill and diagnosed with a gangrenous limb and related fever, disorientation, and electrolyte imbalances. Which of the following would be most important for the nurse to monitor in this client? a) Signs of nausea and vomiting b) Signs of sepsis c) Occurrence of allergic reactions d) Reduced urine output

Signs of sepsis Explanation: If the client is acutely ill with a gangrenous limb, related fever, disorientation, and electrolyte imbalances, the nurse should monitor for signs of sepsis and circulation in the limb for any changes such as severe pain, color changes, and lack of peripheral pulses. It is crucial for the nurse to inform the physician about the problems as they occur or else the surgery may become an emergency. Monitoring for signs of nausea and vomiting, occurrence of allergic reactions, and reduced urine output, although necessary, is not as crucial for the client.

tendons

Skeletal muscles attach to bones by way of strong, fibrous cords called tendons

The client has sustained a rotator cuff tear while playing baseball. The nurse anticipates that the client will receive which immediate conservative treatment?

Sling for the affected arm Correct: The conservative treatment for this client is to place the injured arm in a sling or immobilizer.

Which is a risk-lowering strategy for osteoporosis?

Smoking cessation

Which is a risk-lowering strategy for osteoporosis? a) Increased age b) Smoking cessation c) Diet low in calcium and vitamin D d) Low initial bone mass

Smoking cessation Explanation: Risk-lowering strategies include increased dietary calcium and vitamin D intake, smoking cessation, alcohol and caffeine consumption in moderation, and outdoor activity. Individual risk factors include low initial bone mass and increased age. A lifestyle risk factor is a diet low in calcium and vitamin D.

On a visit to the family physician, a client complains of painful swelling on the lateral side of the great toe, at the metatarsophalangeal joint. After determining that the swelling is a bunion, the physician injects an intra-articular corticosteroid. The client asks the nurse what causes bunions. Which answer is correct?

Some bunions are congenital; others are caused by wearing shoes that are too short or narrow.

A client sustains an injury to the ligaments surrounding a joint. The nurse identifies this as which of the following?

Sprain

Which of the following terms refers to an injury to ligaments and other soft tissues of a joint?

Sprain

A nurse is caring for a client receiving chemotherapy. Which assessment finding places the client at the greatest risk for an infection?

Stage 3 pressure ulcer on the left heel

Sally Ortez, a 10-year-old female, is returning to the pediatric oncology group where you practice nursing. Sally has been diagnosed with cancer which has metastasized to her bones. During client education, Sally's mom asks about tumor staging and it's relation to Sally's condition. What stage would you expect Sally's tumor to be assigned?

Stage IV

Most cases of osteomyelitis are caused by which of the following microorganisms?

Staphylococcus

Most cases of osteomyelitis are caused by which of the following microorganisms? a) Proteus species b) Escherichia coli c) Pseudomonas species d) Staphylococcus

Staphylococcus Explanation: Staphylococcus aureus causes 70% to 80% of bone infections. Proteus species are frequently found in osteomyelitis, but they do not cause the majority of bone infections. Pseudomonas species are frequently found in osteomyelitis, but they do not cause most bone infections. While E. coli is frequently found in osteomyelitis, it does not cause the majority of bone infections.

A client is admitted to an acute care facility with osteomyelitis. Which organism usually causes this infection?

Staphylococcus aureus

A client is diagnosed with osteomyelitis. This is most commonly caused by which of the following?

Staphylococcus aureus

A client is diagnosed with osteomyelitis. This is most commonly caused by which of the following? a) Staphylococcus aureus b) Proteus vulgaris c) Escherichia coli d) Psuedomonas aeruginosa

Staphylococcus aureus Explanation: Staphylococcus aureus causes over 50% of bone infections. Other organisms include Proteus vulgaris and Pseudomonas aeruginosa, as well as E. coli.

The client receiving a first infusion of paclitaxel reports chills and dyspnea. The oncology nurse assesses the client and observes facial flushing, wheezing, and extreme anxiety. What is the priority intervention by the nurse?

Stop the infusion.

A 45-year-old softball player arrives at the emergency department following his injury while sliding into a base during a game. After his examination and radiographs, the physician diagnoses muscle strain and prescribes appropriate treatment. What does the physician mean with the term "strain"?

Stretched or pulled beyond its capacity

An adult is swinging a small child by the arms, and the child screams and grabs his left arm. It is determined in the emergency department that the radial head is partially dislocated. What is this partially dislocated radial head documented as?

Subluxation

A patient is to start with chemotherapy. The patient is worried about going bald in the course of the treatment. How can the nurse assist the patient in being comfortable with his or her body image?

Suggest the use of a wig or cap.

A nurse is preparing a client for discharge after a prolonged hospitalization in which the client had a colon resection and colostomy formation for treatment of colon cancer. The client's family has concerns about managing his care at home. Which factor is most important in ensuring successful home care?

Support from friends and family

When caring for a client with acute osteomyelitis in the right tibia, which measure is most appropriate to implement when repositioning the client's leg?

Support the leg above and below the affected area when positioning. Explanation: Acute osteomyelitis can be very painful. Therefore, the extremity must be handled carefully and moved slowly. The most appropriate action when moving an extremity with acute osteomyelitis is to ensure that the extremity is carefully supported above and below the affected area. A splint may be useful to decrease discomfort. Holding the leg by the ankle or allowing the client to move the leg is inappropriate because doing so does not provide adequate support to the affected area. Applying warm, moist compresses does not decrease the need to adequately support the affected area.

A patient with chronic osteomyelitis has undergone 6 weeks of antibiotic therapy. There is no improvement in the wound appearance. What action would the nurse anticipate to promote healing?

Surgical debridement

In chronic osteomyelitis, antibiotics are adjunctive therapy in which of the following situations?

Surgical debridement

In chronic osteomyelitis, antibiotics are adjunctive therapy in which of the following situations? a) Wound packing b) Surgical debridement c) Vitamin supplements d) Wound irrigation

Surgical debridement Explanation: In chronic osteomyelitis, antibiotics are adjunctive therapy to surgical debridement.

A patient with chronic osteomyelitis has undergone 6 weeks of antibiotic therapy. There is no improvement in the wound appearance. What action would the nurse anticipate to promote healing? a) Wound irrigation b) Surgical debridement c) Wound packing d) Vitamin supplements

Surgical debridement Explanation: In chronic osteomyelitis, surgical debridement is used when the wound fails to respond to antibiotic therapy. Wound packing, vitamin supplements, and wound irrigation are not the standard of care when treating chronic osteomyelitis.

The nurse is gathering objective data for a client at the clinic complaining of arthritic pain in the hands. The nurse observes that the fingers are hyperextended at the proximal interphalangeal joint with fixed flexion of the distal interphalangeal joint. What does the nurse recognize this deformity as?

Swan neck deformity

Morton's neuroma is exhibited by which of the following clinical manifestations?

Swelling of the third (lateral) branch of the median plantar nerve

Morton's neuroma is exhibited by which of the following clinical manifestations? a) Swelling of the third (lateral) branch of the median plantar nerve b) Inflammation of the foot-supporting fascia c) High arm and a fixed equinus deformity d) Longitudinal arch of the foot is diminished

Swelling of the third (lateral) branch of the median plantar nerve Explanation: Morton's neuroma is swelling of the third branch of the median plantar nerve. Pes cavus refers to a foot with an abnormally high arch and a fixed equinus deformity of the forefoot. Flatfoot is a common disorder in which the longitudinal arch of the foot is diminished. Plantar fasciitis is an inflammation of the foot-supporting fascia.

Which oncologic emergency involves the failure in the negative feedback mechanism that normally regulates the release of antidiuretic hormone (ADH)?

Syndrome of inappropriate antidiuretic hormone release (SIADH)

Synovial joints

Synovial joints (e.g., shoulders, wrists, hips, knees, ankles) contain a space between the bones that is filled with synovial fluid, a lubricant that promotes a sliding movement of the ends of the bones

While assessing an adult client's jaw, the nurse hears a clicking popping sound, and the client expresses pain in the joint. The nurse should further assess the client for

TMJ dysfunction

A 42-year-old female with breast cancer has had a radical mastectomy. She will have radiation therapy and then begin chemotherapy. Drug therapy will consist of a combination of doxorubicin, cyclophosphamide, and paclitaxel. What will the nurse include in the teaching plan concerning the drug therapy?

Take special care when shaving or brushing her teeth.

A client had a total abdominal hysterectomy and bilateral oophorectomy for ovarian carcinoma yesterday. She received 2 mg of morphine sulfate I.V. by patient-controlled analgesia (PCA) 10 minutes ago. The nurse was assisting her from the bed to a chair when the client felt dizzy and fell into the chair. The nurse should:

Take the client's blood pressure. The nurse should take the client's blood pressure. She is likely experiencing orthostatic hypotension. The PCA pump does not need to be discontinued because, as soon as the blood pressure stabilizes, the pain medication can be resumed. Administering oxygen is not necessary unless the oxygen saturation also drops. The client should sit in the chair until the blood pressure stabilizes

A nurse is performing discharge teaching for an elderly client with osteoporosis. Which instruction about taking a calcium supplement should the nurse include?

Take the supplement with meals or with orange juice.

A nurse is performing discharge teaching for an elderly client with osteoporosis. Which instruction about taking a calcium supplement should the nurse include? a) Take weekly on the same day and at the same time. b) Remain in an upright position 30 minutes after taking the supplement. c) Take the supplement on an empty stomach with a full glass of water. d) Take the supplement with meals or with orange juice.

Take the supplement with meals or with orange juice. Explanation: Calcium supplements, such as Caltrate or Citracal, are over-the-counter medications. They should be taken with meals or with a beverage high in vitamin C

A 56-year-old woman is admitted for a modified radical mastectomy. The client appears anxious and asks many questions. The nurse's best course of action is to:

Tell the client as much as she wants to know and is able to understand. An important nursing responsibility is preoperative teaching. The recommended guide for teaching is to tell the client as much as she wants to know and is able to understand. Delaying discussion of issues or concerns will most likely increase the client's anxiety. Telling the client to discuss questions with the physician avoids acknowledging the client's concerns.

A client was recently enrolled in a clinical trial for lung cancer. The client's health insurance provider asks the nurse caring for the client about the client's status, treatment regimen, and possible adverse effects of the medication she is taking. How can the nurse best respond?

Tell the provider that a Certificate of Confidentiality was issued; therefore, no information can be released.

A client has been treated for migraine headaches for several months and comes to the clinic stating he is getting no better. The nurse is talking with the client and hears an audible click when the client is moving his jaw. What does the nurse suspect may be happening?

Temporomandibular disorder

A client has been treated for migraine headaches for several months and comes to the clinic stating he is getting no better. The nurse is talking with the client and hears an audible click when the client is moving his jaw. What does the nurse suspect may be happening? a) Dislocated jaw b) Temporomandibular disorder c) Trigeminal neuralgia d) Loose teeth

Temporomandibular disorder Explanation: The disorder can be confused with trigeminal neuralgia and migraine headaches. The client experiences clicking of the jaw when moving the joint, or the jaw can lock, which interferes with opening the mouth. Loose teeth will not cause a clicking of the jaw. The client does not have a dislocated jaw.

Temporomandibular joint syndrome (TMJ)

Temporomandibular joint syndrome is a very common cause of pain with chewing. Ischemic pain with chewing, or jaw claudication, can occur with temporal arteritis, but the lack of tenderness of the scalp overlying the artery makes this less likely. Trigeminal neuralgia can be associated with extreme tenderness over the branches of the trigeminal nerve. While a tumour of the mandible is possible, it is much less likely than the other choices

Gouty arthritis

Tender, painful, reddened, hot, and swollen metatarsophalangeal joint in the great (big) toe is seen in gouty arthritis. This is an inflammatory condition caused by an abnormal buildup of uric acid in the body that becomes deposited in the joints.

Tenderness

Tenderness implies an inflammatory process along with increased temperature. Nodules and ecchymosis are not typically associated with inflammatory processes

plantar fasciitis

Tenderness of the calcaneus of the bottom of the foot may indicate

The nurse is teaching a male client to perform monthly testicular self-examinations. Which of the following points would be appropriate to make?

Testicular cancer is a highly curable type of cancer.

A nurse is teaching a male client to perform monthly testicular self-examinations. Which point is appropriate to make?

Testicular cancer is a highly curable type of cancer. Explanation: Testicular cancer is highly curable, particularly when it's treated in its early stage. Self-examination allows early detection and facilitates the early initiation of treatment. The highest mortality rates from cancer among men are in men with lung cancer. Testicular cancer is found more commonly in younger, not older, men.

A 65-year-old client is scheduled for a right lower lobectomy for lung cancer. During the admission assessment, the client asks for information about a living will and advance directive. The nurse knows that the client understands her teaching about the living will when he says:

The advance directive allows me to state my health care wishes while I'm still able to do so.

After teaching a group of students about systemic lupus erythematosus, the instructor determines that the teaching was successful when the students state which of the following?

The belief is that it is an autoimmune disorder with an unknown trigger.

"A nurse is caring for a 25-year-old client with end-stage testicular cancer who has been referred to hospice care. Which of the following criteria excludes the client from hospice care?

The client entered a clinical trial through the National Cancer Institute

A client with suspected gastric cancer undergoes an endoscopy of the stomach. Which of the following assessments made after the procedure would indicate the development of a potential complication?

The client experiences a sudden increase in temperature.

Which information should a nurse include when consulting with a home health care agency about the nursing care and physical therapy needs of a client who will be discharged after undergoing total hip replacement surgery?

The client lives alone and will be restricted from driving for at least 6 weeks.

To treat cervical cancer, a client has had an applicator of radioactive material placed in the vagina. Which observation by the nurse indicates a radiation hazard?

The client receives a complete bed bath each morning.

The nurse is preparing a client for magnetic resonance imaging (MRI) to confirm or rule out a spinal cord lesion. During the MRI scan, which of the following would pose a threat to the client?

The client wears a watch and wedding band

The nurse is caring for the client with chronic osteomyelitis of the jaw with a draining wound. Which client goal is a priority for the client?

The client will experience a tolerable level of pain. The client will demonstrate wound care. The client will maintain adequate nutritional intake.

the subtalar joint

The joint in the foot is the subtalar

The knee joint is capable of

The knee joint is capable of flexion and extension

Cervical strain

The most common cause of neck pain is cervical strain. This can occur from sleeping in the wrong position, carrying a heavy load, or being in an automobile accident.

A nurse is caring for a client who's experiencing septic arthritis. This client has a history of immunosuppressive therapy and his immune system is currently depressed. Which assignment is the most appropriate for the nurse caring for this client?

The nurse is caring for this client on the intensive care unit.

A nurse is caring for a client who's experiencing septic arthritis. This client has a history of immunosuppressive therapy and his immune system is currently depressed. Which assignment is the most appropriate for the nurse caring for this client? a) The nurse caring for this client is also caring for four other immunosuppressed clients on the medical floor. b) The nurse caring for this client is also caring for four clients receiving chemotherapy for cancer treatment on the oncology floor. c) The nurse is caring for this client on the intensive care unit. d) The nurse caring for this client is also caring for two other immunosuppressed clients on the medical intensive care unit.

The nurse is caring for this client on the intensive care unit. Explanation: This client is critically ill; his diagnosis and immunosuppression place him at a high risk for infection. The most appropriate place for this client is in an intensive care unit, where the nurse can focus exclusively on his health promotion. This client shouldn't be on the oncology floor. This client requires close monitoring. The nurse caring for this client shouldn't also be caring for other clients who may require frequent interventions

An older adult woman has been diagnosed with acute lymphoblastic leukemia (ALL) and her care team has identified potential benefits of imatinib. Which characteristic of this patient's current health status may preclude the use of imatinib?

The patient has chronic heart failure resulting in significant peripheral edema.

A patient taking high doses of cyclophosphamide is exhibiting muscle cramps and confusion and reports headaches. The nurse assesses the laboratory tests and notices the sodium level is low. What does the nurse determine may be occurring with this patient?

The patient is exhibiting signs of syndrome of inappropriate antidiuretic hormone caused by the high dose of cyclophosphamide.

The nurse is educating a patient with lower back pain on proper lifting techniques. The nurse would document what behavior as evidence the education was effective?

The patient placed the load close to the body.

The nurse is educating a patient with lower back pain on proper lifting techniques. The nurse would document what behavior as evidence the education was effective? a) The patient used a narrow base of support. b) The patient bent at the hips and tightened the abdominal muscles. c) The patient placed the load close to the body. d) The patient reached over head with arms fully extended.

The patient placed the load close to the body. Explanation: Instructions for the patient with low back pain should include that when lifting, the patient should avoid overreaching. The patient should also keep the load close to the body, bend the knees and tighten the abdominal muscles, use a wide base of support, and use a back brace to protect the back. Bending at the hips increases the strain on the back muscles when lifting.

The nurse is educating a patient with lower back pain on proper lifting techniques. The nurse would document what behavior as evidence the education was effective? a) The patient used a narrow base of support. b) The patient placed the load close to the body. c) The patient reached over head with arms fully extended. d) The patient bent at the hips and tightened the abdominal muscles.

The patient placed the load close to the body. Explanation: Instructions for the patient with low back pain should include that when lifting, the patient should avoid overreaching. The patient should also keep the load close to the body, bend the knees and tighten the abdominal muscles, use a wide base of support, and use a back brace to protect the back. Bending at the hips increases the strain on the back muscles when lifting.

A nurse is teaching a female client about preventing osteoporosis. Which teaching point is correct?

The recommended daily allowance of calcium may be found in a wide variety of foods. Explanation: Premenopausal women require 1,000 mg of calcium per day. Postmenopausal women require 1,500 mg per day. Clients usually can get the recommended daily requirement of calcium by eating a varied diet. Osteoporosis doesn't show up on ordinary X-rays until 30% of bone has been lost. Bone densitometry, however, can detect bone loss of 3% or less. This test is sometimes recommended routinely for women older than 35 who are at risk for osteoporosis. Strenuous exercise won't cause fractures. Although supplements are available, they aren't always necessary

A nurse is teaching a female client about preventing osteoporosis. Which teaching point is correct? a) Obtaining the recommended daily allowance of calcium requires taking a calcium supplement. b) To prevent fractures, the client should avoid strenuous exercise. c) Obtaining an X-ray of the bones every 3 years is recommended to detect bone loss. d) The recommended daily allowance of calcium may be found in a wide variety of foods.

The recommended daily allowance of calcium may be found in a wide variety of foods. Explanation: Premenopausal women require 1,000 mg of calcium per day. Postmenopausal women require 1,500 mg per day. Clients usually can get the recommended daily requirement of calcium by eating a varied diet. Osteoporosis doesn't show up on ordinary X-rays until 30% of bone has been lost. Bone densitometry, however, can detect bone loss of 3% or less. This test is sometimes recommended routinely for women older than 35 who are at risk for osteoporosis. Strenuous exercise won't cause fractures. Although supplements are available, they aren't always necessary.

A 36-year-old man is receiving three different chemotherapeutic agents for Hodgkin's disease. The nurse explains to the client that the three drugs are given over an extended period because:

The three drugs have a synergistic effect and act on the cancer cells with different mechanisms.

Which of the following statements is most accurate regarding the long-term toxic effects of cancer treatments on the immune system?

The use of radiation and combination chemotherapy can result in more frequent and more severe immune system impairment. Studies of long-term immunologic effects in clients treated for leukemia, Hodgkin's disease, and breast cancer reveal that combination treatments of chemotherapy and radiation can cause overall bone marrow suppression, decreased leukocyte counts, and profound immunosuppression. Persistent and severe immunologic impairment may follow radiation and chemotherapy (especially multiagent therapy). There is no evidence of greater risk of infection in clients with persistent immunologic abnormalities. Suppressor T cells recover more rapidly than the helper T cells.

After receiving chemotherapy for lung cancer, a client's platelet count falls to 98,000/μl. What term should the nurse use to refer to this drop in the platelet count?

Thrombocytopenia

Which sign may be helpful in identifying carpal tunnel syndrome?

Tinel's

Which sign may be helpful in identifying carpal tunnel syndrome? a) Kernig's b) Brudzinski's c) Babinski's d) Tinel's

Tinel's Explanation: Tinel's sign may be used to help identify carpal tunnel syndrome. The presence of the Babinski's sign can identify disease of the brain and spinal cord in adults and also exists as a primitive reflex in infants. The Brudzinski's and Kernig's sign are indicative of meningeal irritation.

A client receiving radiation therapy for thyroid cancer reports mouth and throat pain. While inspecting the mouth and throat, the nurse notices white patches and ulcerations in the oral mucosa. The nurse notifies the radiation oncologist, and expects which intervention for this client?

To administer an antifungal agent

A client receiving radiation therapy for thyroid cancer reports mouth and throat pain. While inspecting the mouth and throat, the nurse notices white patches and ulcerations in the oral mucosa. The nurse notifies the radiation oncologist, and expects which intervention for this client?

To administer an antifungal agent Correct Explanation: White patches and ulcers in the mouth and throat suggest Candidiasis, which is common in immunocompromised clients and in those receiving radiation therapy. It is treated with antifungal agents, as well as by offering yogurt and frequent medicated mouthwash rinses. Oral intake should be encouraged to maintain hydration. Weight loss may occur if left untreated, there is no indication that a daily weight is required at this time. Referring the client to a dentist may be necessary if the ulcers are due to something other than Candidiasis

Pes varus

Toes that point in are termed pes varus

A client with metastatic cancer is experiencing neuropathic pain. Which alternative therapy is most beneficial in treating this type of pain?

Transcutaneous electrical nerve stimulation (TENS)

A 50-year-old woman presents with "left hip pain" of several weeks duration. She is exquisitely tender when the nurse presses over her proximal lateral thigh. What is her most likely health problem?

Trochanteric bursitis

An adverse effect of tamoxifen therapy involves menopausal symptoms.

True

Anemia from chemotherapy treatment is the result of bone marrow suppression.

True

A client is scheduled for a laminectomy to repair a herniated intervertebral disk. When developing the postoperative plan of care, the nurse should include which action?

Turning the client from side to side, using the logroll technique

Which laboratory study is most relevant to treating a client who has sustained a pelvic fracture?

Type and crossmatch

Muscle strength test

Upon assessment, the nurse finds that the client has a slight flicker of contraction in the muscles on the right side. What should the nurse document as the muscle strength rating? The nurse should rate the muscle strength as 1. Muscle rating 4 is given when the client is able to perform active motion against some resistance. When the client is able to perform active movements against gravity, the muscle strength is graded as 3. If the client is able to perform passive ROM, the muscle strength is rated as 2

Your client is receiving radiation therapy. The client asks you about oral hygiene. What advice regarding oral hygiene should you offer?

Use a soft toothbrush and avoid an electronic toothbrush.

A nurse is working with a dying client and the client's family. Which communication technique is most important to use?

Use active listening and silence when communicating.

The client has sustained a fracture of the left tibia. The extremity is immobilized using an external fixation device. Which postoperative instruction will the nurse plan to include in this client's teaching plan?

Use pain medication as prescribed to control pain. Correct: The client should be taught the correct use of prescribed pain medication to control pain adequately.

A client with low back pain is being seen in the clinic. In planning care, which teaching point should the nurse include?

Use the large muscles of the leg when lifting items.

A client with low back pain is being seen in the clinic. In planning care, which teaching point should the nurse include? a) Sleep on the stomach to alleviate pressure on the back. b) Use the large muscles of the leg when lifting items. c) Avoid twisting and flexion activities. d) A soft mattress is most supportive by conforming to the body.

Use the large muscles of the leg when lifting items. Explanation: The large muscles of the leg should be used when lifting.

The client has a grade III compound fracture of the right tibia. To prevent infection, which intervention will the nurse implement?

Using strict aseptic technique when cleaning the site Correct: Using aseptic technique is the best way to prevent infection.

Verruca vulgaris

Verruca vulgaris are painful warts that often occur under a callus

A client with advanced breast cancer is prescribed tamoxifen (Nolvadex). When teaching the client about this drug, the nurse should emphasize the importance of reporting which adverse reaction immediately?

Vision changes

A male patient with a musculoskeletal injury is instructed to alter his diet. The objective of this diet alteration is to facilitate the absorption of calcium from food and supplements. Considering the food intake objective, which of the following food items should the nurse encourage the patient to include in the diet?

Vitamin D fortified milk

A male patient with a musculoskeletal injury is instructed to alter his diet. The objective of this diet alteration is to facilitate the absorption of calcium from food and supplements. Considering the food intake objective, which of the following food items should the nurse encourage the patient to include in the diet? a) Vitamin D-fortified milk b) Bananas c) Red meat d) Green vegetables

Vitamin D-fortified milk Explanation: The nurse should advise the patient to include dietary sources of vitamin D, such as fatty fish, vitamin D-fortified milk, and cereals. These foods protect against bone loss and decrease the risk of fracture by facilitating the absorption of calcium from food and supplements. Red meat, bananas, and green vegetables do not facilitate calcium absorption from food and supplements.

Which of the following may occur if a client experiences compartment syndrome in an upper extremity?

Volkmann's contracture

The nurse teaches the patient with a high risk for osteoporosis about risk-lowering strategies including which of the following actions?

Walk or perform weight-bearing exercises outdoors

The nurse teaches the patient with a high risk for osteoporosis about risk-lowering strategies including which of the following actions? a) Walk or perform weight-bearing exercises outdoors b) Decrease the intake of vitamin A and D c) Increase fiber in the diet d) Reduce stress

Walk or perform weight-bearing exercises outdoors Explanation: Risk-lowering strategies for osteoporosis include walking or exercising outdoors, performing a regular weight-bearing exercise regimen, increasing dietary calcium and vitamin D intake, smoking cessation, and consuming alcohol and caffeine consumption in moderation.

A nurse is planning discharge teaching regarding exercise for a client at risk for osteoporosis. Which of the following exercises would be appropriate?

Walking

A nurse is planning discharge teaching regarding exercise for a client at risk for osteoporosis. Which of the following exercises would be appropriate? a) Bicycling b) Walking c) Yoga d) Swimming

Walking Explanation: Weight-bearing exercises should be incorporated into the client's lifestyle activities.

The nurse is instructing a local community group about ways to reduce the risk for musculoskeletal injury. What information will the nurse include in the teaching plan?

Wear helmets when riding a motorcycle. Correct: Those who ride motorcycles or bicycles should wear helmets to prevent head injury.

A nurse is caring for a patient who is at risk for erythema during antineoplastic drug therapy. What should the nurse suggest to this patient?

Wear loose protective clothing.

The nurse is assisting a client with removing shoes prior to an examination and observes that the client has a flexion deformity of several toes on both feet of the proximal interphalangeal (PIP) joints. What can the nurse encourage the client to do?

Wear properly fitting shoes.

The nurse is assisting a client with removing shoes prior to an examination and observes that the client has a flexion deformity of several toes on both feet of the proximal interphalangeal (PIP) joints. What can the nurse encourage the client to do? a) Wear properly fitting shoes. b) Do active range of motion on the toes. c) Have surgery to fix them. d) Bind the toes so that they will straighten.

Wear properly fitting shoes. Explanation: Hammer toe is a flexion deformity of the PIP joint and may involve several toes and may result from wearing poorly fitting shoes. They will not straighten by binding the toes or doing active range of motion exercises. Surgery is an option but should be discussed with an orthopedic surgeon or podiatrist.

A client received chemotherapy 24 hours ago. Which precautions are necessary when caring for this client?

Wearing personal protective equipment when handling blood, body fluids, or feces

Herniated disc

When the client performs straight leg flexion, the client complains of pain that radiates down his leg Straight leg flexion that produces back and leg pain radiating down the leg may indicate a herniated disc

A client is preparing for discharge from the emergency department after sustaining an ankle sprain. The client is instructed to avoid weight bearing on the affected leg and is given crutches. After instruction, the client demonstrates proper crutch use in the hallway. What additional information is most important to know before discharging the client?

Whether the client needs to navigate stairs routinely at home Explanation: Knowing whether the client must routinely navigate steps at home is most important. If the client must navigate steps, special crutch-walking techniques must be taught to safely navigate the stairs. Although pets, parking on the street, and driving a car with a stick shift can pose problems for the client, these factors aren't important to know before discharging the client with crutches.

After surgery to treat a hip fracture, a client returns from the postanesthesia care unit to the medical-surgical unit. Postoperatively, how should the nurse position the client?

With the leg on the affected side abducted

A client in the final stages of terminal cancer tells his nurse: ""I wish I could just be allowed to die. I'm tired of fighting this illness. I have lived a good life. I only continue my chemotherapy and radiation treatments because my family wants me to."" What is the nurse's best response?

Would you like to meet with your family and your physician about this matter?

A 22-year-old client with an external fixation device attached to his left thigh is unable to bear weight on his left leg. He asks a nurse if he can take a shower. How should the nurse respond to the client's request?

Wrap the device with plastic and then assist the client into the shower using a wheelchair.

The nurse is providing breast cancer education at a community facility. The American Cancer Society recommends that women get mammograms:

Yearly after age 40.

A client has malignant pleural effusions. The nurse should conduct a focused assessment to determine if the client has which of the following? Select all that apply. a) Chest pain. b) Weight gain. c) Dyspnea. d) Peripheral edema. e) Hiccups. f) Cough.

a) Chest pain. c) Dyspnea.

A 17-year-old client has been admitted to the hospital for a biopsy to confirm the diagnosis of bone cancer. The nurse should assess the client for which conditions? Select all that apply. a) Cough b) dypsnea c) fever d) anorexia e) decreased range of motion f) pain g) swelling

a) Cough b) dypsnea f) pain g) swelling

cardiogenic shock includes (select all that apply): a. dobutamine to increase myocardial contractility. b. vasopressors to increase systemic vascular resistance. c. circulatory assist devices such as an intraaortic balloon pump. d. corticosteroids to stabilize the cell wall in the infarcted myocardium. e. Trendelenburg positioning to facilitate venous return and increase preload.

a. dobutamine to increase myocardial contractility. c. circulatory assist devices such as an intraaortic balloon pump.

The mother of a 5-year-old child asks the nurse questions regarding the importance of vigilant use of sunscreen. Which information is most important for the nurse to convey to the mother? a.) Appropriate use of sunscreen decreases the risk of skin cancer. b.) Repeated exposure to the sun causes premature aging of the skin. c.) A child's skin is delicate, and burns easily. d.) In addition to causing skin cancer, repeated sun exposure predisposes the child to other forms of cancer.

a.) Appropriate use of sunscreen decreases the risk of skin cancer.

To assess the joints, the nurse asks a client to perform various movements. As the client moves the arm away from the midline, the nurse evaluates the ability to perform:

abduction.

To assess the joints, a nurse asks a client to perform various movements. As the client moves his arm away from the midline, the nurse evaluates his ability to perform:

abduction. Explanation: A client performs abduction when moving a body part away from the midline. Protraction refers to drawing out or lengthening of a body part. Retraction, the opposite of protraction, refers to drawing back or shortening of a body part. Adduction, the opposite of abduction, is movement of a body part toward the midline.

When the client who has had a hip replacement is lying on the side, the nurse should place pillows or an abductor splint between the legs to prevent:

adduction of the hip joint. Explanation: After hip replacement surgery, the client should be positioned on the nonoperative side with pillows or an abductor splint between the legs to help prevent adduction of the operative leg. This positioning places the hip in proper alignment. Dislocation of the hip can occur if the leg on the affected side is allowed to adduct.

The nurse is managing the care of a client with osteoarthritis. Appropriate treatment strategies for osteoarthritis include:

administration of nonsteroidal anti-inflammatory drugs (NSAIDs).

The nurse is teaching a client with a long leg cast how to use crutches properly while descending a staircase. The nurse should tell the client to:

advance both crutches first.

The nurse is assessing a client with possible osteoarthritis. The most significant risk factor for osteoarthritis is:

age

The development of laryngeal cancer is most clearly linked to which of the following factors?

alcohol and tobacco use Predisposing factors for laryngeal cancer include chronic irritants such as alcohol, tobacco, and exposure to noxious fumes. About 75% of people who develop laryngeal cancer are smokers. The combination of smoking and heavy alcohol intake is even more strongly implicated as a causative agent in laryngeal cancer. Epidemiologic studies indicate that a high-fat diet may be a major factor in the development of cancer of the breast, prostate, and colon, but not laryngeal cancer. Low socioeconomic status is a predisposing factor in cervical cancer but not laryngeal cancer. Artificial sweeteners have been related to the incidence of bladder cancer, but not laryngeal cancer.

Instructions for the patient with low back pain include that when lifting the patient should

avoid overreaching.

Instructions for the patient with low back pain include that when lifting the patient should a) bend the knees and loosen the abdominal muscles. b) avoid overreaching. c) place the load away from the body. d) use a narrow base of support.

avoid overreaching. Explanation: Instructions for the patient with low back pain should include that when lifting, the patient should avoid overreaching. The patient should also keep the load close to the body, bend the knees and tighten the abdominal muscles, use a wide base of support, and use a back brace to protect the back. When lifting, the patient with low back pain should keep the load close to the body. When lifting, the patient with low back pain should bend the knees and tighten the abdominal muscles.

Which information noted by the nurse reviewing the laboratory result of a patient who is receiving chemotherapy is most important report to the health care provider? a) Hemoglobin of 10g/l b) WBC of 1700/ul c) Platelet of 65,000/ul d) Serum creatinine level of 1.2mg/dl

b) WBC of 1700/ul

A Nurse is planning an education program on breast cancer for a community group of women who are of child- bearing age. The nurse plans to base the discussion on the American and Canadian Association guidelines. which of the following should the nurse plan to include in the discussion? select all that apply. a) Women should get mammograms every 3 years between the ages of 22 and 30 and then annually b)Women should begin getting annual mammograms beginning at 40 years of age c) Women between 40 and 60 years of age only need a mammogram every 5 years d)Women should obtain a mammogram after the birth of their of their first child and every 2 years after e) Women who have a family history of breast cancer should obtain their first mammogram at the of 35.

b)Women should begin getting annual mammograms beginning at 40 years of age e) Women who have a family history of breast cancer should obtain their first mammogram at the of 35.

Which statement by a patient who is scheduled for a needle biopsy of the prostate indicates that the patient understands the purpose of a biopsy? a. "The biopsy will tell the doctor whether the cancer has spread to my other organs." b. "The biopsy will help the doctor decide what treatment to use for my enlarged prostate." c. "The biopsy will determine how much longer I have to live." d. "The biopsy will indicate the effect of the cancer on my life."

b. "The biopsy will help the doctor decide what treatment to use for my enlarged prostate."

A patient who has been involved in a motor-vehicle crash is admitted to the ED with cool, clammy skin, tachycardia, and hypotension. All of these orders are written. Which one will the nurse act on first? a. Insert two 14-gauge IV catheters. b. Administer oxygen at 100% per non-rebreather mask. c. Place the patient on continuous cardiac monitor. d. Draw blood to type and crossmatch for transfusions.

b. Administer oxygen at 100% per non-rebreather mask.

Flexion

bending the extremity at the joint and decreasing the angle of the joint

To assess abduction of the shoulders and arms, a nurse should ask a client to:

bring both hands together overhead

A client has been diagnosed with sepsis. The nurse will most likely find which of the following when assessing this client:(SATA) a. Lactic acidosis. b. Mental status changes. c. Elevated temperature. d. Severe hypotension. e. Rapid shallow respirations. f. Oliguria.

c. Elevated temperature. e. Rapid shallow respirations.

While giving care to a client with an internal cervical radiation implant, the nurse finds the implant in the bed. The nurse should take which initial action? a. Pick up the implant with gloved hands and flush it down the toilet. b. Reinsert the implant into the vagina. c. Pick up the implant with long-handled forceps and place it in a lead container. d. Call the health care provider

c. Pick up the implant with long-handled forceps and place it in a lead container.

When caring for a patient with cardiogenic shock and possible MODS, which information obtained by the nurse will help confirm the diagnosis of MODS? a. The patient has crackles throughout both lung fields. b. The patient complains of 8/10 crushing chest pain. c. The patient has an elevated ammonia level and confusion. d. The patient has cool extremities and weak pedal pulses.

c. The patient has an elevated ammonia level and confusion.

A nurse is performing a home visit for a client who received chemotherapy within the past 24 hours. The nurse observes a small child playing in the bathroom, where the toilet lid has been left up. Based on these observations, the nurse modifies the client's teaching plan to include:

chemotherapy exposure and risk factors.

A nurse is performing a home visit for a client who received chemotherapy within the past 24 hours. The nurse observes a small child playing in the bathroom, where the toilet lid has been left up. Based on these observations, the nurse modifies the client's teaching plan to include:

chemotherapy exposure and risk factors. The raised toilet lid exposes the child playing in the bathroom to the risk of inhaling or ingesting chemotherapy agents. The nurse should modify her teaching plan to include content related to chemotherapy exposure and its associated risk factors. Because the client has received chemotherapy, the plan should already include information about expected adverse effects, signs and symptoms of infection, and reinforcement of the medication regimen.

A public health nurse is teaching a class for social workers serving at-risk women from lower socioeconomic backgrounds about breast cancer prevention. A learner asks what he/she can do to better help these women on a daily basis. The nurse advises that the most effective way of helping clients is to do what? Select all that apply. a) Assist the women in finding local healthcare agencies. b) Provide information on access to health insurance. c) Educate the women on various support groups close to their homes. d) Provide education on breast cancer self exams. e) Provide information on free screening programs. ans/ rationale

d) Provide education on breast cancer self exams. e) Provide information on free screening programs. ans/ rationale

The removal of entire breast, pectoralis major and minor muscles and neck lymph nodes which is followed by skin grafting is a procedure called: a. Simple mastectomy b. Modified radical mastectomy c. Radiation therapy d. Radical mastectomy

d. Radical mastectomy

A patient with ovarian cancer tells the nurse, "I don't think my husband cares about me anymore. He rarely visits me." On one occasion when the husband was present, he told the nurse he just could not stand to see his wife so ill and never knew what to say to her. An appropriate nursing diagnosis in this situation is: a. compromised family coping related to disruption in lifestyle and role changes b. impaired home maintenance related to perceived role changes c. risk for caregiver role strain related to burdens of caregiving responsibilities d. interrupted family processes related to effect of illness on family members

d. interrupted family processes related to effect of illness on family members

To help minimize calcium loss from a hospitalized client's bones, the nurse should: a) provide supplemental feedings between meals. b) provide the client dairy products at frequent intervals. c) encourage the client to walk in the hall. d) reposition the client every 2 hours.

encourage the client to walk in the hall. Explanation: Calcium absorption diminishes with reduced physical activity because of decreased bone stimulation. Therefore, encouraging the client to increase physical activity, such as by walking in the hall, helps minimize calcium loss. Turning or repositioning the client every 2 hours wouldn't increase activity sufficiently to minimize bone loss. Providing dairy products and supplemental feedings wouldn't lessen calcium loss — even if the dairy products and feedings contained extra calcium — because the additional calcium doesn't increase bone stimulation or osteoblast activity.

A client who has had a total laryngectomy appears withdrawn and depressed. He keeps the curtain drawn, refuses visitors, and indicates a desire to be left alone. Which nursing intervention would be most therapeutic for the client?

encouraging him to express his feelings nonverbally and in writing The client has undergone body changes and permanent loss of verbal communication. He may feel isolated and insecure. The nurse can encourage him to express his feelings and use this information to develop an appropriate plan of care. Discussing the client's behavior with his wife may not reveal his feelings. Exploring future plans is not appropriate at this time because more information about the client's behavior is needed before proceeding to this level. The nurse can respect the client's need for privacy while also encouraging him to express his feelings

A nurse is caring for a female client following a motor vehicle accident resulting in paraplegia. The client is ready for discharge to home with her husband, who states, "I'm scared to carry her because I'm afraid I'll either hurt my back or drop her." A nurse identifies a need for discharge teaching of the husband in regard to: a) nutritional changes for the client with paraplegia. b) ergonomic principles and body mechanics. c) the importance of monitoring urinary elimination. d) signs and symptoms of chronic back pain that he should report to his physician.

ergonomic principles and body mechanics. Explanation: The husband's statement indicates a need for teaching in regard to client mobility and transfer techniques. Although urinary elimination, nutrition, and pain are components of care for clients with paraplegia, education about ergonomic principles and body mechanics is most appropriate at this time based on the husband's statement.

The client with a laryngectomy does not want to be observed by the family because the opening in the throat is "disgusting." The nurse should:

explore why the client believed the stoma is "disgusting." Explanation: Changes in body image are expected after a laryngectomy, and the nurse should first explore what is upsetting the client the most at this time. Many clients are concerned about how their family members will respond to the physical changes that have occurred as a result of a laryngectomy, but discussing the importance of family support is not helpful; instead, the nurse should allow the client to communicate any negative feelings or concerns that exist because of the surgery. The client's feelings are not related to a knowledge deficit, and therefore, it is too early to begin teaching about stoma care. It is also not helpful to offer reassurances about the change in appearance; the client will require time to adjust to the changed body image.

Two days after being placed in a cast for a fractured femur, the client suddenly has chest pain and dyspnea. The client is confused and has an elevated temperature. The nurse should assess the client for:

fat embolism syndrome. Explanation: Clients with fractures of the long bones such as the femur are particularly susceptible to fat embolism syndrome (FES). Signs and symptoms include chest pain, dyspnea, tachycardia, and cyanosis. Changes in mental status are caused by hypoxemia and can be the first symptoms noted in FES. The client can also be restless and febrile and can develop petechiae. Osteomyelitis is infection of the bone; signs and symptoms of osteomyelitis do not include respiratory symptoms. Compartment syndrome causes signs of localized neurovascular impairment, not systemic symptoms. Venous thrombosis occurs in the lower extremities and is caused by venous stasis.

McMurray's test

flex the knee and hip while in a supine position. Explanation: If the client complains of a "giving in" or "locking" of the knee, perform McMurray's test. With the client in the supine position, ask the client to flex one knee and hip. Then place your thumb and index finger of one hand on either side of the knee. Use your other hand to hold the heel of the foot up. Rotate the lower leg and foot laterally. Slowly extend the knee, noting pain or clicking. Repeat, rotating lower leg and foot medially. Again, note pain or clicking

Passive range-of-motion (ROM) exercises for the legs and assisted ROM exercises for the arms are part of the care regimen for a client with a spinal cord injury. Which observation by the nurse would indicate a successful outcome of this treatment?

free, easy movement of the joints Explanation: ROM exercises help preserve joint motion and stimulate circulation. Contractures develop rapidly in clients with spinal cord injuries, and the absence of this complication indicates treatment success. Range of motion will keep the ankle joints freely mobile. Footdrop, however, is prevented by proper positioning of the ankle and foot, which is usually accomplished with high-top sneakers or splints. External rotation of the hips is prevented by using trochanter rolls. Local ischemia over bony prominences is prevented by following a regular turning schedule.

A nurse is assessing a client with metastatic lung cancer. The nurse should assess the client specifically for:

hoarseness

A nurse is assessing a client with metastatic lung cancer. The nurse should assess the client specifically for:

hoarseness. Explanation: Hoarseness may indicate metastatic disease to the recurrent laryngeal nerve and is commonly noted with left upper lobe lung tumors. Diarrhea and constipation are not associated with lung cancer. Weight loss, not weight gain, can be a symptom of extensive disease.

A client with metastatic brain cancer is admitted to the oncology floor. According to the Self-Determination Act of 1991 concerning the execution of an advance directive, the hospital is required to:

inform the client or legal guardian of their rights to execute an advance directive. The client Self-Determination Act of 1991 requires all health care facilities to notify clients upon admission of their right to execute an advance directive. The facility's ethics committee can decide on a treatment plan if the client is unable and a health care power of attorney hasn't been appointed. Hospital employees aren't required by law to respect an individual's moral rights; however, the health care professional should respect the client's individual rights as part of his professional responsibility. Health care professionals are sometimes concerned that advance directives prevent treatment that might help the client. However, the hospital isn't required to advise clients not to execute their advance directive.

The nurse is developing a teaching plan for a client diagnosed with osteoarthritis. To minimize the risk of injury to the osteoarthritic client, the nurse should instruct the client to:

install safety devices in the home.

The primary reason that a herpes simplex virus (HSV) infection is a serious concern to a client with human immunodeficiency virus (HIV) infection is that it:

is an acquired immunodeficiency virus (AIDS)-defining illness. Explanation: HSV infection is one of a group of disorders that, when diagnosed in the presence of HIV infection, are considered to be diagnostic for AIDS. Other AIDS-defining illnesses include Kaposi's sarcoma; cytomegalovirus of the liver, spleen, or lymph nodes; and Pneumocystis carinii pneumonia. HSV infection is not curable and does not cause severe electrolyte imbalances. Human papillomavirus can lead to cervical cancer.

A nurse notices that a client has decreased range of motion with lateral bending of the cervical spine to the left side. What should the nurse do next in relation to this finding?

it is always important to compare both sides of the body for symmetry before making a judgment that data is abnormal. The nurse should then ask the client about previous injuries to the head and neck. All data must be properly documented in the client's record. If this finding is abnormal, the nurse should alert the health care provider for further orders

After surgery and insertion of a total hip prosthesis, a client develops severe sudden pain and an inability to move the extremity. The nurse interprets these findings as indicating:

joint dislocation. Explanation: The joint has dislocated when the client with a total joint prosthesis develops severe sudden pain and an inability to move the extremity. Clinical manifestations of an infection would include inflammation, redness, erythema, and possibly drainage and separation of the wound. Bleeding could be external (e.g., blood visible from the wound or on the dressing) or internal and manifested by signs of shock (e.g., pallor, coolness, hypotension, tachycardia). The seepage of glue into soft tissue would have occurred in the operating room, when the glue is still in the liquid form. The glue dries into the hard, fixed form before the wound is closed.

Which findings best correlate with a diagnosis of osteoarthritis?

joint stiffness that decreases with activity Explanation: A characteristic feature of osteoarthritis (degenerative joint disease) is joint stiffness that decreases with activity and movement. Erythema and edema over the affected joint, anorexia, weight loss, and fever and malaise are associated with rheumatoid arthritis, a more severe and destructive form of arthritis

As part of a primary cancer prevention program, an oncology nurse answers questions from the public at a health fair. When someone asks about laryngeal cancer, the nurse should explain that:

laryngeal cancer is one of the most preventable types of cancer.

Scoliosis

lateral curvature of the thoracic spine with an increase in the convexity on the curved side Scoliosis may cause elevation of one shoulder

cranial nerves

olfactory, optic, oculomotor, trochlear, trigeminal, abducens, facial, vestibulocochlear, glossopharyngeal, vagus, accessory, hypoglossal the 12 pairs of nerves emerging from the cranial cavity through various openings in the skull. Beginning with the most anterior, they are designated by Roman numerals and named (I) olfactory, (II) optic, (III) oculomotor, (IV) trochlear, (V) trigeminal, (VI) abducens, (VII) facial, (VIII) vestibulocochlear (acoustic), (IX) glossopharyngeal, (X) vagal, (XI) accessory, and (XII) hypoglossal. The cranial nerves originate in the base of the brain and carry impulses for such functions as smell, vision, ocular movement, pupil contraction, muscular sensibility, general sensibility, mastication, facial expression, glandular secretion, taste, cutaneous sensibility, hearing, equilibrium, swallowing, phonation, tongue movement, head movement, and shoulder movement. Certain cranial nerves, particularly V, VII, and VIII, contain two or more distinct functional components considered as independent nerves by some authorities.

Kyphosis

overcurvature of the spine in the thoracic and sacral spine, can result from arthritis, osteoporosis, or trauma.

A nurse is assessing a client with multiple myeloma. The nurse should keep in mind that clients with multiple myeloma are at risk for:

pathologic bone fractures Clients with multiple myeloma are at risk for pathologic bone fractures secondary to diffuse osteoporosis and osteolytic lesions. Also, clients are at risk for renal failure secondary to myeloma proteins by causing renal tubular obstruction. Liver failure and heart failure aren't usually sequelae of multiple myeloma. Hypoxemia isn't usually related to multiple myeloma.

Intravenous carmustine has been prescribed for a patient with cancer. The nurse should help relieve the discomfort of pain and burning during the infusion by

slowing the infusion.

Extension

straightening the extremity at the joint and increasing the angle of the joint

A client diagnosed with seminomatous testicular cancer expresses fear and questions the nurse about his prognosis. The nurse should base the response on the knowledge that:

testicular cancer has a cure rate of 90% when diagnosed early. Explanation: When diagnosed early and treated aggressively, testicular cancer has a cure rate of about 90%. Treatment of testicular cancer is based on tumor type, and seminoma cancer has the best prognosis. Modes of treatment include combinations of orchiectomy, radiation therapy, and chemotherapy. The chemotherapeutic regimen used currently is responsible for the successful treatment of testicular cancer. The nurse should not indicate to the client that the cancer will be cured, even though cure rates are high.

A client has the leg immobilized in a long leg cast. Which finding indicates the beginning of circulatory impairment?

tingling of toes Explanation: Tingling and numbness of the toes would be the earliest indication of circulatory impairment. Inability to move the toes and cyanosis are later indicators. Cast tightness should be investigated because cast tightness can lead to circulatory impairment; it is not, however, an indicator of impairment.

A client at risk for lung cancer asks about the reason for having a computed tomography (CT) scan as part of the initial exam. What is the nurse's best response? "A CT scan is:

useful for distinguishing small differences in tissue density and detecting nodal involvement."

The nurse is assessing a client who is suspected of being in the early symptomatic stages of human immunodeficiency virus (HIV) infection. Which indications of infection should the nurse detect during this stage?

whitish yellow patches in the mouth Explanation: Oropharyngeal candidiasis, or thrush, is the most common infection associated with the early symptomatic stages of HIV infection. Thrush is characterized by whitish yellow patches in the mouth. Various other opportunistic diseases can occur in clients with HIV infection, but they tend to occur later, after the diagnosis of acquired immunodeficiency syndrome has been made. Dyspnea can be indicative of pneumonia, which is caused by a variety of infective organisms. Bloody diarrhea is indicative of cytomegalovirus infection. Hyperpigmented lesions are indicators of Kaposi's sarcoma.

A client is undergoing a total prostatectomy for prostate cancer. The client asks questions about his sexual function. The best response by the nurse nurse is "Loss of the prostate gland means that:

you will be infertile and there will be no ejaculation. You can still experience the sensations of orgasm."

A 21-year-old client undergoes bone marrow aspiration at the clinic to establish a diagnosis of possible lymphoma. Which statement made by the client demonstrates proper understanding of discharge teaching? Select all that apply.

• "I will not be able to play basketball for the next 2 days." • "I will take acetaminophen for pain." • "I can apply an ice pack or a cold compress to the puncture site."

The nurse is caring for the client with chronic osteomyelitis of the jaw with a draining wound. Which nursing diagnosis is appropriate for the client? Select all that apply.

• Acute pain • Disturbed body image • Imbalanced nutrition: less than body requirements

Side effects of cancer treatment are the most common cause of cardiac tamponade, an emergency situation. Medical intervention will include pericardiocentesis. Nursing interventions would include which of the following? Select all that apply.

• Assessment for pulsus paradoxus • Maintaining the oxygen saturation of over 92% • Teaching the patient how to use pursed-lip breathing

A nurse preparing to discharge a client to his home determines that he will need IV antibiotics for 3 weeks. To prevent a delay in service, the nurse needs to carry out which of the following items? Select all that apply.

• Contact the home care agency and provide a detailed report on the client. • Fax the physician's orders and referral documents to the home care agency. • Obtain an order for insertion of a PICC (peripherally inserted central catheter). Explanation: In order to facilitate a timely discharge, the nurse will need to contact the home care agency and provide a detailed report on the client, fax the physician's orders and referral documents to the home care agency, and obtain an order for insertion of a PICC. It is the responsibility of the case manager to determine whether the client's insurance carrier will cover the item. The physician's orders for the antibiotics are sent to the home care agency, not to the hospital pharmacy. (less)

Which of the following are clinical manifestations of impingement syndrome? Select all that apply. a) Limited movement b) Atrophy c) Pain d) Shoulder tenderness e) Muscle spasms

• Pain • Shoulder tenderness • Limited movement • Muscle spasms • Atrophy Explanation: The patient experiences pain, shoulder tenderness, limited movement, muscle spasms, and atrophy. The process may progress to a rotator cuff tear.

The nurse is caring for the client with chronic osteomyelitis of the jaw with a draining wound. Which client goal is a priority for the client? a) The client will maintain adequate nutritional intake. b) The client will maintain effective airway clearance. c) The client will experience a tolerable level of pain. d) The client will demonstrate wound care. e) The client will remain free from injury.

• The client will experience a tolerable level of pain. • The client will demonstrate wound care. • The client will maintain adequate nutritional intake. Explanation: Pain is a priority problem for the client with osteomyelitis, and it can interfere with mobility of joint. In this situation, the client's jaw is the site of infection. Pain in this location can interfere with nutritional intake of the individual. Chronic osteomyelitis presents with a nonhealing ulcer over the infected bone with a connecting sinus that will intermittently and spontaneously drain pus. The client will need to be able to provide wound care in the home setting. Remaining free from injury and maintaining an effective airway clearance are not priority goals for the client.

A nurse is planning an education program on breast cancer for a community group of women who are of child-bearing age. The nurse plans to base the discussion on the American Cancer Association and Canadian Cancer Association guidelines. Which of the following should the nurse plan to include in the discussion? Select all that apply.

• Women who have a family history of breast cancer should obtain their first mammogram at the age of 35. • Women should begin getting annual mammograms beginning at 40 years of age. Explanation: The American Cancer Association and Canadian Cancer Association recommend that women should obtain a yearly mammogram at the age of 40 and those who have a family history of breast cancer in first degree relatives should obtain a baseline mammogram at the age of 35. Women between the ages of 22 and 30 should perform breast self-examinations and have a clinical breast exam every 3 years.

A client, 66 years old, has just been diagnosed with multiple myeloma (a cancer of the plasma) and will be initiating chemotherapy. The nurse, in an outpatient clinic, reviews the medications the client has been taking at home. The medications include pantoprazole (Protonix) for gastroesophageal reflux disease (GERD) and an over-the-counter calcium supplement to prevent osteoporosis. The nurse does the following interventions: (Select all that apply.)

• instructs the client to discontinue calcium • asks about nausea and vomiting • teaches the client to report abdominal or bone pain


Kaugnay na mga set ng pag-aaral

Indiquez de quelle(s) couleur(s) sont les choses suivantes.

View Set

Multiplication Facts for Jakob 2-12

View Set

Materials and Processing Exam 2 (Chap. 3 & 6)

View Set